Labor Relations Case Digests 1

You might also like

Download as docx, pdf, or txt
Download as docx, pdf, or txt
You are on page 1of 34

1

SamahanngManggagawasaHanjinvsBLR, G.R. No. 211145, Oct. 14, 2015 file employees of KNITJOY paid on a daily or piece-rate basis. It did not include regular rank-
and-file office and production employees paid on a monthly basis. Prior to the expiration of
FACTS: SamahanngManggagawasaHanjin Shipyard (Samahan) filed an application for the CBA, the Trade Union of the Philippines and Allied Services (TUPAS) filed a petition for
registration of its name with DOLE with the list of names of a total of 120 officers and the holding of a certification election among KNITJOY’s regular rank-and-file employees paid
members, signatures of the attendees of the meeting, copy of their Consi and by-laws. The on a daily and piece-rate basis. CFW (Confederation of Filipino Workers) won; thereafter
DOLE Regional Ofice No. 3, City of San Fernand, Pampanga, issued the corresponding negotiations for the CBA started between Knitjoy and CFW.
certificate of registration. Respondent Hanjin Heavy Industries and Construction Co., Ltd.
Philippines (Hanjin) prayed for the cancellation of registration of Samahan on the ground that During the pendency of the negotiations, Knitjoy Monthly Employees Union (KMEU) filed a
its members do not fall under any of the times of workers enumerated in Art 249. Hanjin petition for certification election in which the Knitjoy Monthly Employees Association and
claimed that one third (1/3) of the members of the association had definite employers and Confederation of Citizens Labor Union (KMEA-CCLU) and CFW intervened. The petition was
that only ambulant, intermittent, itinerant, rural workers, self-employed, and those without dismissed by the Med-Arbiter instructing them to work towards the formation of a single union
definite employers may form a workers' association.Hanjin also claimed that Samahan in the company. Upon appeal, the Director of BLR reversed the Med-Arbiter’s order.
misrepresented itself when Samahan made it appear that its members where all qualified to
become members of the worker’s association. Samahan did not respond to the pleading ISSUES: (1) WON allowing the creation of a separate existing bargaining unit will abet the
hence the DOLE Regional Director cancelled the certificate of registration. proliferation of unions hence disregarding CFW as the sole and exclusive bargaining agent.
(2) WON the scope of the new CBA binds KMEU and the acceptance by all the
The BLR granted Samahan’s appeal and reversed the ruling of the Regional Director. The members of KMEU of all the benefits of the CBA constitute an over act of ratification.
ruling made mention of the name of Samahan which uses the word “sa” rather than “ng”
meaning Hardin Shipyard was a location and do not pertain to the corporation. On MR, the HELD: (1) The suggested bias of the Labor Code in favor of the one company-one union
BLR affirmed its decision but instructed Samahan to remove “Hanjin Shipyard” from the policy is not without exceptions. The present Article 245 of the Labor Code expressly allows
name. On Appeal, CA reversed the decision of BLR ruling that since 57/120 were actually supervisory employees who are not performing managerial functions to join, assist or form
working in Hanjin, Samahan misrepresented itself in its preamble which created the their separate union but bars them from membership in a labor organization of the rank-and-
impression that all its members were employees of Hanjin. Butat any rate, removing “Hanjin file employees. This provision obviously allows more than one union in a company.
Shipyard” from the name would not prejudice or impair the right to self organization because
it could adopt other appropriate names. The right to form a union or association or to self-organization comprehends two (2) broad
notions, to wit: (a) the liberty or freedom, i.e., the absence of restraint which guarantees that
ISSUES: (1) WON Samahan can form a worker’s association of employees. the employee may act for himself without being prevented by law, and (b) the power, by
(2) WON Removing “Hanjin Shipyard” from the name by reason of the virtue of which an employee may, as he pleases, join or refrain from joining an association.
company’s property right over the company name “Hanjin” is proper.
Furthermore, it is not denied that in the bargaining history of KNITJOY, the CBA has been
HELD: (1) More often than not, the right to self-organization connotes unionism.  consistently limited to the regular rank-and-file employees paid on a daily or piece-rate basis.
Workers, however, can also form and join a workers' association as well as labor- On the other hand, the rank-and-file employees paid on a monthly basis were never included
management councils. Expressed in the highest law of the land is the right of all workers to within its scope. Thus, the negotiations between CFW and KNITJOY following such a
self-organization. This is imbued in Section 3, Article XIII and Section 8, Article III of the 1987 certification could only logically refer to the rank-and-file employees paid on a daily or piece-
Constitution and in relation thereto to Article 3 of the Labor Code. rate basis.

A union refers to any labor organization in the private sector organized for collective (2) No. Considering that (a) the TUPAS solicited certification election was strictly
bargaining and for other legitimate purpose, while a workers' association is an organization of confined to the rank-and-file employees who are paid on a daily or piece-rate basis, (b) the
workers formed for the mutual aid and protection of its members or for any legitimate purpose results of the election must also necessarily confine the certified union’s representation to the
other than collective bargaining. Many associations or groups of employees, or even group it represents and (c) the issue of the plight of the monthly-paid employees was still
combinations of only several persons, may qualify as a labor organization yet fall short of pending, KNITJOY and CFW clearly acted with palpable bad faith and malice in including
constituting a labor union. While every labor union is a labor organization, not every labor within the scope of the new CBA these monthly-paid employees. Thus was effected a
organization is a labor union. The difference is one of organization, composition and conspiracy to defeat and suppress the right of the KMEU and its members to bargain
operation. collectively and negotiate for themselves, to impose upon the latter a contract the negotiation
for which they were not even given notice of, consulted or allowed to participate in, and to
A cursory reading of the law demonstrates that a common element between unionism and oust from the BLR the pending appeal on the certification issue. In the latter case, KNITJOY
the formation of LMCs is the existence of an employer-employee relationship. In contrast, the and CFW are guilty of contumacious conduct. It goes without saying then that the new CBA
existence of employer-employee relationship is not mandatory in the formation of workers' cannot validly include in its scope or coverage the monthly-paid rank-and-file employees of
association.  KNITJOY. It does not bar the holding of a certification election to determine their sole
bargaining agent, and the negotiation for and the execution of a subsequent CBA between
Right to choose whether to form or join a union or workers' association belongs to workers KNITJOY and the eventual winner in said election. 
themselves. The right to form or join a labor organization necessarily includes the right to
refuse or refrain from exercising the said right. It is self-evident that just as no one should be GSIS vsKapisananngmgaMangagawasa GSIS, G.R. No. 170132, Dec 6, 2006
denied the exercise of a right granted by law, so also, no one should be compelled to
exercise such a conferred right. There is no provision in the Labor Code that states that FACTS: A 4-day mass action started on October 4, 2004 where participants were GSIS
employees with definite employers may form, join or assist unions only. personnel including Kapisanan mg mgaManggagawasa GSIS (KMG or the Union), a public
sector union of GSIS rank-and-file employees. While Pasay Mayor issued a rally permit, the
Furthermore, the position that Samahan's members cannot form the association because GSIS employees was not covered by a priod approved leave.
they are not covered by the second sentence of Article 243 (now 249) is incorrect. There is
nothing in the Article 243 as well as in Rule 2 of Department Order (D.O.) No. 40-03, Series The GSIS Investigating Unit manager chaged some 110 KMG members for grave misconduct
of 2003, which provides that workers, with definite employers, cannot form or join a workers' and conduct prejudicial to the best interest of the service. KMG filed suit to prevent the
association for mutual aid and protection. Thus, the Court agrees with Samahan's argument suspension of members of KMG claiming that they have a right to rally for their union's
that the right to form a workers' association is not exclusive to ambulant, intermittent and cause.
itinerant workers. Furthermore, the Court concludes that misrepresentation, to be a ground
for the cancellation of the certificate of registration, must be done maliciously and ISSUE: WON Winston Garcia (GSIS president) have cause to suspend members of
deliberately. KMG.

(2) Yes it is proper. As there is no provision under our labor laws which speak of HELD: Yes. It should be stressed right off that the civil service encompasses all
the use of name by a workers' association, the Court refers to the Corporation Code, which branches and agencies of the Government, including government-owned or controlled
governs the names of juridical persons. The policy underlying the prohibition in Section 18 corporations (GOCCs) with original charters, like the GSIS, or those created by special law.
against the registration of a corporate name which is "identical or deceptively or confusingly As such, employees of covered GOCCs are part of the civil service system and are subject to
similar" to that of any existing corporation or which is "patently deceptive" or "patently circulars, rules and regulations issued by the Civil Service Commission (CSC) on discipline,
confusing" or "contrary to existing laws," is the avoidance of fraud upon the public which attendance and general terms/conditions of employment, inclusive of matters involving self-
would have occasion to deal with the entity concerned, the evasion of legal obligations and organization, strikes, demonstrations and like concerted actions.
duties, and the reduction of difficulties of administration and supervision over corporations.
For the same reason, it would be misleading for the members of Samahan to use "Hanjin The question that immediately comes to the fore, therefore, is whether or not the mass action
Shipyard" in its name as it could give the wrong impression that all of its members are staged by or participated in by said GSIS employees partook of a strike or prohibited
employed by Hanjin. Hence the SC reinstated the decision of BLR as modified by its concerted mass action. With the view we take of the events that transpired on October 4-7,
November 28, 2011 Resolution 2004, what respondents members launched or participated in during that time partook of a
strike or, what contextually amounts to the same thing, a prohibited concerted activity. The
Knitjoy Manufacturing, Inc. vs Ferrer-Calleja, G.R. No. 81883, Sept. 23, 1992 phrase prohibited concerted activity refers to any collective activity undertaken by
government employees, by themselves or through their employees organization, with the
FACTS: Petitioner KNITJOY had a collective bargaining agreement (CBA) with the intent of effecting work stoppage or service disruption in order to realize their demands or
Federation of Filipino Workers (FFW). The bargaining unit covered only the regular rank-and- force concessions, economic or otherwise; it includes mass leaves, walkouts, pickets and
2

acts of similar nature. This was shown when 48% of employees took to the streets on the first (2) No. Art. 245 (which superseded Art. 246) of the Labor Code is the result of
day, leaving the other employees to fend for themselves in a government office. the amendment of the Labor Code in 1989 by R.A. No. 6715, otherwise known as the
Herrera-Veloso Law. Unlike the Industrial Peace Act or the provisions of the Labor Code
For, as articulated earlier, any collective activity undertaken by government employees with which it superseded, R.A. No. 6715 provides separate definitions of the terms "managerial"
the intent of effecting work stoppage or service disruption in order to realize their demands or and "supervisory employees". Although the definition of "supervisory employees" seems to
force concessions, economic or otherwise, is a prohibited concerted mass action and have been unduly restricted to the last phrase of the definition in the Industrial Peace Act, the
doubtless actionable administratively. legal significance given to the phrase "effectively recommends" remains the same. In fact, the
The appellate court faulted petitioner Garcia for not first taping existing grievance machinery distinction between top and middle managers, who set management policy, and front-line
and other modes of settlement agreed upon in the GSIS-KMG Collective Negotiations supervisors, who are merely responsible for ensuring that such policies are carried out by the
Agreement (CAN) before going full steam ahead with his formal charges. If the finger of rank and file, is articulated in the present definition. When read in relation to this definition in
blame, therefore, is to be pointed at someone for non-exhaustion of less confrontational Art. 212(m), it will be seen that Art. 245 faithfully carries out the intent of the Constitutional
remedies, it should be at the respondent union for spearheading a concerted mass action Commission in framing Art. III, §8 of the fundamental law. Nor is the guarantee of
without resorting to available settlement mechanism. As it were, it was KMG, under Atty. organizational right in Art. III, §8 infringed by a ban against managerial employees forming a
Alberto Velasco, which opened fire first. That none of the parties bothered to avail of the union. The right guaranteed in Art. III, §8 is subject to the condition that its exercise should be
grievance procedures under the GSIS-KMG CNA should not be taken against the GSIS. At for purposes "not contrary to law." In the case of Art. 245, there is a rational basis for
best, both GSIS management and the Union should be considered as in pari delicto. prohibiting managerial employees from forming or joining labor organizations.

United Pepsi-Cola Supervisory Union vsLaguesma, G.R. No. 122226, Mar 25, 1998 The rationale for this inhibition has been stated to be, because if these managerial
employees would belong to or be affiliated with a Union, the latter might not be assured of
FACTS: Petitioner is a union of supervisory employees. It appears that on March 20, their loyalty to the Union in view of evident conflict of interests. The Union can also become
1995 the union filed a petition for certification election on behalf of the route managers at company-dominated with the presence of managerial employees in Union membership. After
Pepsi-Cola Products Philippines, Inc. However, its petition was denied by the med-arbiter all, those who qualify as top or middle managers are executives who receive from their
and, on appeal, by the Secretary of Labor and Employment, on the ground that the route employers information that not only is confidential but also is not generally available to the
managers are managerial employees and, therefore, ineligible for union membership. public, or to their competitors, or to other employees. It is hardly necessary to point out that to
Petitioner filed a motion for reconsideration, pressing for resolution its contention that the first say that the first sentence of Art. 245 is unconstitutional would be to contradict the decision in
sentence of Art. 245 of the Labor Code, so far as it declares managerial employees to be that case.
ineligible to form, assist or join unions, contravenes Art. III, §8 of the Constitution which
provides: Republic vs. Kawashima TextimeMfg, G.R. No. 160352, July 23, 2008

The right of the people, including those employed in the public and private sectors, to form FACTS:Kawashima Free Workers Union-PTGWO Local Chapter No. 803 (KFWU) filed a
unions, associations, or societies for purposes not contrary to law shall not be abridged. petition for certification election. Respondent filed a MTD on the ground that KFWU did not
acquire any legal personality because its membership of mixed rank and file and supervisory
ISSUES: (1) WON the route managers at Pepsi-Cola Products Philippines, Inc. are employees. Med-Arbiter Bactin found KFWU's legal personality defective and dismissed the
managerial employees. petition for certification election. DOLE reversed the decision while CA reverted to the
(2) WON Art. 245, insofar as it prohibits managerial employees from forming, previous decision of the Med-Arbiter.
joining or assisting labor unions, violates Art. III, §8 of the Constitution.
ISSUE: (1) WON mixed membership is a ground for dismissal of a petition for
HELD: (1) Yes. The term "manager" generally refers to "anyone who is responsible for certification election.
subordinates and other organizational resources." (2) WON the legitimacy of a duly registered labor organization can be collaterally
attacked in a petition for certification election thru a MTD filed by employer.
FIRST-LINE MANAGERS — The lowest level in an organization at which individuals are
responsible for the work of others is called first-line or first-level management. First-line HELD: (1) R.A. No. 9482 should answer this. A new provision, Article 245-A is inserted
managers direct operating employees only; they do not supervise other managers. into the Labor Code to read as follows:
Examples of first-line managers are the "foreman" or production supervisor in a
manufacturing plant, the technical supervisor in a research department, and the clerical "Art. 245-A. Effect of Inclusion as Members of Employees Outside the Bargaining Unit. - The
supervisor in a large office. First-level managers are often called supervisors. inclusion as union members of employees outside the bargaining unit shall not be a ground
for the cancellation of the registration of the union. Said employees are automatically deemed
MIDDLE MANAGERS — The term middle management can refer to more than one level in removed from the list of membership of said union." Moreover, under Section 4, a pending
an organization. Middle managers direct the activities of other managers and petition for cancellation of registration will not hinder a legitimate labor organization from
sometimes also those of operating employees. Middle managers' principal initiating a certification election. Furthermore, under Section 12 of R.A. No. 9481, employers
responsibilities are to direct the activities that implement their organizations' policies have no personality to interfere with or thwart a petition for certification election filed by a
and to balance the demands of their superiors with the capacities of their subordinates. legitimate labor organization (Employer as Bystander rule).
A plant manager in an electronics firm is an example of a middle manager.
However, R.A. No. 9481 took effect only on June 14, 2007; hence, it applies only to labor
TOP MANAGERS — Composed of a comparatively small group of executives, top representation cases filed on or after said date. As the petition for certification election
management is responsible for the overall management of the organization. It subject matter of the present petition was filed by KFWU on January 24, 2000, R.A. No. 9481
establishes operating policies and guides the organization's interactions with its cannot apply to it. R.A. No. 875 is controlling in this case.
environment. Typical titles of top managers are "chief executive officer," "president,"
and "senior vice-president." Actual titles vary from one organization to another and are Nothing in R.A. No. 875, however, tells of how the questioned mingling can affect the
not always a reliable guide to membership in the highest management classification. legitimacy of the labor organization. It may be observed that nothing is said of the effect of
such ineligibility upon the union itself or on the status of the other qualified members thereof
"Managerial employees" may therefore be said to fall into two distinct categories: the should such prohibition be disregarded. Considering that the law is specific where it intends
"managers" per se, who compose the former group described above, and the "supervisors" to divest a legitimate labor union of any of the rights and privileges granted to it by law, the
who form the latter group. Whether they belong to the first or the second category, managers, absence of any provision on the effect of the disqualification of one of its organizers upon the
vis-a-vis employers, are, likewise, employees. legality of the union, may be construed to confine the effect of such ineligibility only upon the
membership of the supervisor. In other words, the invalidity of membership of one of the
organizers does not make the union illegal, where the requirements of the law for the
To qualify as managerial employee, there must be a clear showing of the exercise of
organization thereof are, nevertheless, satisfied and met.
managerial attributes under paragraph (m), Article 212 of the Labor Code as amended.
Designations or titles of positions are not controlling. (m) - "Managerial employee" is one who
Thus, when the issue of the effect of mingling was brought to the fore in Toyota, the Court,
is vested with powers or prerogatives to lay down and execute management policies and/or
citing Article 245 of the Labor Code, as amended by R.A. No. 6715, held:
to hire, transfer, suspend, lay off, recall, discharge, assign or discipline employees.
Supervisory employees are those who, in the interest of the employer, effectively recommend
Clearly, based on this provision, a labor organization composed of both rank-and-file and
such managerial actions if the exercise of such authority is not merely routinary or clerical in
supervisory employees is no labor organization at all. It cannot, for any guise or purpose, be
nature but requires the use of independent judgment. All employees not falling within any of
a legitimate labor organization. Not being one, an organization which carries a mixture of
the above definitions are considered rank-and-file employees for purposes of this Book.
rank-and-file and supervisory employees cannot possess any of the rights of a legitimate
labor organization, including the right to file a petition for certification election for the purpose
That distinction is evident in the work of the route managers which sets them apart from of collective bargaining. It becomes necessary, therefore, anterior to the granting of an order
supervisors in general. Unlike supervisors who basically merely direct operating employees in allowing a certification election, to inquire into the composition of any labor organization
line with set tasks assigned to them, route managers are responsible for the success of the whenever the status of the labor organization is challenged on the basis of Article 245 of the
company's main line of business through management of their respective sales teams. Such Labor Code.
management necessarily involves the planning, direction, operation and evaluation of their
individual teams and areas which the work of supervisors does not entail. But then, on June 21, 1997, the 1989 Amended Omnibus Rules was further amended by
Department Order No. 9, series of 1997 (1997 Amended Omnibus Rules). Specifically, the
3

requirement under Sec. 2(c) of the 1989 Amended Omnibus Rules - that the petition for best suited to serve the reciprocal rights and duties of the parties under the collective
certification election indicate that the bargaining unit of rank-and-file employees has not been bargaining provisions of the law."
mingled with supervisory employees - was removed. Instead, what the 1997 Amended
Omnibus Rules requires is a plain description of the bargaining unit. All said, while the latest In light of these considerations, the Solicitor General has opined that separate bargaining
issuance is R.A. No. 9481, the 1997 Amended Omnibus Rules, had already set the tone for units in the three different plants of the division will fragmentize the employees of the said
it. Toyota and Dunlop no longer hold sway in the present altered state of the law and the division, thus greatly diminishing their bargaining leverage. The fact that the three plants are
rules. Hence, Court reverses the ruling of the CA and reinstates that of the DOLE granting located in three different places, namely, in Cabuyao, Laguna, in Otis, Pandacan, Metro
the petition for certification election of KFWU. Manila, and in San Fernando, Pampanga is immaterial. Geographical location can be
completely disregarded if the communal or mutual interests of the employees are not
(2) Except when it is requested to bargain collectively, an employer is a mere sacrificed as demonstrated. Neither are there regional differences that are likely to impede
bystander to any petition for certification election; such proceeding is non-adversarial and the operations of a single bargaining representative.
merely investigative, for the purpose thereof is to determine which organization will represent
the employees in their collective bargaining with the employer. The choice of their Sugbuanan Rural Bank vs. Laguesma, G.R. No. 116194, February 2, 2000
representative is the exclusive concern of the employees; the employer cannot have any
partisan interest therein; it cannot interfere with, much less oppose, the process by filing a FACTS:Petitioner Sugbuanon Rural Bank, Inc., (SRBI, for brevity) is a duly-registered
motion to dismiss or an appeal from it; not even a mere allegation that some employees banking institution with principal office in Cebu City and a branch in Mandaue City. Private
participating in a petition for certification election are actually managerial employees will lend respondent SRBI Association of Professional, Supervisory, Office, and Technical Employees
an employer legal personality to block the certification election. The employer's only right in Union (APSOTEU) is a legitimate labor organization affiliated with the Trade Unions
the proceeding is to be notified or informed thereof. Congress of the Philippines (TUCP).1âwphi1.nêt

San Miguel Corp. Supervisors & Exempt Employees Union vs. Laguesma, G.R. No. 110399, On October 8, 1993, the DOLE Regional Office in Cebu City granted Certificate of
Aug 15, 1997 Registration No. R0700-9310-UR-0064 to APSOTEU-TUCP, hereafter referred to as the
union.
FACTS: Petitioner union filed before the Department of Labor and Employment (DOLE) a
Petition for Direct Certification or Certification Election among the supervisors and exempt On October 26, 1993, the union filed a petition for certification election of the supervisory
employees of the SMC Magnolia Poultry Products Plants of Cabuyao, San Fernando and employees of SRBI. It alleged, among others, that: (1) APSOTEU-TUCP was a labor
Otis. Med-Arbiter ordered the conduct of certification election which respondent San Miguel organization duly-registered with the Labor Department; (2) SRBI employed 5 or more
Corporation pointed out that Med-Arbiter should not have grouped together the three supervisory employees; (3) a majority of these employees supported the petition: (4) there
separate plants into one bargaining unit and in including supervisory levels 3 and above was no existing collective bargaining agreement (CBA) between any union and SRBI; and (5)
whose positions are confidential in nature. Undersecretary Laguesma granted the no certification election had been held in SRBI during the past 12 months prior to the petition.
reconsideration and directed the conduct of separate certification elections. Upon San
Miguel's MR, a further Order was issued declaraingthat S3 and S4 Supervisors and the so- On October 28, 1993, the Med-Arbiter gave due course to the petition. The pre-certification
called exempt employees are admittedly confidential employees and therefore, they are not election conference between SRBI and APSOTEU-TUCP was set for November 15, 1993.
allowed to form, join or assist a labor union for purposes of collective bargaining following the
above court's ruling. Consequently, they are not allowed to participate in the certification On November 12, 1993, SRBI filed a motion to dismiss the union’s petition. It sought to
election. prevent the holding of a certification election on two grounds. First, that the members of
APSOTEU-TUCP were in fact managerial or confidential employees.
ISSUES: (1) WON Supervisory employees 3 and 4 and the exempt employees of the
company are considered confidential employees, hence ineligible from joining a union. ISSUES: (1) Whether or not the members of the respondent union are managerial
(2) WON they should constitute an appropriate single bargaining unit. employees and/or highly-placed confidential employees, hence prohibited by law from joining
labor organizations and engaging in union activities.
HELD: (1) No. There is no question that the said employees, supervisors and the
exempt employees, are not vested with the powers and prerogatives to lay down and execute (2) Whether or not the Med-Arbiter may validly order the holding of a certification
management policies and/or to hire, transfer, suspend, layoff, recall, discharge or dismiss election upon the filing of a petition for certification election by a registered union, despite the
employees. They are, therefore, not qualified to be classified as managerial employees who, petitioner’s appeal pending before the DOLE Secretary against the issuance of the union’s
under Article 245 of the Labor Code, are not eligible to join, assist or form any labor registration.
organization. In the very same provision, they are not allowed membership in a labor
organization of the rank-and-file employees but may join, assist or form separate labor HELD: (1) Petitioner’s explanation does not state who among the employees has access to
organizations of their own. information specifically relating to its labor to relations policies. Even Cashier Patricia Maluya,
who serves as the secretary of the bank’s Board of Directors may not be so classified.
(2) Yes. Confidential employees are those who (1) assist or act in a confidential
capacity, (2) to persons who formulate, determine, and effectuate management policies in the Confidential employees are those who (1) assist or act in a confidential capacity, in regard (2)
field of labor relations. The two criteria are cumulative, and both must be met if an employee to persons who formulate, determine, and effectuate management policies [specifically in the
is to be considered a confidential employee — that is, the confidential relationship must exist field of labor relations].9 The two criteria are cumulative, and both must be met if an
between the employee and his supervisor, and the supervisor must handle the prescribed employee is to be considered a confidential employee — that is, the confidential relationship
responsibilities relating to labor relations. must exist between the employee and his superior officer; and that officer must handle the
prescribed responsibilities relating to labor relations.
The exclusion from bargaining units of employees who, in the normal course of their duties,
become aware of management policies relating to labor relations is a principal objective Art. 245 of the Labor Code does not directly prohibit confidential employees from engaging in
sought to be accomplished by the ''confidential employee rule." The broad rationale behind union activities. However, under the doctrine of necessary implication, the disqualification of
this rule is that employees should not be placed in a position involving a potential conflict of managerial employees equally applies to confidential employees. The confidential-employee
interests. An important element of the "confidential employee rule" is the employee's need to rule justifies exclusion of confidential employees because in the normal course of their duties
use labor relations information. Thus, in determining the confidentiality of certain employees, they become aware of management policies relating to labor relations. It must be stressed,
a key question frequently considered is the employee's necessary access to confidential however, that when the employee does not have access to confidential labor relations
labor relations information. Granting arguendo that an employee has access to confidential information, there is no legal prohibition against confidential employees from forming,
labor relations information but such is merely incidental to his duties and knowledge thereof is assisting, or joining a union.
not necessary in the performance of such duties, said access does not render the employee
a confidential employee. (2) One of the rights of a legitimate labor organization under Article 242(b) of the
Labor Code is the right to be certified as the exclusive representative of all employees in an
In the case at bar, supervisors 3 and above may not be considered confidential employees appropriate bargaining unit for purposes of collective bargaining. Having complied with the
merely because they handle "confidential data" as such must first be strictly classified as requirements of Art. 234, it is our view that respondent union is a legitimate labor union.
pertaining to labor relations for them to fall under said restrictions. The information they Article 257 of the Labor Code mandates that a certification election shall automatically be
handle are properly classifiable as technical and internal business operations data which, to conducted by the Med-Arbiter upon the filing of a petition by a legitimate labor
our mind, has no relevance to negotiations and settlement of grievances wherein the organization.16 Nothing is said therein that prohibits such automatic conduct of the
interests of a union and the management are invariably adversarial. Since the employees are certification election if the management appeals on the issue of the validity of the union’s
not classifiable under the confidential type, this Court rules that they may appropriately form a registration. On this score, petitioner’s appeal was correctly dismissed.
bargaining unit for purposes of collective bargaining. Furthermore, even assuming that they
are confidential employees, jurisprudence has established that there is no legal prohibition Coca-Cola vs. IlocosProf’l&Tech’l Employees Union, G.R. No. 193798, Sept 9, 2015
against confidential employees who are not performing managerial functions to form and join
a union. FACTS: Petitioner Coca-Cola Bottlers Philippines, Inc. (CCBPI) is a domestic corporation
duly organized and operating under the Philippine laws. It is primarily engaged in the
An appropriate bargaining unit may be defined as "a group of employees of a given beverage business, which includes the manufacture of carbonated soft drinks. On the other
employer, comprised of all or less than all of the entire body of employees, which the hand, respondent Ilocos Professional and Technical Employees Union (IPTEU) is a
collective interest of all the employees, consistent with equity to the employer, indicate to be
4

registered independent labor organization with address at CCBPI Ilocos Plant in Barangay FACTS: ICMC an accredited refugee processing center in Morong Bataan, is a non-profit
Catuguing, San Nicolas, Ilocos Norte. agency involved in international humanitarian and voluntary work. It is duly registered with the
United Nations Economic and Social Council (ECOSOC) and enjoys Consultative status II. It
On July 9, 2007, IPTEU filed a verified Petition for certification election seeking to represent a has the activities parallel to those of the International Committee for Migrtion (ICM) and the
bargaining unit consisting of approximately twenty-two (22) rank-and-file professional and International Committee of the Red Cross (ICRC).
technical employees of CCBPI Ilocos Norte Plant. CCBPI prayed for the denial and dismissal
of the petition, arguing that the some members are supervisory and confidential employees; On July 14, 1986, Trade Union of the Philippines and Allied Services (TUPAS) filed with the
hence, ineligible for inclusion as members of IPTEU. then Ministry of Labor and Employment a Petition for Certification Election among the rank
and file members employed by the ICMC. The latter opposed the petition on the ground that it
ISSUE: WON the members are confidential employees and should be exempt. enjoys diplomatic immunity.

HELD: No. Even if the 16 challenged voters may have access to information which are In the second case, the International Rice Research Institute was a fruit of memorandum of
confidential from the business standpoint, the exercise of their right to self-organization could understanding between the Philippine government and the Ford and Rochefeller
not be defeated because their common functions do not show that there exist a confidential Foundations. It was intended to be an autonomous, philanthropic tax-free, non-profit, non
relationship within the realm of labor relations. stock organization designed to carry out the principal objective of conducting “ basic research
on the rice plant.”
In this case, organizational charts, detailed job descriptions, and training programs were It was organized and registered with the SEC as a private corporation subject to all laws and
presented by CCBPI before the Mediator-Arbiter, the SOLE, and the CA. Despite these, the regulations. However, by virtue of P.D no. 1620, IRRI was granted the status, prerogatives,
Mediator-Arbiter ruled that employees who encounter or handle trade secrets and financial privileges and immunities of an international organization.
information are not automatically classified as confidential employees. It was admitted that
the subject employees encounter and handle financial as well as physical production data The KAPISANAN NG MANGGAGAWA AT TAC SA IRRI-ORGANIZED LABOR
and other information which are considered vital and important from the business operations' ASSOCIATION IN LINE INDUSTRIES AND AGRICULTURE (Kapisanan) filed a petition for
standpoint. Nevertheless, it was opined that such information is not the kind of information direct certification election with regional office of the Department of Labor and Employment.
that is relevant to collective bargaining negotiations and settlement of grievances as would IRRI opposed the petition invoking Pres. Decree no.1620 conferring upon it the status of an
classify them as confidential employees. The SOLE, which the CA affirmed, likewise held that international organization and granting it immunity from all civil, criminal, and administrative
the questioned voters do not have access to confidential labor relations information. proceedings under Philippine laws. The Med-Arbiter upheld the opposition on the basis of PD
1620 and dismissed the petition for direct certification.
Central Negros Electric Cooperative vs. DOLE, G.R. No. 94045, September 13, 1991 On appeal by BLR Director, set aside the med-arbiter’s decision and contends that
immunities and privileges granted to IRRI do not include exemption from coverage of our
FACTS: On August 15, 1987, CENECO entered into a collective bargaining agreement with labor laws.
CURE, a labor union representing its rank-and-file employees, providing for a term of three
years retroactive to April 1, 1987 and extending up to March 31, 1990. On December 28, ISSUES: (1) Whether or not the grant of diplomatic privileges and immunities to ICMC
1989, CURE wrote CENECO proposing that negotiations be conducted for a new collective extends to immunity from the application of Philippine labor laws.
bargaining agreement (CBA). (2) Whether or not the Secretary of Labor committed grave abuse of discretion in
dismissing the petition for certification election filed by Kapisanan.
On January 18, 1990, CENECO denied CURE’s request on the ground that, under applicable
decisions of the Supreme Court, employees who at the same time are members of an electric HELD: (1) Yes. It is a recognized principle of international law and under our system of
cooperative are not entitled to form or join a union. separation of powers that diplomatic immunity is essentially a political question and courts
should refuse to look beyond a determination by the executive branch of the government, and
Prior to the submission of the proposal for CBA renegotiation, CURE members, in a general where the plea of diplomatic immunity is recognized and affirmed by the executive branch of
assembly held on December 9, 1989, approved Resolution No. 35 whereby it was agreed the government as in the case at bar, it is then the duty of the courts to accept the claim of
that ‘tall union members shall withdraw, retract, or recall the union members’ membership immunity upon appropriate suggestion by the principal law officer of the government . . . or
from Central Negros Electric Cooperative, Inc. in order to avail (of) the full benefits under the other officer acting under his direction. Hence, in adherence to the settled principle that
existing Collective Bargaining Agreement entered into by and between CENECO and CURE, courts may not so exercise their jurisdiction . . . as to embarrass the executive arm of the
and the supposed benefits that our union may avail of under the renewed CBA. government in conducting foreign relations, it is accepted doctrine that in such cases the
judicial department of (this) government follows the action of the political branch and will not
However, the withdrawal from membership was denied by CENECO on February 27, 1990 embarrass the latter by assuming an antagonistic jurisdiction.
under Resolution No. 90.
Employees are not without recourse whenever there are disputes to be settled because each
Issue: WON the employees of CENECO who withdrew their membership from the specialized agency shall make provision for appropriate modes of settlement of disputes out
cooperative are entitled to form or join CURE for purposes of the negotiations for a collective of contracts or other disputes of private character to which the specialized agency is a party.
bargaining agreement proposed by the latter. Moreover, pursuant to article IV of memorandum of abuse of privilege by ICMC, the
government is free to withdraw the privileges and immunities accorded.
Held:The right of the employees to self-organization is a compelling reason why their
withdrawal from the cooperative must be allowed. As pointed out by CURE, the resignation of (2) No. There are basically three propositions underlying the grant of
the member- employees is an expression of their preference for union membership over that international immunities to international organizations. These principles, contained in the ILO
of membership in the cooperative. The avowed policy of the State to afford fall protection to Memorandum are stated thus: 1) international institutions should have a status which protects
labor and to promote the primacy of free collective bargaining mandates that the employees’ them against control or interference by any one government in the performance of functions
right to form and join unions for purposes of collective bargaining be accorded the highest for the effective discharge of which they are responsible to democratically constituted
consideration. international bodies in which all the nations concerned are represented; 2) no country should
derive any national financial advantage by levying fiscal charges on common international
Thus, member employees of a cooperative may withdraw as members of the cooperative in funds; and 3) the international organization should, as a collectivity of States members, be
order to join labor union. Membership in a cooperative is voluntary; inherent in it is the right accorded the facilities for the conduct of its official business customarily extended to each
not to join. other by its individual member States.
___________________________
NOTES: (San Jose Electric Service Cooperative vs. Ministry of Labor) The grant of immunity from local jurisdiction to ICMC and IRRI is clearly necessitated by their
international character and respective purposes. The objective is to avoid the danger of
1. A cooperative, therefore, is by its nature different from an ordinary business concern being partiality and interference by the host country in their internal workings. The exercise of
run either, by persons, partnerships or corporations. Its owners and/or members are the ones jurisdiction by the Department of Labor in these instances would defeat the very purpose of
who run and operate the business while the others are its employees. immunity, which is to shield the affairs of international organizations, in accordance with
international practice, from political pressure or control by the host country to the prejudice of
2. An employee therefore of such a cooperative who is a member and co-owner thereof member States of the organization, and to ensure the unhampered performance of their
cannot invoke the right to collective bargaining for certainly an owner cannot bargain with functions.
himself or his co-owners. Employees of cooperatives who are themselves members of the
cooperative have no right to form or join labor organizations for purposes of collective Neither are the employees of IRRI without remedy in case of dispute with management as, in
bargaining for being themselves co-owners of the cooperative. fact, there had been organized a forum for better management-employee relationship as
evidenced by the formation of the Council of IRRI Employees and Management (CIEM)
3. However, in so far as it involves cooperatives with employees who are not members or co- wherein "both management and employees were and still are represented for purposes of
owners thereof, certainly such employees are entitled to exercise the rights of all workers to maintaining mutual and beneficial cooperation between IRRI and its employees." The
organization, collective bargaining, negotiations and others as are enshrined in the existence of this Union factually and tellingly belies the argument that Pres. Decree No. 1620,
Constitution and existing laws of the country. which grants to IRRI the status, privileges and immunities of an international organization,
deprives its employees of the right to self-organization.
Int’l Catholic Immigration Commission vs. Calleja, G.R. No. 85750, September 28, 1990
NUWHRAIN vs. Sec. of Labor, G.R. No. 181531, July 31, 2009
5

FACTS: A certification election was conducted on June 16, 2006 among the rank-and-file It is well-settled that under the so-called “double majority rule,” for there to be a valid
employees of respondent Holiday Inn Manila Pavilion Hotel (the Hotel) with the following certification election, majority of the bargaining unit must have voted AND the winning union
results: must have garnered majority of the valid votes cast.

EMPLOYEES IN VOTERS’ LIST = 353 Prescinding from the Court’s ruling that all the probationary employees’ votes should be
TOTAL VOTES CAST = 346 deemed valid votes while that of the supervisory employees should be excluded, it follows
NUWHRAIN-MPHC = 151 that the number of valid votes cast would increase – from 321 to 337. Under Art. 256 of the
HIMPHLU =169 Labor Code, the union obtaining the majority of the valid votes cast by the eligible voters shall
NO UNION = 1 be certified as the sole and exclusive bargaining agent of all the workers in the appropriate
SPOILED = 3 bargaining unit. This majority is 50% + 1. Hence, 50% of 337 is 168.5 + 1 or at least 170.
SEGREGATED = 22
HIMPHLU obtained 169 while petitioner received 151 votes. Clearly, HIMPHLU was not able
In view of the significant number of segregated votes, contending unions, petitioner, to obtain a majority vote. The position of both the SOLE and the appellate court that the
NUHWHRAIN-MPHC, and respondent Holiday Inn Manila Pavillion Hotel Labor Union opening of the 17 segregated ballots will not materially affect the outcome of the certification
(HIMPHLU), referred the case back to Med-Arbiter to decide which among those votes would election as for, so they contend, even if such member were all in favor of petitioner, still,
be opened and tallied. 11 votes were initially segregated because they were cast by HIMPHLU would win, is thus untenable.
dismissed employees, albeit the legality of their dismissal was still pending before the Court
of Appeals. Six other votes were segregated because the employees who cast them were It bears reiteration that the true importance of ascertaining the number of valid votes cast is
already occupying supervisory positions at the time of the election. Still five other votes were for it to serve as basis for computing the required majority, and not just to determine which
segregated on the ground that they were cast by probationary employees and, pursuant to union won the elections. The opening of the segregated but valid votes has thus become
the existing Collective Bargaining Agreement (CBA), such employees cannot vote. It bears material.
noting early on, however, that the vote of one Jose Gatbonton (Gatbonton), a probationary
employee, was counted. To be sure, the conduct of a certification election has a two-fold objective: to determine the
appropriate bargaining unit and to ascertain the majority representation of the bargaining
Med-Arbiter Calabocal ruled for the opening of 17 out of the 22 segregated votes, specially representative, if the employees desire to be represented at all by anyone. It is not simply the
those cast by the 11 dismissed employees and those cast by the six supposedly supervisory determination of who between two or more contending unions won, but whether it effectively
employees of the Hotel. ascertains the will of the members of the bargaining unit as to whether they want to be
represented and which union they want to represent them.
Petitioner, which garnered 151 votes, appealed to the Secretary of Labor and Employment
(SOLE), arguing that the votes of the probationary employees should have been opened Having declared that no choice in the certification election conducted obtained the required
considering that probationary employee Gatbonton’s vote was tallied. And petitioner averred majority, it follows that a run-off election must be held to determine which between HIMPHLU
that respondent HIMPHLU, which garnered 169 votes, should not be immediately certified as and petitioner should represent the rank-and-file employees.
the bargaining agent, as the opening of the 17 segregated ballots would push the number of ____________
valid votes cast to 338 (151 + 169 + 1 + 17), hence, the 169 votes which HIMPHLU garnered NOTES:
would be one vote short of the majority which would then become 169.
A run-off election refers to an election between the labor unions receiving the 2 highest
Secretary affirmed the decision of the med-arbiter. In fine, the SOLE concluded that the number of votes in a certification or consent election with 3 or more choices, where such a
certification of HIMPHLU as the exclusive bargaining agent was proper. certified or consent election results in none of the 3 or more choices receiving the majority of
the valid votes cast; provided that the total number of votes for all contending unions is at
ISSUES: (1) whether employees on probationary status at the time of the certification least 50% of the number of votes cast.
elections should be allowed to vote
(2) whether HIMPHLU was able to obtain the required majority for it to be Heirs of Teodulo M. Cruz vs. CIR, G.R. No. L-23331-32, December 27, 1969
certified as the exclusive bargaining agent.
FACTS: June 21, 1952, the Santiago Labor Union, composed of workers of the Santiago
HELD: (I)On the first issue, the Court rules in the affirmative. Rice Mill, a business enterprises engaged in the, buying. and milling of palay at Santiago,
Isabela, and owned and operated by King Hong Co., Inc., filed before the respondent Court
The inclusion of Gatbonton’s vote was proper not because it was not questioned but because of Industrial Relations (CIR) cases for petition for overtime pay, premium pay for night,
probationary employees have the right to vote in a certification election. The votes of the six Sunday and holiday work, and for reinstatement of workers illegally laid off. CIR favored the
other probationary employees should thus also have been counted. As Airtime Specialists, union by a split decision of 3-2 vote. The case reached the Supreme Court but the SC still
Inc. v. Ferrer-Calleja holds: favored the union. The SC remanded the records for enforcement by respondent CIR. In
examination of books, said CIR’s Chief Examiner filed his Partial Report regarding the
In a certification election, all rank and file employees in the appropriate bargaining unit, computation of the benefits rendered in the case in favor of the Union.
whether probationary or permanent are entitled to vote. This principle is clearly stated in Art.
255 of the Labor Code which states that the “labor organization designated or selected by the Petitioners claim that in this computation of the Examiner did not, include the claims of 70
majority of the employees in an appropriate bargaining unit shall be the exclusive other laborers whose total claims (for back wages), at the rate of P6,300.00 each and would
representative of the employees in such unit for purposes of collective bargaining.” Collective be P441,000.00. Therefore, the correct. grand total amount due the laborers would be
bargaining covers all aspects of the employment relation and the resultant CBA negotiated by 864,756.74.
the certified union binds all employees in the bargaining unit. Hence, all rank and file
employees, probationary or permanent, have a substantial interest in the selection of the The trial judge took no action on the latest Urgent Motion of the union, wherein it emphasized
bargaining representative. The Code makes no distinction as to their employment status as that respondent, with Court’s action rejecting its appeal, no longer had any excuse for
basis for eligibility in supporting the petition for certification election. The law refers to “all” the refusing to comply with the deposit Order. Instead, an unscheduled conference was called
employees in the bargaining unit. All they need to be eligible to support the petition is to and held on October 31, 1963 in the chambers of the trial judge, and attended by
belong to the “bargaining unit.” (Emphasis supplied) representatives of respondent firm, including their counsels of record and the President of the
union and 8 directors of the union. Four of these nine union representatives, including the
For purposes of this section (Rule II, Sec. 2 of Department Order No. 40-03, series of 2003), union president himself, had no claims or awards whatever under the judgment. Said union
any employee, whether employed for a definite period or not, shall beginning on the first day officials were not assisted by counsel, as petitioner Mary Concepcion, counsel of record of
of his/her service, be eligible for membership in any labor organization. the union, was not present, not having been notified of the conference.

All other workers, including ambulant, intermittent and other workers, the self-employed, rural In this conference respondent firm made again the same offer to settle and quitclaim the
workers and those without any definite employers may form labor organizations for their judgment in favor of the union members for the same amount of P110,000.00, which offer
mutual aid and protection and other legitimate purposes except collective bargaining. had already been ‘rejected by the union at the earlier conference held on June 25, 1963. But
(Emphasis supplied) this time, respondent and the directors of the union decided to settle the case amicably with
the payment by the firm of the same amount of P110,000.00 which was deposited with the
The provision in the CBA disqualifying probationary employees from voting cannot override Court’s disbursing officer “immediately upon the signing of the settlement which will be
the Constitutionally-protected right of workers to self-organization, as well as the provisions of prepared by the respondent firm through its counsel.”
the Labor Code and its Implementing Rules on certification elections and jurisprudence
thereon. One of the union director together with 49 of its members questioned the amicable settlement
that took place. They claim that the Board of Directors did not have any express authority of
A law is read into, and forms part of, a contract. Provisions in a contract are valid only if they the members of the Santiago Labor Union to enter into any compromise for the sum of
are not contrary to law, morals, good customs, public order or public policy. P110,000.00, that it was tainted by apparent bad faith on the part of the President of the
Union, that the amount of P110,000.00 is unconscionable, considering, that the total claims of
(2) As to whether HIMPHLU should be certified as the exclusive bargaining the members of the union is more than P400,000.00.
agent, the Court rules in the negative.
6

ISSUE: Whether the amicable settlement is valid. The settlement in the case at bar was precipitately approved without verification of the union
boards authority to execute the compromise settlement and that there was no such authority.
HELD: NO. Petitioners were not accorded due process of law. The union was deprived of the
assistance of its counsel. The lack or due deliberation and caution in the trial judge’s instant Victoriano vs. Elizalde Rope Workers’ Union, G.R. No. L-25246, September 12, 1974
approval of the settlement is seen from the stipulations therein that the union thereby waived
and quitclaimed any and all claims which it may have against the respondent, as well as the FACTS: Benjamin Victoriano (Appellee), a member of the religious sect known as the
claim of each and every one of the members of the union against respondent, when precisely “Iglesiani Cristo”, had been in the employ of the Elizalde Rope Factory, Inc. (Company) since
the authority of the union board members to enter into any such compromise or settlement 1958. He was a member of the Elizalde Rope Workers’ Union (Union) which had with the
was under express challenge by petitioner Magalpo, a board member herself which the trial Company a CBA containing a closed shop provision which reads as follows: “Membership in
judge completely disregarded. the Union shall be required as a condition of employment for all permanent employees
workers covered by this Agreement.”
Petitioners were deprived of the formal conference on the and of their right to be assisted by
the union counsel as expressly requested, so that a fair hearing could be accorded Under Sec 4(a), par 4, of RA 975, prior to its amendment by RA 3350, the employer was not
petitioners and an opportunity afforded them to air their serious charges of bad faith and lack precluded “from making an agreement with a labor organization to require as a condition of
authority against the Union leadership. Certainly, all these serious questions and charges employment membership therein, if such labor organization is the representative of the
made by petitioners could have been threshed out and verified, if the formal conference had employees.” On June 18, 1961, however, RA 3350 was enacted, introducing an amendment
been held with the presence of union counsel. to par 4 subsection (a) of sec 4 of RA 875, as follows: “xxx but such agreement shall not
cover members of any religious sects which prohibit affiliation of their members in any such
The transcript of the conference is deficient and does not reflect the actual discussions and labor organization”.
proceedings. This is to be deplored, for in a matter of such great importance, especially
where the union officials were unassisted by counsel in an unscheduled conference, care Being a member of a religious sect that prohibits the affiliation of its members with any labor
should be taken by the trial judge that the proceedings are faithfully recorded. organization, Appellee presented his resignation to appellant Union. The Union wrote a
formal letter to the Company asking the latter to separate Appellee from the service because
We find the forcing through of the settlement arbitrary, unfair and unconscionable. he was resigning from the Union as a member. The Company in turn notified Appellee and
his counsel that unless the Appellee could achieve a satisfactory arrangement with the Union,
Another reason for striking down the settlement is the lack of any express or specific authority the Company would be constrained to dismiss him from the service.
of the president and majority. of the union board of directors to execute the same and scale
down the estimated P423,756.74 judgment liability of respondent firm in favor of the Appellee filed an action for injunction to enjoin the Company and the Union from dismissing
individual union members to P110,000.00. On the contrary, petitioner board member Appellee. The Union invoked the “union security clause” of the CBA and assailed the
Magalpo timely challenged the authority or the union board to execute any such settlement, constitutionality of RA 3350 and contends it discriminatorily favors those religious sects which
expressly informing the trial judge that the union had specifically appointed an entity in ban their members from joining labor unions.
Manila, the “CREAM, Inc.,” as its attorney in-fact and “exclusive authorized representative for
the evaluation, adjustment and -liquidation or its claim against respondent. These union ISSUE:Whether Appellee has the freedom of choice in joining the union or not.
members have repudiated the former union president, Maylem and his board of directors, for
having betrayed the union members, and the new union leadership. HELD:YES. The Constitution and RA 875 recognize freedom of association. Sec 1 (6) of Art
The authority of the union, to execute a settlement of the judgment award in favor of the III of the Constitution of 1935, as well as Sec 7 of Art IV of the Constitution of 1973, provide
individual union members, cannot be presumed but must be expressly granted. that the right to form associations or societies for purposes not contrary to law shall not be
abridged. Section 3 of RA 875 provides that employees shall have the right to self-
Just as this Court has stricken down unjust exploitation of laborers by oppressive employers, organization and to form, join of assist labor organizations of their own choosing for the
so will it strike down their unfair treatment by their own unworthy leaders. The Constitution purpose of collective bargaining and to engage in concerted activities for the purpose of
enjoins the State to afford protection to labor. Fair dealing is equally demanded of unions as collective bargaining and other mutual aid or protection. What the Constitution and the
well as of employers in their dealings with employees. The union has been evolved as an Industrial Peace Act recognize and guarantee is the “right” to form or join associations. A
organization of collective strength for the protection of labor against the unjust exactions of right comprehends at least two broad notions, namely: first, liberty or freedom, i.e., the
capital, but equally important is the requirement of fair dealing between the union and its absence of legal restraint, whereby an employee may act for himself without being prevented
members, which is fiduciary in nature, and arises out of two factors: “one is the degree of by law; and second, power, whereby an employee may, as he pleases, join or refrain from
dependence of the individual employee on the union organization; the other, a corollary of the joining an association. It is, therefore, the employee who should decide for himself whether
first, is the comprehensive power vested in the union with respect to the individual.” The he should join or not an association; and should he choose to join, he himself makes up his
union may, be considered but the agent of its members for the purpose of securing for them mind as to which association he would join; and even after he has joined, he still retains the
fair and just wages and good working conditions and is subject to the obligation of giving the liberty and the power to leave and cancel his membership with said organization at any time.
members as its principals all information relevant to union and labor matters entrusted to it. The right to join a union includes the right to abstain from joining any union. The law does not
enjoin an employee to sign up with any association.
The union leadership in the case at bar was recreant in its duty towards the union members
in apparently having failed to disclose to the union members the full situation of their The right to refrain from joining labor organizations recognized by Section 3 of the Industrial
judgment credit against respondent, to wit, that they were in the advantageous position of Peace Act is, however, limited. The legal protection granted to such right to refrain from
being able to require enforcement of the respondent court’s P200,000.00-deposit order, and joining is withdrawn by operation of law, where a labor union and an employer have agreed
in presuming that it had authority to waive and quitclaim the estimated P423,756.74-judgment on a closed shop, by virtue of which the employer may employ only members of the collective
credit of the union members for the unconscionable amount of P110,000.00, which had bargaining union, and the employees must continue to be members of the union for the
already been previously rejected by the workers. Respondent firm could not claim that it dealt duration of the contract in order to keep their jobs. By virtue of a closed shop agreement,
in good faith with the union officials, for it hastily executed the purported settlement before the enactment of RA 3350, if any person, regardless of his religious beliefs, wishes to
notwithstanding the serious charges of bad faith against the union leadership, and the non- be employed or to keep his employment he must become a member of the collective
holding of the scheduled conference where the union leaders, at their express request, could bargaining union. Hence, the right of said employee not to join the labor union is curtailed and
be duly assisted by union counsel. withdrawn.

The interests of the individual worker can be better protected on the whole by a strong union To that all-embracing coverage of the closed shop arrangement, RA No.3350 introduced an
aware of its moral and legal obligations to represent the rank and file faithfully and secure for exception, when it added to Section 4 (a) (4) of the Industrial Peace Act the following proviso:
them the best wages and working terms and conditions in the process of collective “but such agreement shall not cover members of any religious sects which prohibit affiliation
bargaining. As has been aptly pointed out, the will of the majority must prevail over that of the of their members in any such labor organization”. Republic Act No. 3350 merely excludes
minority in the process, for “under the philosophy of collective responsibility, an employer who ipso jure from the application and coverage of the closed shop agreement the employees
bargains in good faith should be entitled to rely upon the promises and agreements of the belonging to any religious sects which prohibit affiliation of their members with any labor
union representatives with whom he must deal. under the compulsion of, law and contract. organization. What the exception provides is that members of said religious sects cannot be
The ‘collective bargaining process should be carried on between parties who can mutually compelled or coerced to join labor unions even when said unions have closed shop
respect and rely upon the authority of each other.” Where, however, collective bargaining agreements with the employers; that in spite of any closed shop agreement, members of said
process is not involved, and what is at stake are back wages already earned by the individual religious sects cannot be refused employment or dismissed from their jobs on the sole
workers by way of overtime, premium and differential pay, and final judgment has been ground that they are not members of the collective bargaining union. It does not prohibit the
rendered in their favor, as in the present case, the real parties in interest with direct material members of said religious sects from affiliating with labor unions. It still leaves to said
interest, as against the union which has only served as a vehicle for collective action to members the liberty and the power to affiliate, or not to affiliate, with labor unions. If,
enforce their just claims, are the individual workers themselves. Authority of the union to notwithstanding their religious beliefs, the members of said religious wets prefer to sign up
waive or quitclaim all or part of the judgment award in favor of the individual workers cannot with the labor union, they can do so. If in deference and fealty to their religious faith, they
be lightly presumed but must be expressly granted, and the employer, as judgment debtor, refuse to sign up, they can do so; the law does not coerce them to join; neither does the law
must deal in all good faith with the union as the agent of the individual workers. The Court in prohibit them from joining, and neither may the employer or labor union compel them to join.
turn should certainly verify and assure itself of the fact and extent of the authority of the union
leadership to execute any compromise or settlement of the judgment on behalf of the The Company was partly absolved by law from the contractual obligation it had with the
individual workers who are the real judgment creditors. Union of employing only Union members in permanent positions. It cannot be denied,
therefore, that there was indeed an impairment of said union security clause.
7

The prohibition to impair the obligation of contracts is not absolute and unqualified. The HELD: No. It has been held that "A full and complete pardon, granted after conviction,
prohibition is general. The prohibition is not to be read with literal exactness, for it prohibits removes all penalties and legal disabilities, and restores the defendant to all his civil rights."
unreasonable impairment only. In spite of the constitutional prohibition, the State continues to Continuing, the court went on to say that "pardon completely destroys the effect of the
possess authority to safeguard the vital interests of its people. Legislation appropriate to judgment ... (and) 'obliterates, in legal contemplation, the offense itself; and hence its effect is
safeguarding said interests may modify or abrogate contracts already in effect. For not only to make the offender a new man'" (Stephens v. State of ex rel. Goldsberry, 11 Okl. 262, 239
are existing laws read into contracts in order to fix the obligations as between the parties, but P. 450). In a similar vein, this Court, thru Mr. Justice Laurel, stated that "an absolute pardon
the reservation of essential attributes of sovereign power is also read into contracts as a not only blots out the crime committed but removes all disabilities resulting from the
postulate of the legal order. The contract clause of the Constitution. must be not only in conviction; and that when granted after the term of imprisonment has expired, absolute
harmony with, but also in subordination to, in appropriate instances, the reserved power of pardon removes all that is left of the consequences of the conviction;" (Pelobello v. Palatino,
the state to safeguard the vital interests of the people. This has special application to 72 Phil. 441).
contracts regulating relations between capital and labor which are not merely contractual, and
said labor contracts, for being impressed with public interest, must yield to the common good. Tancinco vs. Pura Ferrer-Calleja, G.R. No. 78131, January 20, 1988

The purpose to be achieved by RA 3350 is to insure freedom of belief and religion, and to FACTS: The union's Vice-President, Carlos Dalmacio was promoted to the position of
promote the general welfare by preventing discrimination against those members of religious Department Head, thereby disqualifying him for union membership. Said incident, among
sects which prohibit their members from joining labor unions, confirming thereby their natural, others led to a strike spearheaded by Lacanilao group, respondents herein. Another group
statutory and constitutional right to work, the fruits of which work are usually the only means however, led by herein petitioners staged a strike inside the company premises. After four (4)
whereby they can maintain their own life and the life of their dependents. days the strike was settled. An agreement was entered saying that all monthly-paid
employees shall be United under one union, the ITM Monthly Employees Association (ITM-
The individual employee, at various times in his working life, is confronted by two aggregates MEA), to be affiliated with ANGLO and that the management of ITM recognizes ANGLO as
of power collective labor, directed by a union, and collective capital, directed by management. the sole and exclusive bargaining agent of all the monthly-paid employees. However both
The union, an institution developed to organize labor into a collective force and thus protect parties failed to agree on the list of voters. In a pre-election conference, attendees made a
the individual employee from the power of collective capital, is, paradoxically, both the unilateral ruling excluding some 56 employees consisting of the Manila office employees,
champion of employee rights, and a new source of their frustration. Moreover, when the members of IglesianiKristo, non-time card employees, drivers of Mrs. Salazar and the
Union interacts with management, it produces yet a third aggregate of group strength from cooperative employees of Mrs. Salazar.
which the individual also needs protection – the collective bargaining relationship.
During the elction, Lacanilao won with 3 votes margin over Tancino prompting petitioners to
The free exercise of religious profession or belief is superior to contract rights. In case of make a protest and count the votes of the 56 employees. The Med Arbiter directed the
conflict, the latter must yield to the former. opening and counting of the 56 segregated votes. Private respondents categorized the
challenged voters into four groups namely, the Manila Employees, that they are personal
The purpose of RA 3350 is to serve the secular purpose of advancing the constitutional right employees of Mr. Lee; the IglesianiKristo, that allowing them to vote will be anomalous since
to the free exercise of religion, by averting that certain persons be refused work, or be it is their policy not to participate in any form of union activities; the non-time card employees,
dismissed from work, or be dispossessed of their right to work and of being impeded to that they are managerial employees; and the employees of the cooperative as non-ITM
pursue a modest means of livelihood, by reason of union security agreements. To help its employees.
citizens to find gainful employment whereby they can make a living to support themselves
and their families is a valid objective of the state. The Constitution even mandated that “the ISSUE: WON the exclusion of said voters are arbitrary, whimsical and without legal basis.
State shall afford protection to labor, promote full employment and equality in employment,
ensure equal work opportunities regardless of sex, race or creed and regulate the relation HELD: Yes. The public respondent ruled to set aside the Resolution of July 25, 1986 of the
between workers and employers.” Med-Arbiter based on its own findings that 51 of the 56 disenfranchised voters were not yet
union members at the time of the election of union officers on May 26, 1986 on the ground
The primary effects of the exemption from closed shop agreements in favor of members of that their names do not appear in the records of the Union submitted to the Labor
religious sects that prohibit their members from affiliating with a labor organization, is the Organization Division of the Bureau of Labor on April 24, 1986. The finding does not have a
protection of said employees against the aggregate force of the collective bargaining leg to stand on. Submission of the employees names with the BLR as qualified members of
agreement, and relieving certain citizens of a burden on their religious beliefs; and by the union is not a condition sine qua non to enable said members to vote in the election of
eliminating to a certain extent economic insecurity due to unemployment, which is a serious union's officers. It finds no support in fact and in law.
menace to the health, morals, and welfare of the people of the State, the Act also promotes
the well-being of society. It is our view that the exemption from the effects of closed shop In the case before Us, considering that none of the parties insisted on the use of the payroll
agreement does not directly advance, or diminish, the interests of any particular religion. period-list as voting list and considering further that the 51 remaining employees were
Although the exemption may benefit those who are members of religious sects that prohibit correctly ruled to be qualified for membership, their act of joining the election by casting their
their members from joining labor unions, the benefit upon the religious sects is merely votes on May 26, 1986 after the May 10, 1986 agreement is a clear manifestation of their
incidental and indirect. intention to join the union. They must therefore be considered ipso facto members thereof
Said employees having exercised their right to unionism by joining ITM-MEA their decision is
The purpose of RA 3350 was not to grant rights to labor unions. The rights of labor unions paramount. Their names could not have been included in the list of employee submitted on
are amply provided for in Republic Act No. 875 and the new Labor Code. April 24, 1986 to the Bureau of Labor for the agreement to join the union was entered into
only on May 10, 1986. Indeed the election was supervised by the Department of Labor where
The Act does not require as a qualification, or condition, for joining any lawful association said 56 members were allowed to vote. Private respondents never challenged their right to
membership in any particular religion or in any religious sect; neither does the Act require vote then.
affiliation with a religious sect that prohibits its members from joining a labor union as a
condition or qualification for withdrawing from a labor union. Joining or withdrawing from a Manalad vs. Trajano, G.R. Nos. 72772-73, June 28, 1989
labor union requires a positive act Republic Act No. 3350 only exempts members with such
religious affiliation from the coverage of closed shop agreements. So, under this Act, a FACTS: The parties are employees of United Dockhandlers, Inc, rival groups in the
religious objector is not required to do a positive act-to exercise the right to join or to resign Associated Port Checkers and Workers' Union (APCWU) Petitioner led by Ricardo R.
from the union. He is exempted ipso jure without need of any positive act on his part. Manalad, with respondent Pablo B. Babula heading the group of private
respondents.Although qualifications have been earlier questioned, Manalad et al won the
Flora vs. Oximana, G.R. No. L-19745, January 31, 1964, 10 SCRA 212 elections for APCWU officerson November 26, 1984. Babula et al filed petition for review and
on July 3, 1985, the court promulgated aresolution to dismiss petition for lack of merit and
FACTS: In 1926, Oximana was convicted of the crime of abusosdeshonestos (moral have petitioner Babula et al vacant APCWU offices and turnover management to Director of
turpitude) for which he was sentenced to 3 years 6 months and 25 days imprisonment which the Bureau of Labor Relations, all for immediate execution, to be followed by a special
he served until December 4, 1930. As a consequence, a complaint was lodged against him elections to be held on July 20, 1985 (to be held under the personal supervision of Director
before the Court of Industrial Relations on February 2, 1961 by a prosecutor of said court Trajano and his staff). Babula et al were alleged to refuse compliance with the above
seeking as president of the union on the strength of the previous of Section 17(e) of Republic resolution as documented in the petition filed by Manalad et al. The July 20 1985 special
Act 875. In this complaint, the union was made party respondent because of complainant's election was held having Babula et al as winners and duly elected officials of
desire to restrain Oximana from performing the duties and functions of his office as president APCWU.Manalad et al filed petition to disqualify Babula et al as winners due to their non-
and to have a new election held for the purpose of electing a new qualified president. compliance to the July 3 1985 resolution, but Director Trajano dismissed their petition and
proclaimed Babula et al as the winners of the July 20, 1985 special elections. Manalad et al,
When the case was called for hearing, the parties submitted a stipulation of facts wherein, then, filed petition to SC to reverse resolution of Trajano, have Babula et al disqualified and
among other things, it was agreed that on April 1, 1961 the President of the Philippines annul the July 20 1985 elections/conduct re-elections.In 1988, when 3-year term for the
granted Oximana full, absolute and plenary pardon for the crime he had committed in 1926, disputed 1985 election expired, a new set of officers for ACPWU has been elected despite
thereby restoring him to the full enjoyment of his civil and political rights, one of which is the motion for RTO. Manalad et al prayed for the annulment of 1988 elections.
holding of the position now disputed by complainants.
ISSUE:Whether or not motion for annulment of 1988 elections is moot and academic
ISSUE: WON Oximana can be disqualified as president due to the offense he was convicted
of.
8

HELD: Yes. The court found the motion for annulment of the 1988 ACPWU elections moot Private respondents instituted a complaint against the petitioners and the union counsel
and academic for the ff reasons: A) It is pointless and unrealistic to insist on annulling an before the Department of Labor and Employment (DOLE) for illegal deduction of attorney's
election of officers whose terms had already expired. We must consequently abide by our fees as well as for quantification of the benefits in the 1992 CBA.
consistent ruling that where certain events or circumstances have taken place during the
pendency of the case which would render the case moot and academic, the petition should ISSUE: WON the attorney's fees should be refunded to those union members who have not
be dismissed. B) The court respects the will of the majority of the workers who voted in the signified their conformity to the check-off of attorney's fees.
November 28, 1988 elections. C) Contentions of petitioners do not adequately establish the
basis for contempt but respondents have satisfactorily answered the averments thereon. D) HELD: Yes. Petitioners argue that the General Membership Resolution authorizing the bank
Obtaining the second highest number of votes does not mean that they will thereby be to check-off attorney's fee from the first lump sum payment of the legal benefits to the
considered as the elected officers if the true winners are disqualified. employees under the new CBA satisfies the legal requirements for such assessment. Private
respondents, on the other hand, claim that the check-off provision in question is illegal
Del Pilar Academy vs. Del Pilar Academy Employees’ Union, G.R. No. 170112, Ap 30, 2008 because it was never submitted for approval at a general membership meeting called for the
purpose and that it failed to meet the formalities mandated by the Labor Code.
FACTS: In September 1994, the Del Pilar Academy and the Del Pilar Academy Employees
Union entered into a collective bargaining agreement where it was agreed that: In check-off, the employer, on agreement with the Union, or on prior authorization from
employees, deducts union dues or agency fees from the latter's wages and remits them
a. the employees, teaching and non-teaching staff, shall have a salary increase; directly to the union. It assures continuous funding; for the labor organization. As this Court
b. the teaching staff shall have a maximum load of 23 hours per week in teaching; has acknowledged, the system of check-off is primarily for the benefit of the union and only
c. any overload shall be paid extra; indirectly for the individual employees.
d. there shall be an increase in the longevity pay;
e. teaching staff who have rendered service for 6 consecutive semester are entitled to Art. 241 has three (3) requisites for the validity of the special assessment for union's
receive pay during summer breaks; incidental expenses, attorney's fees and representation expenses. These are: 1)
f. non-union members who have rendered at least 1 year of service shall be entitled to 15 authorization by a written resolution of the majority of all the members at the general
days leave with pay. membership meeting called for the purpose; (2) secretary's record of the minutes of the
meeting; and (3) individual written authorization for check off duly signed by the employees
Since the new CBA benefits non-union members, the union asked Del Pilar to deduct agency concerned.
fees from the salaries of non-union members. Del Pilar refused to do so hence a labor case
was filed by the union. Clearly, attorney's fees may not be deducted or checked off from any amount due to an
employee without his written consent.
In its defense, Del Pilar avers that it cannot collect agency fees because the non-union
members refused to provide Del Pilar a check off authorization to make deductions from their After a thorough review of the records, we find that the General Membership Resolution of
salaries; and that further, the non-union members are not benefited because regardless of October 19, 1991 of the SolidBank Union did not satisfy the requirements laid down by law
the CBA, employees are going to be given a salary increase pursuant to a program by the and jurisprudence for the validity of the ten percent (10%) special assessment for union's
DECS which mandates all private schools to provide for salary increase based on tuition incidental expenses, attorney's fees and representation expenses. There were no individual
profits. written check off authorizations by the employees concerned and so the assessment cannot
be legally deducted by their employer.
ISSUE: Whether or not the Union is entitled to collect agency fees from non-union members,
and if so whether an individual written authorization is necessary for a valid check off. From all the foregoing, we are of the considered view that public respondent did not act with
grave abuse of discretion in ruling that the workers through their union should be made to
HELD: Yes and No. The collection of AGENCY FEES in an amount equivalent to union dues shoulder the expenses incurred for the services of a lawyer. And accordingly the
and fees, from employees who are not union members, is recognized by Article 248(e) of the reimbursement should be charged to the union's general fund or account. No deduction can
Labor Code. When so stipulated in a collective bargaining agreement or authorized in writing be made from the salaries of the concerned employees other than those mandated by law.
by the employees concerned, the Labor code and its Implementing Rules recognize it to be
the duty of the employer to deduct the sum equivalent to the amount of union dues, as Verceles vs. BLR-DOLE, G.R. No. 152322, February 15, 2005
agency fees, from the employee's wages for direct remittance to the union. The system is
referred to as CHECK OFF. No requirement of written authorization from the non-union FACTS: The case arose from a memorandum filed by Petitoners against Private Respondent
employees is necessary if the non-union employees accept the benefits resulting from the for allegedly spreading false rumors and creating disinformation among the members of the
CBA. said association.

Del Pilar admitted failure to deduct the agency fees from the salaries of non-union The rumors, according to Petitioners happened when Private respondents, in filing a
employees, but justifies the non-deduction by the absence of individual written authorization. complaint before the DOLE-NCR complained of petitioners’ refusal to render financial and
It posits that Article 248(e) is inapplicable considering that its employees derived no benefits other reports, and deliberate refusal to call general and special meetings. According to the
from the CBA. The annual salary of its employee is a benefit mandated by law, and not findings of CA, the financial statements for the years 1995 up to 1997 were submitted to
derived from the CBA - this argument cannot be sustained. DOLE-NCR only on 06 February 1998 while that for the year 1998 was submitted only on 16
March 1999. The last association’s meeting was conducted on 21 April 1995, and the copy of
Contrary to what Del Pilar wants to portray, the grant of annual salary increase is not the only the minutes thereon was submitted to BLR-DOLE only on 24 February 1998.
provision in the CBA that benefited the non-union employees. The union negotiated teaching
hour limitations, additional compensation for overload units, payment of longevity pay, etc. Petitioners do not hide the fact that they belatedly submitted their financial reports and the
These provisions in the CBA surely benefited the non-union employees, justifying the minutes of their meetings to the DOLE.
collection of and the union's entitlement to agency fees.
Petitioners’ Contention: The issue of belatedly submitting these reports, according to the
Accordingly, no requirement of written authorization for the non-union employees is needed petitioners, had been rendered moot and academic by their eventual compliance. Besides,
to effect a valid check off. Article 248(e) makes it explicit that Article 241(o), requiring written this has been the practice of the association. Moreover, the petitioners likewise maintain that
authorization is inapplicable to non-union employees, especially in this case where the non- the passage of General Assembly Resolution No. 10 dated 10 December 1997 and
union employees receive several benefits under the CBA. Resolution No. 8, Series of 2000, following the application of the principle that the sovereign
majority rules, cured any liability that may have been brought about by their belated actions.

ISSUE: Whether or not the non-holding of meetings and non-submission of reports by


the petitioners moot and academic, and whether the decision to hold meetings and submit
Gabriel vs. Sec. of Labor, G.R. No. 115949, March 16, 2000 reports contradict and override the sovereign will of the majority?

FACTS: Petitioners comprise the Executive Board of the SolidBank Union, the duly HELD: No. This issue was precipitated by the Court of Appeals decision affirming the order of
recognized collective bargaining agent for the rank and file employees of Solid Bank DOLE Regional Director Maximo B. Lim for the petitioners to hold a general membership
Corporation. Private respondents are members of said union. meeting wherein they make open and available the union’s/association’s books of accounts
and other documents pertaining to the union funds, and to regularly conduct special and
The Union decided to retain the service of Atty. Ignacio Lacsina as union counsel and general membership meetings in accordance with the union’s constitution and by-laws.
approved a resolution providing that 10% of the total economic benefits that may be secured
through the negotiations be given to Atty. Lacsina as attorney's fees. It also contained an The passage of General Assembly Resolution No. 10 dated 10 December 1997 and
authorization for SolidBank Corporation to check-off said attorney's fees from the first lump Resolution No. 8, Series of 2000, which supposedly cured the lapses committed by the
sum payment of benefits to the employees under the new CBA and to turn over said amount association’s officers and reiterated the approval of the general membership of the acts and
to Atty. Lacsina and/or his duly authorized representative. The bank then, on request of the collateral actions of the association’s officers cannot redeem the petitioners from their
union, made payroll deductions for attorney's fees from the CBA benefits paid to the union predicament. The obligation to hold meetings and render financial reports is mandated by
members in accordance with the abovementioned resolution. UEEA’s constitution and by-laws. This fact was never denied by the petitioners. Their
9

eventual compliance, as what happened in this case, shall not release them from the withdrew petitioner's opposition to a certification election. Petitioner appealed claiming
obligation to accomplish these things in the future. AttyBatalla was only authorized to agree to the holding of certification elections subject to the
following conditions: (1) there would only be one general election; (2) in this general election,
Prompt compliance in rendering financial reports together with the holding of regular the individual sales offices shall still comprise separate bargaining units.
meetings with the submission of the minutes thereon with the BLR-DOLE and DOLE-NCR
shall negate any suspicion of dishonesty on the part of UEEA’s officers. This is not only true ISSUE: (1) WON Union represents an appropriate bargaining unit.
with UEEA, but likewise with other unions/associations, as this matter is imbued with public (2) WON petitioner is bound by its lawyer's act of agreeing to consider the sales
interest. Undeniably, transparency in the official undertakings of union officers will bolster personnel in the north Luzon sales area as one bargaining unit.
genuine trade unionism in the country.
HELD: (1) Yes. A bargaining unit is a "group of employees of a given employer,
ISAE vs. Quisumbing, G.R. No. 128845, June 1, 2000 comprised of all or less than all of the entire body of employees, consistent with equity to the
employer, indicate to be the best suited to serve the reciprocal rights and duties of the parties
FACTS: Private respondent International School, Inc. (School), pursuant to PD 732, is a under the collective bargaining provisions of the law." The fundamental factors in determining
domestic educational institution established primarily for dependents of foreign diplomatic the appropriate collective bargaining unit are: (1) the will of the employees (Globe Doctrine);
personnel and other temporary residents. The decree authorizes the School to employ its (2) affinity and unity of the employees' interest, such as substantial similarity of work and
own teaching and management personnel selected by it either locally or abroad, from duties, or similarity of compensation and working conditions (Substantial Mutual Interests
Philippine or other nationalities, such personnel being exempt from otherwise applicable laws Rule); (3) prior collective bargaining history; and (4) similarity of employment status. The
and regulations attending their employment, except laws that have been or will be enacted for existence of a prior collective bargaining history is neither decisive nor conclusive in the
the protection of employees. School hires both foreign and local teachers as members of its determination of what constitutes an appropriate bargaining unit. Indeed, the test of grouping
faculty, classifying the same into two: (1) foreign-hires and (2) local-hires. is mutuality or commonality of interests. The employees sought to be represented by the
collective bargaining agent must have substantial mutual interests in terms of employment
The School grants foreign-hires certain benefits not accorded local-hires. Foreign-hires are and working conditions as evinced by the type of work they perform.
also paid a salary rate 25% more than local-hires.
(2) Yes. Petitioner claims that Atty. Batalla was merely a substitute lawyer for
When negotiations for a new CBA were held on June 1995, petitioner ISAE, a legitimate labor Atty. Christine Ona, who got stranded in Legaspi City. Atty. Batalla was allegedly unfamiliar
union and the collective bargaining representative of all faculty members of the School, with the collective bargaining history of its establishment. Petitioner claims it should not be
contested the difference in salary rates between foreign and local-hires. This issue, as well as bound by the mistake committed by its substitute lawyer. Insofar as the alleged "mistake" of
the question of whether foreign-hires should be included in the appropriate bargaining unit, the substitute lawyer is concerned, we find that this mistake was the direct result of the
eventually caused a deadlock between the parties. negligence of petitioner's lawyers. It will be noted that Atty. Ona was under the supervision of
two (2) other lawyers, Attys. Jacinto de la Rosa, Jr. and George C. Nograles. There is nothing
ISAE filed a notice of strike. Due to the failure to reach a compromise in the NCMB, the in the records to show that these two (2) counsels were likewise unavailable at that time.
matter reached the DOLE which favored the School. Hence this petition. Instead of deferring the hearing, petitioner's counsels chose to proceed therewith. Indeed,
prudence dictates that, in such case, the lawyers allegedly actively involved in SMC's labor
ISSUE: Whether the foreign-hires should be included in bargaining unit of local- hires. case should have adequately and sufficiently briefed the substitute lawyer with respect to the
matters involved in the case and the specific limits of his authority. Unfortunately, this was not
HELD: NO. The Constitution, Article XIII, Section 3, specifically provides that labor is entitled done in this case. The negligence of its lawyers binds petitioner.
to “humane conditions of work.” These conditions are not restricted to the physical workplace
– the factory, the office or the field – but include as well the manner by which employers treat Legend International vs. KMI. Independent, G.R. No. 169754, February 23, 2011
their employees.
FACTS: KML filed with the Med-Arbitration Unit of the DOLE, San Fernando, Pampanga, a
Discrimination, particularly in terms of wages, is frowned upon by the Labor Code. Article 248 Petition for Certification Election. LEGEND moved to dismiss the petition alleging that KML is
declares it an unfair labor practice for an employer to discriminate in regard to wages in order not a legitimate labor organization because its membership is a mixture of rank and file and
to encourage or discourage membership in any labor organization. supervisory employees in violation of Article 245 of the Labor Code. KML argued that even if
41 of its members are indeed supervisory employees and therefore excluded from its
The Constitution enjoins the State to “protect the rights of workers and promote their welfare, membership, the certification election could still proceed because the required number of the
In Section 18, Article II of the constitution mandates “to afford labor full protection”. The State total rank and file employees necessary for certification purposes is still sustained. KML also
has the right and duty to regulate the relations between labor and capital. These relations are claimed that its legitimacy as a labor union could not be collaterally attacked in the
not merely contractual but are so impressed with public interest that labor contracts, certification election proceedings but only through a separate and independent action for
collective bargaining agreements included, must yield to the common good. cancellation of union registration.

However, foreign-hires do not belong to the same bargaining unit as the local-hires. The Med-Arbiter rendered judgment dismissing for lack of merit the petition for certification
election. Since Article 245 of the Labor Code expressly prohibits supervisory employees from
A bargaining unit is a group of employees of a given employer, comprised of all or less than joining the union of rank and file employees, the Med-Arbiter concluded that KML is not a
all of the entire body of employees, consistent with equity to the employer indicate to be the legitimate labor organization.
best suited to serve the reciprocal rights and duties of the parties under the collective
bargaining provisions of the law. The Office of the Secretary of DOLE rendered its Decision granting KML’s appeal thereby
reversing and setting aside the Med-Arbiter’s Decision. The Office of the Secretary of DOLE
The factors in determining the appropriate collective bargaining unit are (1) the will of the held that KML’s legitimacy as a union could not be collaterally attacked. It declared that any
employees (Globe Doctrine); (2) affinity and unity of the employees’ interest, such as violation of the provision of Article 245 does not ipso facto render the existence of the labor
substantial similarity of work and duties, or similarity of compensation and working conditions organization illegal.
(Substantial Mutual Interests Rule); (3) prior collective bargaining history; and (4) similarity of
employment status. The basic test of an asserted bargaining unit’s acceptability is whether or LEGEND filed a Petition for Certiorari with the Court of Appeals , which found no grave abuse
not it is fundamentally the combination which will best assure to all employees the exercise of of discretion on the part of the Office of the Secretary of DOLE. LEGEND filed a Petition for
their collective bargaining rights. Certiorari with the Court of Appeals. held that the issue on the legitimacy of KML as a labor
organization has already been settled with finality in Case No. RO300-0108-CP-001. The
In the case at bar, it does not appear that foreign-hires have indicated their intention to be March 26, 2002 Decision of the Bureau of Labor Relations upholding the legitimacy of KML
grouped together with local-hires for purposes of collective bargaining. The collective as a labor organization had long become final and executory for failure of LEGEND to appeal
bargaining history in the School also shows that these groups were always treated the same.
separately. Foreign-hires have limited tenure; local-hires enjoy security of tenure. Although
foreign-hires perform similar functions under the same working conditions as the local-hires, ISSUE: Whether or not the the CA erred in denying the petition for certiorari.
foreign-hires are accorded certain benefits not granted to local-hires such as housing,
transportation, shipping costs, taxes and home leave travel allowances. These benefits are HELD: The petition is partly meritorious. LABOR LAW: Certification election
reasonably related to their status as foreign-hires, and justify the exclusion of the former from
the latter. To include foreign-hires in a bargaining unit with local-hires would not assure either Records show that (in the cancellation of registration case) LEGEND has timely filed on
group the exercise of their respective collective bargaining rights. September 6, 2002 a petition forcertiorari before the Court of Appeals which was docketed as
CA-G.R. SP No. 72659 assailing the March 26, 2002 Decision of the Bureau of Labor
San Miguel vs. Laguesma, G.R. No. 100485, September 21, 1994 Relations.

FACTS: North Luzon Magnnolia Sales Labor Union (Union) filed with the DOLE a certification However, a certification election may still be conducted during the pendency of the
election among all the regular sales personnel of Magnolia Dairy Products in the North Luzon cancellation proceedings. This is because at the time the petition for certification was filed,
Sales Area. Petitioner opposed the petition and questioned the appropriateness of the the petitioning union is presumed to possess the legal personality to file the same. There is
bargaining unit sought to be represented by respondent union. It claimed that its bargaining therefore no basis for LEGEND’s assertion that the cancellation of KML’s certificate of
history in its sales offices, plants and warehouses is to have a separate bargaining unit for registration should retroact to the time of its issuance or that it effectively nullified all of KML’s
each sales office. The petition was heard with Atty. Batallarepreseting petitioner. Atty. Batalla activities, including its filing of the petition for certification election and its demand to
10

collectively bargain. Also, the legitimacy of the legal personality of KML cannot be collaterally The election held on August 25, 1985 was not for the purpose of determining which labor
attacked in a petition for certification election proceeding. union should be the bargaining representative in the negotiation for a collective contract,
there being an existing collective bargaining agreement yet to expire on July 31, 1986; but
Algire vs. de Mesa, G.R. No. 97622, October 19, 1994 only to determine which labor union shall administer the said existing contract.

Facts: Universal Robina Textile Monthly Salaried Employees Union (URTMSEU) filed a Article 257 of the New Labor Code expressly states that No certification election issue shall
petition for the holding of an election of union officers to represent the union in the collective be entertained if a collective agreement which has been submitted in accordance with Article
bargaining agreement with the management of Universal robina Textile. The employees were 231 of the Code exists between the employer and a legitimate labor organization except
to choose between Algire, et al. and de Mesa et al. within 60 days prior to the expiration of the life of such certified collective bargaining
agreement. There should be no obstacle to the right of the employees to petition for a
Issue: Whether the election held was a consent election certification election at the proper time.

Held: Yes, it was. The election held on November 15, 1990 was a consent election and not a As aforestated, the existing collective bargaining agreement was due to expire on July 31,
certification election. It was an agreed one, the purpose being merely to determine the issue 1986. The Med-Arbiter found that the petition and intervention were supported by more than
of majority representation of all the workers in the appropriate collective bargaining unit. It is a 30% of the members of the bargaining unit. Because of this, Article 258 of the Labor Code
separate and distinct process and has nothing to do with the import and effort of a makes it mandatory for the BLR to conduct a certification election. Once it has been verified
certification election. that the petition for certification election has the support of at least 30% of the employees in
the bargaining unit, it must be granted. It becomes under the circumstances, ‘mandatory’ .
Warren Manufacturing Workers Union vs. BLR, G.R. No. 76185, March 30, 1988
UST Faculty Union vs. Gamilla, G.R. No. 131235, November 16, 1999
FACTS: On June 13, 1985, Philippine Agricultural, Commercial and Industrial Workers Union
(PACIWU) filed a petition for certification election, alleging compliance with the jurisdictional FACTS: Private Responednts are duly elected officers of the UST Faculty Union (USTFU).
requirements. The union has a subsisting five-year CBA with UST. The petitioners on the other hand,
questioned before the Med-Arbiter, that the COMELEC was not constituted in accordance
On July 7, 1985, petitioner thru counsel filed a motion to dismiss the petition on the ground with USTFU’s constitution and by-laws (CBL) and that no rules had been issued to govern
that there exists a C.B.A. between the respondent and the Warren Mfg. Union which took the conduct of the 05 October 1996 election. Med-Arbiter issued a TRO enjoining the conduct
effect upon its signing on July 16, 1985 and to expire on July 31, 1986. of elections. However, a general faculty assembly was held as scheduled. The general
assembly was attended by members of the USTFU and, as admitted by the appellants, also
While the petition was under hearing, PACIWU filed a Notice of Strike and on conciliation by “non-USTFU members [who] are members in good standing of the UST Academic
meeting, a Return-to-Work Agreement was signed and stipulating that to resolve the issue of Community Collective Bargaining Unit”. On this occasion, appellants were elected as
union representation at Warren Mfg. Corp., parties have agreed to the holding of a consent USTFU’s new set of officers by acclamation and clapping of hands.
election among the rank and file on August 25, 1985 at the premises of the company to be
supervised by MOLE. It is clearly understood that the certified union in the said projected On 03 December 1996, appellants and UST allegedly entered into another CBA covering the
election shall respect and administer the existing CBA at the company until its expiry date on period from 01 June 1996 to 31 May 2001. Said CBA was ratified by a majority of the UST
July 31, 1986. faculty community.

On 12 August 1985, an Order was issued by BLR, directing that a consent election be held ISSUE: WON the election of the officers in this case was valid
among the rank and file workers of the company, with the following contending unions:
HELD: NO. The importance of a union’s constitution and bylaws cannot be overemphasized.
1. Philippine Agricultural, Commercial and Industrial Workers Union (PACIWU); They embody a covenant between a union and its members and constitute the fundamental
2. Warren Mfg. Workers Union (WMWU); law governing the members’ rights and obligations. As such, the union’s constitution and
3. No Union. bylaws should be upheld, as long as they are not contrary to law, good morals or public
policy.
On August 25, 1985, said consent election was held, and yielded the following results:
A union election is held pursuant to the union’s constitution and bylaws, and the right to vote
PACIWU—- 94 in it is enjoyed only by union members. A union election should be distinguished from a
WMWU—-193 certification election, which is the process of determining, through secret ballot, the sole and
exclusive bargaining agent of the employees in the appropriate bargaining unit, for purposes
On June 5, 1986, the PACIWU filed a petition for certification election followed by the filing of of collective bargaining. Specifically, the purpose of a certification election is to ascertain
a petition for the same purposes by the SamahanngManggagawasa Warren Manufacturing whether or not a majority of the employees wish to be represented by a labor organization
Corporation-Alliance of Nationalist and Genuine Labor Organizations (Anglo for short) which and, in the affirmative case, by which particular labor organization.
petitions were both opposed by Warren Manufacturing Corporation on the grounds that
neither petition has 30% support; that both are barred by the one-year no certification election In a certification election, all employees belonging to the appropriate bargaining unit can vote.
law and the existence of a duly ratified CBA. Therefore, a union member who likewise belongs to the appropriate bargaining unit is entitled
to vote in said election. However, the reverse is not always true; an employee belonging to
The Med-Arbiter-NCR, MOLE ordered on August 18, 1986 the holding of a certification the appropriate bargaining unit but who is not a member of the union cannot vote in the union
election within 20 days from receipt to determine the exclusive bargaining representative of election, unless otherwise authorized by the constitution and bylaws of the union. Verily,
all the rank and file employees of the Warren Manufacturing Corporation, with the choices: union affairs and elections cannot be decided in a non-union activity.

1. Philippine Agricultural, Commercial and Industrial Workers Union (PACIWU); In both elections, there are procedures to be followed. Thus, the October 4, 1996 election
2. Warren Mfg. Workers Union; cannot properly be called a union election, because the procedure laid down in the USTFU’s
3. SamahanngManggagawasa Warren Mfg. Corporation-ANGLO; and CBL for the election of officers was not followed. It could not have been a certification election
4. No Union. either, because representation was not the issue, and the proper procedure for such election
was not followed. The participation of non-union members in the election aggravated its
Both Warren Manufacturing Corporation and petitioner filed separate motions treated as irregularity.
appeals by the BLR which was dismissed. Hence, this petition solely by WMWU. Petitioner
invoked the one-year no certification election rule and the principle of the Contract Bar Rule. Oriental Tin Can & Metal Sheet vs. Laguesma, G.R. No. 116779, August 28, 1998

ISSUE: Whether the Certification Election should be granted. FACTS: Oriental Tin Can and Metal Sheet Manufacturing Company, Inc. (the company) is
engaged in the manufacture of tin can containers and metal sheets. On March 3, 1994, it
HELD: YES. The records show that petitioner admitted that what was held on August 25, entered into a collective bargaining agreement (CBA) with petitioner Oriental Tin Can Labor
1985 at the Company’s premises and which became the root of this controversy, was a Union (OTCLU) as the existing CBA was due to expire on April 15, 1994. Four days later, 248
consent election and not a certification election. A consent election is an agreed one, its of the company’s rank-and-file employees authorized the Federation of Free Workers (FFW)
purpose being merely to determine the issue of majority representation of all the workers in to file a petition for certification election. On March 10, 1994, however, this petition was
the appropriate collective bargaining unit while a certification election is aimed at determining repudiated via a written waiver by 115 of the signatories who, along with other employees
the sole and exclusive bargaining agent of all the employees in an appropriate bargaining unit totalling 897, ratified the CBA on the same date.
for the purpose of collective bargaining. From the very nature of consent election, it is a
separate and distinct process and has nothing to do with the import and effect of a On March 18, 1994, armed with Charter Certificate No. IV-MEE-089, respondent Oriental Tin
certification election. Neither does it shorten the terms of an existing CBA nor entitle the Can Workers Union — Federation of Free Workers (OTCWU-FFW) filed a petition for
participants thereof to immediately renegotiate an existing CBA although it does not preclude certification election with the National Capital Region office of the Department of Labor and
the workers from exercising their right to choose their sole and exclusive bargaining Employment (DOLE), pursuant to Article 256 of the Labor Code. Purporting to represent the
representative after the expiration of the 60 day freedom period. regular rank-and-file employees of the company, the petition was accompanied by the
“authentic signatures” of 25% of the employees/workers in the bargaining unit.
11

The above petition for certification elections was opposed by the OTCLU. For its part, the ISSUES: (1) whether a certificate for non-forum shopping is required in a petition for
company filed a comment alleging inter alia that the new CBA was ratified by 897 out of the certification election;
1,020 rank-and-file employees within the bargaining unit. The OTCLU then filed a motion to (2) whether petitioner’s motion for reconsideration which was treated as an
dismiss and/or position paper reiterating its position that the petition did not comply with the appeal by the Secretary of Labor should not have been given due course for failure to attach
25% signature requirement and maintaining that the new CBA was a bar to a certification proof of service on respondent and
election. 3) whether petitioner had the legal personality to file the petition for certification
election.
The certification election was allowed.
HELD: (1)The CA erred in declaring that a certificate of non-forum shopping was
ISSUE: WON the company has a personality to challenge the conduct of a certification required in a petition for certification election.Notably, under the Labor Code and the rules
elections. pertaining to the form of the petition for certification election, there is no requirement for a
certificate of non-forum shopping.
HELD: NONE. It is a well-established rule that certification elections are exclusively the
concern of employees; hence, the employer lacks the legal personality to challenge the (2) As to the treatment of Motion for Reconsideration as an Appeal, the motion
same. Law and policy demand that employers take a strict, hands-off stance in certification was properly treated as an appeal because it substantially complied with the formal
elections. The bargaining representative of employees should be chosen free from any requisites.
extraneous influence of management. A labor bargaining representative, to be effective, must
owe its loyalty to the employees alone and to no other. (3) The Court agrees with the petitioner that the erroneous inclusion of one
supervisory employee in the union of rank-and-file employees was not a ground to impugn its
The only instance when an employer may concern itself with employee representation legitimacy as a legitimate labor organization which had the right to file a petition for
activities is when it has to file the petition for certification election because there is no existing certification election.
CBA in the unit and it was requested to bargain collectively, pursuant to Article 258 of the
Labor code. After filing the petition, the role of the employer ceases and it becomes a mere Oriental Tin Can & Metal Sheet vs. Laguesma, G.R. No. 116751, August 28, 1998
bystander. The company’s interference in the certification election below by actively opposing
the same is manifestly uncalled-for and unduly creates a suspicion that it intends to establish FACTS: Oriental Tin Can and Metal Sheet Manufacturing Company, Inc. (the company) is
a company union. engaged in the manufacture of tin can containers and metal sheets. On March 3, 1994, it
entered into a collective bargaining agreement (CBA) with petitioner Oriental Tin Can Labor
The designation or selection of the bargaining representative without, however, going through Union (OTCLU) as the existing CBA was due to expire on April 15, 1994. Four days later, 248
the process set out by law for the conduct of a certification election applies only when of the company’s rank-and-file employees authorized the Federation of Free Workers (FFW)
representation is not in issue. There is no problem if a union is unanimously chosen by a to file a petition for certification election. On March 10, 1994, however, this petition was
majority of the employees as their bargaining representative, but a question of representation repudiated via a written waiver by 115 of the signatories who, along with other employees
arising from the presence of more than one union in a bargaining unit aspiring to be the totalling 897, ratified the CBA on the same date.
employees’ representative, can only be resolved by holding a certification election under the
supervision of the proper government authority. On March 18, 1994, armed with Charter Certificate No. IV-MEE-089, respondent Oriental Tin
Can Workers Union — Federation of Free Workers (OTCWU-FFW) filed a petition for
NOTE: It is uncontroverted that the petition for certification election in this case was filed on certification election with the National Capital Region office of the Department of Labor and
March 18, 1994, twenty-eight days before the expiration of the existing CBA on April 15, Employment (DOLE), pursuant to Article 256 of the Labor Code. Purporting to represent the
1994, and well within the 60-day period provided for by the Code. The OTCLU, however, is regular rank-and-file employees of the company, the petition was accompanied by the
concerned with the effect of the employees’ ratification of the new CBA on the timely filing of “authentic signatures” of 25% of the employees/workers in the bargaining unit.
the petition for certification election. Would such ratification nullify the petition?
The above petition for certification elections was opposed by the OTCLU. For its part, the
The law dictates a negative reply. The filing of a petition for certification election during the company filed a comment alleging inter alia that the new CBA was ratified by 897 out of the
60-day freedom period gives rise to a representation case that must be resolved even though 1,020 rank-and-file employees within the bargaining unit. The OTCLU then filed a motion to
a new CBA has been entered into within that period. This is clearly provided for in the dismiss and/or position paper reiterating its position that the petition did not comply with the
aforequoted Section 4, Rule V, Book V of the Omnibus Rules Implementing the Labor Code. 25% signature requirement and maintaining that the new CBA was a bar to a certification
The reason behind this rule is obvious. A petition for certification election is not necessary election.The certification election was allowed.
where the employees are one in their choice of a representative in the bargaining process.
Moreover, said provision of the Omnibus Rules manifests the intent of the legislative authority ISSUE: WON the company has a personality to challenge the conduct of a certification
to allow, if not encourage, the contending unions in a bargaining unit to hold a certification elections.
election during the freedom period. The agreement prematurely signed by the union and the
company during the freedom period does not affect the petition for certification election filed HELD: NONE. It is a well-established rule that certification elections are exclusively the
by another union. concern of employees; hence, the employer lacks the legal personality to challenge the
same. Law and policy demand that employers take a strict, hands-off stance in certification
As regards the 25% support requirement, the same has been met. As previously held by the elections. The bargaining representative of employees should be chosen free from any
SC, once the required percentage requirement has been reached, the employees’ withdrawal extraneous influence of management. A labor bargaining representative, to be effective, must
from union membership (waiver in this case) taking place after the filing of the petition for owe its loyalty to the employees alone and to no other.
certification election will not affect the petition. On the contrary, the presumption arises that
the withdrawal was not free but was procured through duress, coercion or for a valuable The only instance when an employer may concern itself with employee representation
consideration. Hence, the subsequent disaffiliation of the 6 employees from the union will not activities is when it has to file the petition for certification election because there is no existing
be counted against or deducted from the previous number who had signed up for certification CBA in the unit and it was requested to bargain collectively, pursuant to Article 258 of the
Labor code. After filing the petition, the role of the employer ceases and it becomes a mere
bystander. The company’s interference in the certification election below by actively opposing
SAMMA-LIKHA vs. SAMMA Corporation, G.R. No. 167141, March 13, 2009 the same is manifestly uncalled-for and unduly creates a suspicion that it intends to establish
a company union.
FACTS: Petitioner, SAMMA-LIKHA filed a petition for certification election in the Department
of Labor and Employment (DOLE), which the respondent moved for the dismissal of said The designation or selection of the bargaining representative without, however, going through
petition. the process set out by law for the conduct of a certification election applies only when
representation is not in issue. There is no problem if a union is unanimously chosen by a
In an order, med-arbiter Arturo Cosuco ordered the dismissal of the petition on the following majority of the employees as their bargaining representative, but a question of representation
grounds: arising from the presence of more than one union in a bargaining unit aspiring to be the
employees’ representative, can only be resolved by holding a certification election under the
(1) lack of legal personality for failure to attach the certificate of registration supervision of the proper government authority.
purporting to show its legal personality; _____________________________________
(2) prohibited mixture of rank-and-file and supervisory employees and NOTE: It is uncontroverted that the petition for certification election in this case was filed on
(3) failure to submit a certificate of non-forum shopping. March 18, 1994, twenty-eight days before the expiration of the existing CBA on April 15,
1994, and well within the 60-day period provided for by the Code. The OTCLU, however, is
Petitioner then moved for reconsideration where Labor Acting Secretary Manuel Imson, concerned with the effect of the employees’ ratification of the new CBA on the timely filing of
treating the motion as an appeal, rendered a decision reversing the order of the med-arbiter. the petition for certification election. Would such ratification nullify the petition?
He ruled that the legal personality of a union cannot be collaterally attacked but may only be
questioned in an independent petition for cancellation of registration. The law dictates a negative reply. The filing of a petition for certification election during the
60-day freedom period gives rise to a representation case that must be resolved even though
Respondent filed a petition for certiorari in the CA assailing the resolution of the Secretary of a new CBA has been entered into within that period. This is clearly provided for in the
Labor but the CA reversed the same.Hence, this petition. aforequoted Section 4, Rule V, Book V of the Omnibus Rules Implementing the Labor Code.
The reason behind this rule is obvious. A petition for certification election is not necessary
12

where the employees are one in their choice of a representative in the bargaining process. construction, and transportation employees of the T. Sora campus. Thus, the computation of
Moreover, said provision of the Omnibus Rules manifests the intent of the legislative authority the quorum should be based on it, not on the total number of employees in all five campuses
to allow, if not encourage, the contending unions in a bargaining unit to hold a certification of St. James.
election during the freedom period. The agreement prematurely signed by the union and the
company during the freedom period does not affect the petition for certification election filed Phil Scout Veterans Security vs. Torres, G.R. No. 92357, July 21, 1993
by another union.
FACTS:On April 6, 1989, private respondent labor union, PGA Brotherhood Association –
As regards the 25% support requirement, the same has been met. As previously held by the Union of Filipino Workers (UFW), hereinafter referred to as “the Union ” filed a petition for
SC, once the required percentage requirement has been reached, the employees’ withdrawal Direct Certification/Certification Election among the rank and file employees of Philippine
from union membership (waiver in this case) taking place after the filing of the petition for Scout Veterans Security and Investigation Agency (PSVSIA), GVM Security and
certification election will not affect the petition. On the contrary, the presumption arises that Investigations Agency, Inc. (GVM). andAbaquin Security and Detective Agency, Inc. (ASDA).
the withdrawal was not free but was procured through duress, coercion or for a valuable These three agencies were collectively referred to by private respondent Union as the “PGA
consideration. Hence, the subsequent disaffiliation of the 6 employees from the union will not Security Agency,” which is actually the first letters of the corporate names of the agencies.
be counted against or deducted from the previous number who had signed up for certification
On April 11, 1989, summons was issued to the management of PSVSIA, GVM, ASDA (PGA
R. Transport Corp. vs. Laguesma, G.R. No. 106830, November 16, 1993 Security Agency) at 82 E. Rodriquez Avenue, Quezon City.

FACTS: On January 4, 1991, respondent Christian Labor Organization of the Philippines On April 11, 26, 1986, petitioners filed a single comment alleging therein that the said three
(CLOP), filed with the Med-Arbitration Unit of the DOLE a petition for certification election security agencies have separate and distinct corporate personalities while PGA Security
among the rank and file employees of the petitioner R Transport Corporation. Agency is not a business or corporate entity and does not possess any personality
whatsoever; the petition was unclear as to whether the rank-and-file employees mentioned
On April 8, 1991, Med-Arbiter A. Dizon dismissed the petition on the ground that the therein refer to those of the three security agencies collectively and if so, the labor union
bargaining unit sought to be represented by respondent did not include all the eligible cannot seek a certification election in three separate bargaining units in one petition.
employees of petitioner but only the drivers, conductors and conductresses to the exclusion
of the inspectors, inspectresses, dispatchers, mechanics and washerboys. Issue: WON petitioners can interfere with the certification election proceeding.

On May 10, 1991, respondent. CLOP rectified its mistake and filed a second petition for Held:Except where the employer has to file a petition for certification election pursuant to
certification election,which included all the rank and file employees of the company, who hold Article 258 of the Labor Code because of a request to bargain collectively, it has nothing to
non-managerial. and non-supervisorial positions. do with a certification election which is the sole concern of the workers. Its role in a
certification election has aptly been described in Trade Unions of the Philippines and Allied
On July 3, 1991, Med-Arbiter R. Parungo rendered a decision, which ordered that a Services (TUPAS) v. Trajano, as that of a mere by-stander. It has no legal standing in a
certification election among the regular rank and file workers of petitioner company be certification election as it cannot oppose the petition or appeal the Med-Arbiter’s orders
conducted. related thereto. An employer that involves itself in a certification election lends suspicion to
the fact that it wants to create a company union.
On October 16, 1991, the Associated Labor Unions (ALU-TUCP) filed a motion for
intervention (NCR OD-M-91-01-002) and alleged that it has members in the proposed This Court’s disapprobation of management interference in certification elections is even
bargaining unit. Subsequently, the National Federation of Labor Unions (NAFLU) filed a more forceful in Consolidated Farms, Inc. v. Noriel, where we held:
separate petition for certification election (NCR-OD-M-91-10-058) and a motion to
consolidate related cases to avoid confusion. On a matter that should be the exclusive concern of labor, the choice of a collective
bargaining representative, the employer is definitely an intruder. His participation, to say the
Petitioner argued that the second petition for a certification election by respondent CLOP, least, deserves no encouragement. This Court should be the last agency to lend support to
NAFLU and ALU-TUCP were barred at least for a period of one year from the time the first such an attempt at interference with a purely internal affair of labor.
petition of CLOP was dismissed pursuant to Section Rule V, Book V of the Omnibus Rules
Implementing the Labor Code as amended. DHL Philippines Corp. vs. BuklodngManggagawang DHL Philippines Corp., G.R. No.
152094, July 22, 2004
ISSUE: WON the first petition bars the certification election for one year based on Facts:
Certification-bar-rule.
Tagaytay Highlands International Golf Club vs. Tagaytay Highlands Employees’ Union, G.R.
HELD: No. Section 3, Rule V, Book V of the Omnibus Rules Implementing the Labor Code as No. 142000, January 22, 2003
amended provides as follows:
FACTS: On October 16, 1997, the Tagaytay Highlands Employees Union (THEU)–Philippine
When to file — In the absence of collective bargaining agreement duly registered in Transport and General Workers Organization (PTGWO) representing majority of the rank-
accordance with Article 231 of the Code, a petition for certification election may be filed any and-file employees of THIGCI, filed a petition for certification election before the DOLE.
time. However, no certification election may be held within one year from the date of the
issuance of a final certification election result (Emphasis supplied). THIGCI, opposed THEU’s petition on the ground that the list of union members submitted by
it was defective and fatally flawed as it included the names and signatures of supervisors,
The phrase "final certification election result" means that there was an actual conduct of resigned, terminated and absent without leave (AWOL) employees, as well as employees of
election i.e. ballots were cast and there was a counting of votes. In this case, there was no The Country Club, Inc., a corporation distinct and separate from THIGCI; and that out of the
certification election conducted precisely because the first petition was dismissed, on the 192 signatories to the petition, only 71 were actual rank-and-file employees of THIGCI.
ground of a defective petition which did not include all the employees who should be properly
included in the collective bargaining unit. THIGCI also alleged that some of the signatures in the list of union members were secured
through fraudulent and deceitful means, and submitted copies of the handwritten denial and
St. James School of Quezon City vsSamahangManggagawasa St. James School of Quezon withdrawal of some of its employees from participating in the petition. THEU asserted that it
City, G.R. No. 151326, November 23, 2005 had complied with all the requirements for valid affiliation and inclusion in the roster of
legitimate labor organizations pursuant to DOLE Department Order No. 9, series of 1997,on
FACTS: Respondent SamahangManggagawa filed a petition for a certification election to account of which it was duly granted a Certification of Affiliation by DOLE on October 10,
determine the collective bargaining representative of the motorpool, construction and 1997 and that Section 5, Rule V of said Department Order provides that the legitimacy of its
transportation employees of St. James School of QC. The certification election was held at registration cannot be subject to collateral attack, and for as long as there is no final order of
the DOLE office in which there were 149 eligible voters. 84 cast their votes. St. James filed a cancellation, it continues to enjoy the rights accorded to a legitimate organization.
protest challenging the 84 votes, alleging that it had 179 rank and file employees and none of
them voted in the certification election. It argued that those who voted were not its regular ISSUE: W/N supervisory employees are prohibited from joining a labor union.
employees but construction workers of an independent contractor.
HELD: NO. The statutory authority for the exclusion of supervisory employees in a rank-and-
ISSUES: 1. WON the formation of the labor union was valid file union, and vice-versa, is Article 245 of the Labor Code, to wit:
2. WON the certification election was valid
Article 245. Ineligibility of managerial employees to join any labor organization; right of
HELD: 1. Yes. The records show that prior to the holding of a certification election, St. supervisory employees. — Managerial employees are not eligible to join, assist or form any
James filed a petition for cancellation of SamahangManggagawa on the ground of lack of labor organization. Supervisory employees shall not be eligible for membership in a labor
employer-employee relations between the union members and St. James. This went all the organization of the rank-and-file employees but may join, assist or form separate labor
way up to the CA which ruled that the independent contractor is a labor only contractor, and organizations of their own.
thus, an agent of St. James which is the real employer.
While above-quoted Article 245 expressly prohibits supervisory employees from joining a
2. Yes. St. James has five campuses. The members of the Samahan are rank-and-file union, they were not prohibited from forming, assisting and joining their own
employees of the TandangSora campus. The Samahan seeks to represent the motor pool, union.
13

TOTAL VOTES CAST - 401[4]


As for the lack of mutuality of interest argument of petitioner, it, at all events, does not lie
given, as found by the court a quo, its failure to present substantial evidence that the assailed Yokohama challenged 78 votes cast by dismissed employees. On the other hand, the Union
employees are actually occupying supervisory positions. challenged 68 votes cast by newly regularized rank-and-file employees and another five (5)
votes by alleged supervisor-trainees.
While petitioner submitted a list of its employees with their corresponding job titles and
ranks,24 there is nothing mentioned about the supervisors’ respective duties, powers and The CA held that the 78 employees who contested their dismissal were entitled to vote under
prerogatives that would show that they can effectively recommend managerial actions which Article 212 (f) of the Labor Code and Section 2, Rule XII of the rules implementing Book V of
require the use of independent judgment.25 the Labor Code. However, it disallowed the votes of the 68 newly regularized employees
since they were not included in the voters list submitted during the July 12, 2001 pre-election
As this Court put it in Pepsi-Cola Products Philippines, Inc. v. Secretary of Labor: conference.

Designation should be reconciled with the actual job description of subject employees x ISSUE: WON the votes of the 78 employees who were dismissed but their cases for illegal
xxThe mere fact that an employee is designated manager does not necessarily make him dismissal were challeged, should be allowed
one. Otherwise, there would be an absurd situation where one can be given the title just to be
deprived of the right to be a member of a union. In the case of National Steel Corporation vs. HELD: YES. Section 2, Rule XII, the rule in force during the November 23, 2001 certification
Laguesma (G. R. No. 103743, January 29, 1996), it was stressed that: election clearly, unequivocally and unambiguously allows dismissed employees to vote
during the certification election if the case they filed contesting their dismissal is still pending
What is essential is the nature of the employee’s function and not the nomenclature or title at the time of the election.
given to the job which determines whether the employee has rank-and-file or managerial
status or whether he is a supervisory employee. (Emphasis supplied). Even the new rule has explicitly stated that without a final judgment declaring the legality of
dismissal, dismissed employees are eligible or qualified voters. Thus,
Acoje Workers Union vs. NAMAWU, G.R. No. L-18848, April 23, 1943
RULE IX
FACTS:Department of Labor, through the BLR, conducted on June 9, 1961, a “consent CONDUCT OF CERTIFICATION ELECTION
election” among the workers of the Acoje Mining Company at Santa Cruz, Zambales, in Section 5. Qualification of voters; inclusion-exclusion. . . . An employee who has been
which 5 labor unions participated, namely, the Acoje United Workers’ Union, the Acoje Labor dismissed from work but has contested the legality of the dismissal in a forum of appropriate
Union (PELTA), the Acoje Labor Union (PLUM), respondent National Mines and Allied jurisdiction at the time of the issuance of the order for the conduct of a certification election
Workers’ Union (NAMAWU), and petitioner Acoje Workers’ Union. NAMAWU won in the said shall be considered a qualified voter, unless his/her dismissal was declared valid in a final
election. judgment at the time of the conduct of the certification election.
XXX
Petitioner Union — which had been defeated by respondent Union by a margin of 282 votes
— had filed a motion to invalidate said election upon several grounds. After hearing, the SamahanngManggagawasa Pacific Plastic vs. Laguesma, G.R. No. 111245, January 31,
lower court issued, on July 21, 1961, the order appealed from holding that said motion was 1997
without merit, and certifying respondent Union NAMAWU as the sole and exclusive
bargaining agent of all the workers of the Company. MR of petitioner was denied, hence this FACTS: Petitioner SAMAHAN and respondent MNMPP are labor unions of rank and file
present appeal by certiorari, and petitioner now maintains that the lower court should have employees at the Pacific Plastic Corporation (PPC) in Valenzuela, Metro Manila. On August
invalidated the aforementioned election for the same was “the result of acts of terrorism, 24, 1990, MNMPP filed a Petition for Certification Election, alleging that there were more or
force, threat and intimidation employed by” agents of respondent Union. The petitioner also less 130 rank and file employees at the PPC whom it was seeking to represent. SAMAHAN
questioned the list of qualified voters that was used during the election which was based on countered by seeking the cancellation of MNMPPs union registration. As a result, MNMPPs
the payroll of the employees. petition to be certified as the bargaining agent was dismissed. MNMPP appealed to the
Secretary of Labor who, on March 5, 1991, reversed the decision of the Med-Arbiter and
ISSUE:Can a payroll be used as the basis for qualified employee- voters? ordered the holding of a certification election among the rank and file employees of the PPC.
The PPC filed a Motion for Reconsideration but its motion was denied. Accordingly, the
HELD:YES. It appears that labor unions concerned agreed, not only to the holding of the representation officer of the Secretary of Labor held a pre-election conference on May 6,
aforementioned election, but also to the use of the Company payroll of March 31, 1961, as 1991, during which the PPC was required to submit the list of its rank and file employees
the basis for determining who are qualified to vote subject to the approval of the lower court. based on the company payroll three (3) months prior to the filing of the petition. Respondent
The Company presented its aforementioned payroll to said court and stated that the labor company failed to submit the list even after three DOLE orders. Meanwhile, on September
unions had been furnished copy thereof, at least 3 days prior thereto. Said labor unions were 23, 1991, SAMAHAN and MNMPP agreed to hold the certification election on October 29,
given an opportunity to make their comments and observations on the list of workers 1991 on the basis of the list of employees submitted by MNMPP, without prejudice to the
contained in the payroll and to ask or suggest the inclusion or exclusion of names therein or submission by petitioner SAMAHAN of its own list on October 17, 1991. Thereafter, they
therefrom. Petitioner’s representative then stated that it would abide by whatever ruling agreed to postpone election to await the list of employees requested from the Social Security
the court may make on the matter of inclusion and exclusion of voters. Indeed, on May 19, System.
1961, the court issued the corresponding order for the holding of the election and made its
ruling on the question as to who were qualified to vote, and petitioner did not move for a The Certification Election was held on October 6, 1992 with 62 votes cast out of 98 Eligible
reconsideration of said ruling. Hence, petitioner may no longer contest the accuracy of the voters on the list of SSS.
aforementioned voters list.
ISSUE: WON the SSS lists indicating 98 covered employees can be used as substitute for
As to the allegation of petitioner that there are many cases where the workers were three (3) monthly payrolls required for the purpose of determining the qualified voters and the
threatened, coerced and intimidated to vote for the NAMAWU, is but a general allegation, majority vote needed in an election.
without anything to indicate the number of workers involved, without the supporting affidavit
of any of them, and without an offer to introduce their testimony or the testimony of any of HELD: Yes. It should ideally be the payroll which should have been used for the purpose of
them was. Petitioner’s contention is insufficient to warrant the invalidation of the the election. However, the unjustified refusal of a company to submit the payroll in its
aforementioned election. custody, despite efforts to make it produce it, compelled resort to the SSS list as the next
best source of information. After all, the SSS list is a public record whose regularity is
Yokohama Tire Phils. Vs. Yokohama Employees Union, G.R. No. 159553, December 10, presumed. In Port Workers Union of the Philippines (PWUP) v. Undersecretary of Labor and
2007 Employment, this Court underscored the policy of the Labor Code of encouraging the holding
of a certification election as the definitive and certain way of ascertaining the choice of
FACTS: On October 7, 1999, respondent Yokohama Employees Union (Union) filed a petition employees as to the labor organization in a collective bargaining unit.
for certification election among the rank-and-file employees of Yokohama. Upon appeal from
the Med-Arbiters order dismissing the petition, the Secretary of the Department of Labor and Petitioners objection to the use of the SSS should have been raised during the elections and
Employment (DOLE) ordered an election with (1) Yokohama Employees Union and (2) No formalized in its election protest. The SC agrees with private respondent MNMPP in its
Union as choices.[3] The election held on November 23, 2001 yielded the following result: Opposition to SAMAHANs election protest dated October 15, 1992 that under the
Implementing Rules, grounds of protests not raised before the close of the proceedings and
YOKOHAMA EMPLOYEES UNION - 131 duly formalized within five (5) days after the close of the election proceedings are deemed
NO UNION - 117 waived.
SPOILED - 2
----- Cirtek Employees Labor Union-FFW vs. Cirtek Electronics, G.R. No. 190515, June 6, 2011
250
Facts: This resolves the motion for reconsideration and supplemental motion for
VOTES CHALLENGED BY [YOKOHAMA] - 78 reconsideration filed by respondent, Cirtek Electronics, Inc., of the Court’s Decision dated
VOTES CHALLENGED BY [UNION] - 73 November 15, 2010.
------
TOTAL CHALLENGED VOTES - 151
14

Respondent-movant maintains that the Secretary of Labor cannot insist on a ruling beyond NATU. It appears, however, that NATU itself as a labor federation, was not registered with
the compromise agreement entered into by the parties; and that, as early as February 5, the Department of Labor.
2010, petitioner Union had already filed with the Department of Labor and Employment
(DOLE) a resolution of disaffiliation from the Federation of Free Workers resulting in the Collective Bargaining Agreement was concluded between the parties on April 1, 1968, the
latter’s lack of personality to represent the workers in the present case. term of which expired on March 31, 1971.

Issue: WON petitioner lost its personality to represent the workers because of its disaffiliation Sec. 1. The COMPANY recognizes the UNION as the sole and exclusive collective
from the Federation of Free Workers. bargaining agent for all its workers and employees in all matters concerning wages, hours of
work, and other terms and conditions of employment.
Held:The issue of disaffiliation is an intra-union dispute which must be resolved in a different
forum in an action at the instance of either or both the FFW and the Union or a rival labor Sec. 1 —. . . Employees who are already members of the UNION at the time of the signing of
organization, not the employer. this Agreement or who become so thereafter shall be required to maintain their membership
therein as a conditionof continued employment. xxx
Indeed, as respondent-movant itself argues, a local union may disaffiliate at any time from its
mother federation, absent any showing that the same is prohibited under its constitution or Sec. 3—Any employee who is expelled from the UNION for joining another federation or
rule. Such, however, does not result in it losing its legal personality altogether. Verily, Anglo- forming another union, or who fails or refuses to maintain his membership therein as
KMU v. Samahan Ng MgaManggagawangNagkakaisaSa Manila Bay Spinning Mills At J.P. required, . . . shall, upon written request of the UNION be discharged by the COMPANY.
Coats enlightens:
May 21, 1971, respondent company and THEU-NATU entered into a new Collective
A local labor union is a separate and distinct unit primarily designed to secure and maintain Bargaining Agreement which ended on March 31, 1974. This new CBA incorporated the
an equality of bargaining power between the employer and their employee-members. A local previous union-shop security clause and the attached checkoff authorization form.
union does not owe its existence to the federation with which it is affiliated. It is a separate
and distinct voluntary association owing its creation to the will of its members. The mere act NATU received a letter dated December 15, 1973, jointly signed by the incumbent officers of
of affiliation does not divest the local union of its own personality, neither does it give the the local union informing the NATU that THEU was disaffiliating from the NATU federation.
mother federation the license to act independently of the local union. It only gives rise to a Secretary of the THEU, NemesioBarro, made an announcement in an open letter to the
contract of agency where the former acts in representation of the latter. (emphasis and general membership of the THEU, concerning the latter’s disaffiliation from the NATU and its
underscoring supplied) affiliation with the Confederation of General Workers (CGW). The letter was passed around
among the members of the THEU-NATU, to which around one hundred and thirtyseven (137)
MR denied. signatures appeared as having given their consent to and acknowledgment of the decision to
disaffiliate the THEU from the NATU.
Philippine Skylanders, Inc. vs. NLRC, G.R. No. 127374, January 31, 2002
THEU-CGW held its annual election of officers, with Jose Encinas elected as President. On
FACTS: In November 1993 the Philippine Skylanders Employees Association (PSEA), a local January 3, 1974, Encinas, in his capacity as THEU-CGW President, informed the respondent
labor union affiliated with the Philippine Association of Free Labor Unions (PAFLU), won in company of the result of the elections. On January 9, 1974, PacificoRosal, President of the
the certification election conducted among the rank and file employees of Philippine Confederation of General Workers (CGW), wrote a letter in behalf of complainant THEU-
Skylanders, Inc. (PSI). Its rival union, Philippine Skylanders Employees Association-WATU CGWto the respondent company demanding the remittance of the union dues collected by
(PSEA-WATU) immediately protested the result of the election before the Secretary of Labor. the Tropical Hut Food Mart, Incorporated to the THEU-CGW, but this was refused by the
Several months later, PSEA sent PAFLU a notice of disaffiliation. PSEA subsequently respondent company.
affiliated itself with the National Congress of Workers (NCW), changed its name to Philippine
Skylanders Employees Association – National Congress of Workers (PSEA-NCW), and to A request made by the NATU federation to the respondent company to dismiss him (Encinas)
maintain continuity within the organization, allowed the former officers of PSEA-PAFLU to in view of his violation of Section 3 of Article III of the Collective Bargaining Agreement. At the
continue occupying their positions as elected officers in the newly-formed PSEA-NCW. On 17 request of NATU, respondent company applied for clearance with the Secretary of Labor to
March 1994 PSEA-NCW entered into a collective bargaining agreement with PSI which was dismiss the other officers and members of THEU-CGW. The company also suspended them
immediately registered with the Department of Labor and Employment. Meanwhile, effective that day. NLRC Case No. LR-2521 was filed by THEU-CGW and individual
apparently oblivious to PSEA’s shift of allegiance, PAFLU Secretary General SerafinAyroso complainants against private respondents for unfair labor practices.
wrote Mariles C. Romulo requesting a copy of PSI’s audited financial statement. On 30 July
1994 PSI through its personnel manager Francisco Dakila denied the request citing as Acting as temporary chairman, presided over the election of officers of the remaining THEU-
reason PSEA’s disaffiliation from PAFLU and its subsequent affiliation with NCW. NATU in an emergency meeting pending the holding of a special election to be called at a
later date.THEU-CGW asked the employees to affirm their membership. Some did not
Issue: WON PSEA’s disaffiliation is legitimate. abideso theywere informed that they will be dismissed under the CBA.

Held: At the outset, let it be noted that the issue of disaffiliation is an inter-union conflict the President/General Manager of respondent company, upon Dilag’s request, suspended twenty
jurisdiction of which properly lies with the Bureau of Labor Relations (BLR) and not with the four (24) workers on March 5, 1974,another thirty seven (37) on March 8, 1974 and two (2)
Labor Arbiter. We upheld the right of local unions to separate from their mother federation on more on March 11, 1974, pending approval by the Secretary of Labor of the application for
the ground that as separate and voluntary associations, local unions do not owe their creation their dismissal.
and existence to the national federation to which they are affiliated but, instead, to the will of
their members. Yet the local unions remain the basic units of association, free to serve their Labor Arbiter, Arbitrator Daniel Lucas issued an orderdated March 21, 1974, holding that the
own interests subject to the restraints imposed by the constitution and by-laws of the national issues raised by the parties became moot and academic with the issuance of NLRC Order
federation, and free also to renounce the affiliation upon the terms laid down in the dated February 25, 1974 in NLRC Case No. LR-2670, which directed the holding of a
agreement which brought such affiliation into existence. There is nothing shown in the certification election among the rank and file workers of the respondent company between
records nor is it claimed by PAFLU that the local union was expressly forbidden to disaffiliate the THEU-NATU and THEUCGW. He also ordered: a) the reinstatement of all complainants;
from the federation nor were there any conditions imposed for a valid breakaway. As such, b) for the respondent company to cease and desist from committing further acts of dismissals
the pendency of an election protest involving both the mother federation and the local union without previous order from the NLRC and for the complainant Tropical Hut Employees
did not constitute a bar to a valid disaffiliation. It was entirely reasonable then for PSI to enter UNION-CGW to file representation cases on a case to case basis during the freedom period
into a collective bargaining agreement with PSEA-NCW. As PSEA had validly severed itself provided for by the existing CBA between the parties.
from PAFLU, there would be no restrictions which could validly hinder it from subsequently
affiliating with NCW and entering into a collective bargaining agreement in behalf of its NLRC reversed the decision. Secretary of Labor rendered a decision affirming the findings of
members. Policy considerations dictate that in weighing the claims of a local union as against the Commission.
those of a national federation, those of the former must be preferred. Parenthetically though,
the desires of the mother federation to protect its locals are not altogether to be shunned. It ISSUE: 1) whether or not the petitioners failed to exhaust administrative remedies when
will however be to err greatly against the Constitution if the desires of the federation would be they immediately elevated the case to this Court without an appeal having been made to the
favored over those of its members. That, at any rate, is the policy of the law. For if it were Office of the President;
otherwise, instead of protection, there would be disregard and neglect of the lowly 2) whether or not the disaffiliation of the local union from the national federation
workingmen. was valid; and
3) whether or not the dismissal of petitioner employees resulting from their
Tropical Hut Employees Union vs. Tropical Hut, G.R. No. L-43495-99, January 20, 1999 unions disaffiliation for the mother federation was illegal and constituted unfair labor practice
on the part of respondent company and federation
FACTS: January 2, 1968, the rank and file workers of the Tropical Hut Food Market
Incorporated, referred to herein as respondent company, organized a local union called the RULING: 1) remedy of appeal from the Secretary of Labor to the Office of the President is
Tropical Hut Employees Union, known for short as the THEU, elected their officers, adopted not a mandatory requirement before resort to courts can be had, but an optional relief
their constitution and by-laws and immediately sought affiliation with the National Association provided by law to parties seeking expeditious disposition of their labor disputes. Failure to
of Trade Unions (NATU). On January 3, 1968, the NATU accepted the THEU application for avail of such relief shall not in any way served as an impediment to judicial intervention. And
affiliation. Following such affiliation with NATU, Registration Certificate No. 5544-IP was where the issue is lack of power or arbitrary or improvident exercise thereof, decisions of the
issued by the Department of Labor in the name of the Tropical Hut Employees Union —
15

Secretary of Labor may be questioned in acertiorariproceeding without prior appeal to the August 21, 2000, Ventures filed a Petition to cancel the Union’s certificate of registration
President. alleging that the Union deliberately and maliciously included the names of more or less 82
former employees no longer connected with Ventures in its list of members who attended the
2) local union, being a separate and voluntary association, is free to serve the organizational meeting and in the adoption/ratification of its constitution and by-laws; that No
interest of all its members including the freedom to disaffiliate when circumstances warrant. organizational meeting and ratification actually took place; and the Union’s application for
This right is consistent with the constitutional guarantee of freedom of association. All registration was not supported by at least 20% of the rank-and-file employees of Ventures.
employees enjoy the right to self organization and to form and join labor organizations of their
own choosing for the purpose of collective bargaining and to engage in concerted activities Regional Director of DOLE- Region III favored Ventures and resolved to Cancel the
for their mutual aid or protection. This is a fundamental right of labor that derives its existence Certificate of the union. On appeal, the BLR Director granted the Union’s appeal and
from the Constitution. reversing the decision of RD. Ventures went to the CA. The CA dismissed Ventures’ petition
as well as the MR. Hence, this petition for review
The inclusion of the word NATU after the name of the local union THEU in the registration
with the Department of Labor is merely to stress that the THEU is NATU’s affiliate at the time ISSUE:Whether the registration of the Union must be cancelled.
of the registration. It does not mean that the said local union cannot stand on its own. Neither
can it be interpreted to mean that it cannot pursue its own interests independently of the HELD:NO. The right to form, join, or assist a union is specifically protected by Art. XIII,
federation. A local union owes its creation and continued existence to the will of its members Section 3 of the Constitution and such right, according to Art. III, Sec. 8 of the Constitution
and not to the federation to which it belongs. When the local union withdrew from the old and Art. 246 of the Labor Code, shall not be abridged. Once registered with the DOLE, a
federation to join a new federation, it was merely exercising its primary right to labor union is considered a legitimate labor organization endowed with the right and privileges
organization for the effective enhancement and protection of common interests. In the granted by law to such organization. While a certificate of registration confers a union with
absence of enforceable provisions in the federation’s constitution preventing disaffiliation of a legitimacy with the concomitant right to participate in or ask for certification election in a
local union a local may sever its relationship with its parent. Nothing in the constitution and by bargaining unit, the registration may be canceled or the union may be decertified as the
laws of THEU NATU, prohibits the disaffiliation from NATU. Besides NATU is not even bargaining unit, in which case the union is divested of the status of a legitimate labor
recognized as a national federation. organization. Among the grounds for cancellation is the commission of any of the acts
enumerated in Art. 239(a) of the Labor Code, such as fraud and misrepresentation in
3) When the THEU disaffiliated from its mother federation, the former did not connection with the adoption or ratification of the union’s constitution and like documents.
lose its legal personality as the bargaining union under the CBA. Moreover, the union security The Court, has in previous cases, said that to decertify a union, it is not enough to show that
clause embodied in the agreements cannot be used to justify thedismissals meted to the union includes ineligible employees in its membership. It must also be shown that there
petitioners since it is not applicable to the circumstances obtaining in this case. The CBA was misrepresentation, false statement, or fraud in connection with the application for
imposes dismissal only in case an employee is expelled from the union for joining another registration and the supporting documents, such as the adoption or ratification of the
federation or for forming another union or who fails or refuses to maintain membership constitution and by-laws or amendments thereto and the minutes of ratification of the
therein. The case at bar does not involve the withdrawal of merely some employees from the constitution or by-laws, among other documents.
union but of the whole THEU itself from its federation. Clearly, since there is no violation of
the union security provision in theCBA, there was no sufficient ground to terminate the The evidence presented by Ventures consist mostly of separate hand-written statements of
employment of petitioners. 82 employees who alleged that they were unwilling or harassed signatories to the attendance
sheet of the organizational meeting. However these evidence was presented seven months
Elisco-Elirol Labor Union (NAFLU) vs. Noriel, G.R. No. L-41995, December 29, 1977 after the union filed its petition for cancellation of registration. Hence these statements
partake of the nature of withdrawal of union membership executed after the Union’s filing of a
Facts: Elisco-Elirol Labor Union (NAFLU) negotiated and executed a CBA with Elizalde Steel petition for certification election on March 21, 2000. We have said that the employees’
Consolidated Inc. However, Elisco-Elirol then was not yet a registered union. In order to be withdrawal from a labor union made before the filing of the petition for certification election is
able to execute the CBA, they had the union registered, which was granted. They likewise presumed voluntary, while withdrawal after the filing of such petition is considered to be
moved to disaffiliate themselves with NAFLU. Elizalde, however, refused to recognized them involuntary and does not affect the same. Now then, if a withdrawal from union membership
as the SEBA and it dismissed the officers of the union because of the union security clause in done after a petition for certification election has been filed does not vitiate such petition, it is
the CBA. Elisco-Elirol filed a complaint for unfair labor practice with the BLR. The BLR but logical to assume that such withdrawal cannot work to nullify the registration of the union.
dismissed. The Court is inclined to agree with the CA that the BLR did not abuse its discretion nor
gravely err when it concluded that the affidavits of retraction of the 82 members had no
Issue: Whether or not Elisco-Elirol is the SEBA evidentiary weight.

Held: YES. The error of BLR is not perceiving that the employees and members of the local The registration or the recognition of a labor union after it has submitted the corresponding
union did not form a new union but merely registered the local union as was their right. papers is not ministerial on the part of the BLR. It becomes mandatory for the BLR to check if
Petitioner Elisco-Elirol Labor Union-NAFLU, consisting of employees and members of the the requirements under Art. 234 of the Labor Code have been sedulously complied with. If
local union was the principal party to the agreement. NAFLU as the mother union" in the union’s application is infected by falsification and like serious irregularities, especially
participating in the execution of the bargaining agreement with respondent company acted those appearing on the face of the application and its attachments, a union should be denied
merely as agent of the local union, which remained the basic unit of the association existing recognition as a legitimate labor organization. The issuance to the Union of Certificate of
principally and freely to serve the common interest of all its members, including the freedom Registration, in the case at bar, necessarily implies that its application for registration and the
to disaffiliate when the circumstances so warranted as in the present case. supporting documents thereof are prima facie free from any vitiating irregularities.

"(T)he locals are separate and distinct units primarily designed to secure and maintain an The relevance of the 82 individuals’ active participation in the Union’s organizational meeting
equality of bargaining power between the employer and their employee-members in the and the signing ceremonies thereafter comes in only for purposes of determining whether or
economic struggle for the fruits of the joint productive effort of labor and capital; and the not the Union, even without the 82, would still meet what Art. 234(c) of the Labor Code
association of the locals into the national union (as PAFLU) was in furtherance of the same requires to be submitted, requiring that the union applicant must file the names of all its
end. These associations are consensual entities capable of entering into such legal relations members comprising at least twenty percent (20%) of all the employees in the bargaining unit
with their members. The essential purpose was the affiliation of the local unions into a where it seeks to operate.
common enterprise to increase by collective action the common bargaining power in respect
of the terms and conditions of labor. Yet the locals remained the basic units of association, In its union records on file with this Bureau, respondent union submitted the names of 542
free to serve their own and the common interest of all, subject to the restraints imposed by members. This number easily complied with the 20% requirement, be it 1,928 or 2,202
the Constitution and By-Laws of the Association, and free also to renounce the affiliation for employees in the establishment. Even subtracting the 82 employees from 542 leaves 460
mutual welfare upon the terms laid down in the agreement which brought it into existence." union members, still within 440 or 20% of the maximum total of 2,202 rank-and-file
(Liberty Cotton Mills Workers Union vs. Liberty Cotton Mills Inc.) employees of the employer Venture.

Such maintenance of the membership clause could not be so distorted.. What is paramount Whatever misgivings the petitioner may have with regard to the 82 dismissed employees is
is the security of tenure of the workers and not the security of the union. better addressed in the inclusion-exclusion proceedings during a pre-election conference.
The issue surrounding the involvement of the 82 employees is a matter of membership or
S.S. Ventures International, Inc. vs. S.S. Ventures Labor Union, G.R. No. 161690, July 23, voter eligibility. It is not a ground to cancel union registration.
2008
For fraud and misrepresentation to be grounds for cancellation of union registration under
FACTS:Petitioner S.S. Ventures International, Inc. (Ventures), a PEZA- registered export firm Article 239, the nature of the fraud and misrepresentation must be grave and compelling
with principal place of business at Phase I-PEZA- Bataan Export Zone, Mariveles, Bataan, is enough to vitiate the consent of a majority of union members.
in the business of manufacturing sports shoes. Respondent S.S. Ventures Labor Union
(Union) is a labor organization registered with the DOLE. Kiok Loy vs. NLRC, G.R. No. 54334, January 22, 1986

March 21, 2000, the Union filed with DOLE-Region III a petition for certification election in Doctrine: Unfair labor practice is committed when it is shown that the respondent employer,
behalf of the rank-and-file employees after having been served with a written bargaining proposal by the petitioning Union, did not
even bother to submit an answer or reply to the said proposal.
16

Facts:The PambansangKilusangPaggawa, a legitimate late labor federation, won and was reason to refuse to negotiate in good faith with the union. For refusing to send a counter-
subsequently certified in a resolution by the Bureau of Labor Relations as the sole and proposal to the union and to bargain anew on the economic terms of the CBA, the company
exclusive bargaining agent of the rank-and-file employees of Sweden Ice Cream Plant. committed an unfair labor practice under Article 248 of the Labor Code.

The Union furnished the Company with two copies of its proposed collective bargaining ART. 253-A. Terms of a collective bargaining agreement. – Any Collective Bargaining
agreement. At the same time, it requested the Company for its counter proposals. Both Agreement that the parties may enter into shall, insofar as the representation aspect is
requests were ignored and remained unacted upon by the Company. concerned, be for a term of five (5) years. No petition questioning the majority status of the
incumbent bargaining agent shall be entertained and no certification election shall be
Thereafter, the Union filed a "Notice of Strike", with the Bureau of Labor Relations (BLR) on conducted by the Department of Labor and Employment outside of the sixty-day period
ground of unresolved economic issues in collective bargaining. immediately before the date of expiry of such five year term of the Collective Bargaining
Agreement. All other provisions of the Collective Bargaining Agreement shall be renegotiated
Conciliation proceedings then followed during the thirty-day statutory cooling-off period. But not later than three (3) years after its execution….
all attempts towards an amicable settlement failed.
ART. 248. Unfair labor practices of employers. – It shall be unlawful for an employer to
The case was brought to the National Labor Relations Commission (NLRC) for compulsory commit any of the following unfair labor practice:
arbitration pursuant to Presidential Decree No. 823, as amended. But the Company
requested for a lot of postponements. NLRC ruled that respondent Sweden Ice Cream is (g) To violate the duty to bargain collectively as prescribed by this Code;
guilty of unjustified refusal to bargain, in violation of Section (g) Article 248 (now Article 249),
of P.D. 442, as amended. Under Article 252 abovecited, both parties are required to perform their mutual obligation to
meet and convene promptly and expeditiously in good faith for the purpose of negotiating an
Issue: Whether the Company is guilty of unfair labor practice for refusal to bargain agreement. The union lived up to this obligation when it presented proposals for a new CBA
to GMC within three (3) years from the effectivity of the original CBA. But GMC failed in its
Held: Yes. Petition dismissed for lack of merit. Collective bargaining is one of the democratic duty under Article 252. What it did was to devise a flimsy excuse, by questioning the
frameworks under the New Labor Code, designed to stabilize the relation between labor and existence of the union and the status of its membership to prevent any negotiation.
management and to create a climate of sound and stable industrial peace. It is a mutual
responsibility of the employer and the Union and is characterized as a legal obligation. ART. 250. Procedure in collective bargaining. – The following procedures shall be observed
in collective bargaining:
Article 249, par. (g) of the Labor Code makes it an unfair labor practice for an employer to
refuse "to meet and convene promptly and expeditiously in good faith for the purpose of (a) When a party desires to negotiate an agreement, it shall serve a written
negotiating an agreement with respect to wages, hours of work, and all other terms and notice upon the other party with a statement of its proposals. The other party
conditions of employment including proposals for adjusting any grievance or question arising shall make a reply thereto not later than ten (10) calendar days from receipt of
under such an agreement and executing a contract incorporating such agreement, if such notice.
requested by either party.
GMC’s failure to make a timely reply to the proposals presented by the union is indicative of
The mechanics of collective bargaining are set in motion only when the following jurisdictional its utter lack of interest in bargaining with the union. Its excuse that it felt the union no longer
preconditions are present, namely, represented the workers, was mainly dilatory as it turned out to be utterly baseless.

(1) possession of the status of majority representation of the employees' Failing to comply with the mandatory obligation to submit a reply to the union’s proposals,
representative in accordance with any of the means of selection or designation GMC violated its duty to bargain collectively, making it liable for unfair labor practice.
provided for by the Labor Code;
(2) proof of majority representation; and San Pedro Hospital of Digos vs. Secretary of Labor, G.R. No. 104624, October 11, 2006
(3) a demand to bargain under Article 251, par. (a) of the New Labor Code.
FACTS: Petitioner had a three-year collective bargaining agreement (CBA) covering the
A Company's refusal to make counter proposal if considered in relation to the entire period December 15, 1987 until December 15, 1990, with herein private respondent,
bargaining process, may indicate bad faith since the Union's request for a counter proposal is NagkabiusangMamumuosa San Pedro Hospital of Digos — National Federation of Labor
left unanswered. Besides, petitioner Company's approach and attitude-stalling the negotiation (NAMASAP-NFL), the exclusive bargaining agent of the hospital’s rank-and-file workers. After
by a series of postponements, non-appearance at the hearing conducted, and undue delay in the parties failed to reach agreement on the issues of raising wages, the union during the
submitting its financial statements, lead to no other conclusion except that it is unwilling to meeting of February 19, 1991 declared a deadlock.
negotiate and reach an agreement with the Union.
On February 20, 1991, respondent union saturated petitioner’s premises with streamers and
General Milling Corp. vs. CA, G.R. No. 146728, February 11, 2004 picketed the hospital. The operations of the hospital having come to a grinding halt, the
hospital management considered the union actions as tantamount to a strike. On May 28,
FACTS: In its two plants located at Cebu City and Lapu-Lapu City, petitioner General Milling 1991, respondent union struck. Despite the NCMB’s call for a conciliation conference, nurses
Corporation (GMC) employed 190 workers. They were all members of private respondent and nurse aides who were members of the union abandoned their respective department and
General Milling Corporation Independent Labor Union. On April 28, 1989, GMC and the union joined the picket line a week later. Doctors began leaving the hospital and the number of
concluded a collective bargaining agreement (CBA) which included the issue of patients dwindled. The last patient was discharged on June 10, 1991.
representation effective for a term of three years. The day before the expiration of the CBA,
the union sent GMC a proposed CBA, with a request that a counter-proposal be submitted On June 12, 1991, a “Notice of Temporary Suspension of Operation” was issued by petitioner
within ten (10) days. However, GMC had received collective and individual letters from hospital and submitted to the local office of the NCMB on June 14, 1991. Then Secretary of
workers who stated that they had withdrawn from their union membership, on grounds of Labor Nieves Confessor assumed jurisdiction over the labor dispute and issued an order
religious affiliation and personal differences. Believing that the union no longer had standing directing all workers to return to work. However, this order was received by petitioner only on
to negotiate a CBA, GMC did not send any counter-proposal. June 20, 1991. In the meantime, it had already notified the DOLE via its letter dated June 13,
1991, which was received by the DOLE on June 14, 1991, that it would temporarily suspend
On December 16, 1991, GMC wrote a letter to the union’s officers, RitoMangubat and Victor operations for six (6) months effective June 15, 1991, or up to December 15, 1991. Petitioner
Lastimoso. The letter stated that it felt there was no basis to negotiate with a union which no thus refused the return of its striking workers on account of such suspension of operations.
longer existed, but that management was nonetheless always willing to dialogue with them
on matters of common concern and was open to suggestions on how the company may ISSUE: WON the Secretary can validly compel the employer to enter into a new CBA even
improve its operations. In answer, the union officers wrote a letter dated December 19, 1991 during temporary suspension of operations (what if in permanent closure?)
disclaiming any massive disaffiliation or resignation from the union and submitted a
manifesto, signed by its members, stating that they had not withdrawn from the union. HELD: Temporary suspension of operations is recognized as a valid exercise of management
prerogative provided it is not carried out in order to circumvent the provisions of the Labor
NLRC held that the action of GMC in not negotiating was ULP. Code or to defeat the rights of the employees under the Code. The determination to cease or
suspend operations is a prerogative of management that the State usually does not interfere
ISSUE: WON the company (GMC) should have entered into collective bargaining with the with, as no business can be required to continue operating at a loss simply to maintain the
union workers in employment. Such an act would be tantamount to a taking of property without due
process of law, which the employer has a right to resist. But where it is shown that the
HELD: The law mandates that the representation provision of a CBA should last for five closure is motivated not by a desire to prevent further losses, but to discourage the workers
years. The relation between labor and management should be undisturbed until the last 60 from organizing themselves into a union for more effective negotiation with management, the
days of the fifth year. Hence, it is indisputable that when the union requested for a State is bound to intervene.
renegotiation of the economic terms of the CBA on November 29, 1991, it was still the
certified collective bargaining agent of the workers, because it was seeking said renegotiation The burden of proving that such a temporary suspension is bona fide falls upon the employer.
within five (5) years from the date of effectivity of the CBA on December 1, 1988. The union’s In this instance, petitioner had to establish the fact of its precarious financial health, that its
proposal was also submitted within the prescribed 3-year period from the date of effectivity of cessation of operation was really necessitated by its financial condition, and that said
the CBA, albeit just before the last day of said period. It was obvious that GMC had no valid condition would probably be alleviated or improved, or its losses abated, by undertaking such
17

suspension of operation. The fact that the conciliator never asked for them is no sufficient The rule is that unless expressly assumed. labor contracts such as employment contracts
excuse for not presenting the same, as such was petitioner’s duty. Neither is it acceptable for and CBAs are not enforceable against a transferee of an enterprise, labor contracts being IN
petitioner to allege that latest financial statement (for the year 1991) were still being prepared PERSONAM, thus, binding only between the parties. A labor contract merely creates an
by its accountants and not yet ready for submission, since the financial statement for the prior action in personam and does not create an real right which should be respected by third
years 1989 and 1990 would have sufficed. parties. This conclusion draws its force from the right of an employer to select his employees
and to decide when to engage them as protected under our Constitution and the same can
It is a hornbook rule that employers who contemplate terminating the services of their only be restricted by law through the exercise of police power.
workers must base their decisions on more than just flimsy excuses, considering that the
dismissal of an employee from work involves not only the loss of his position but, what is As a general rule, there is no law requiring a bona fide purchaser of assets of an on-going
more important, his means of livelihood. The same principle applies in temporary suspension concern to absorb in its employ the employees of the latter. However, although the purchaser
of operations, as in this case, considering that it involves laying off employees for a period of is not legally bound to absorb in its employ the employees of the seller, the parties are liable
six months. Petitioner, having wretchedly failed to justify by even the most rudimentary proof to the employees if the transaction between is clothed with bad faith.
its temporary suspension of operations, must bear the consequences thereof. We thus hold
that the Secretary of Labor and Employment did not act with grave abuse of discretion in Rivera vs. Espiritu, G.R. No. 135547, January 23, 2002
finding the temporary suspension unjustified and illegal.
FACTS: PAL was suffering from a difficult financial situation in 1998. It was faced with
The order of the secretary in ordering the hospital to enter into a new CBA was valid. bankruptcy and was forced to adopt a rehabilitation plan and downsized its labor force by
more than 1/3. PALEA (PAL Employees Association) went on a four-day strike to protest
Secretary was of the impression that petitioner would operate again after the lapse of the six- retrenchment measures in July 1998. PAL ceased operations on Sep 23, 1998.
month suspension of operations on December 16, 1991, and so ordered the parties to enter
into and formalize a new CBA to govern their relations upon resumption of operations. On the PALEA board again wrote the President on Sep 28, 1998. Among others, it proposed the
other hand, the aforequoted portion of the Order must be understood in the context of the suspension of the PAL-PALEA CBA for a period of ten years, subject to certain conditions.
Secretary’s finding that the temporary suspension was only for circumventing the return-to- PALEA members accepted such terms through a referendum on Oct 2, 1998. PAL resumed
work order, but in spite of which he held that he could not order petitioner to continue domestic operations on Oct 7, 1998.
operations as “this would infringe on its inherent right to manage and conduct its own
business affairs”; he thus ordered instead the payment of backwages to the returning workers Seven officers and members of PALEA filed instant petition to annul the Sep 27, 1998
who were refused admittance by petitioner on June 21, 1991. And as above adverted to, he agreement entered into between PAL and PALEA.
also ordered the parties to execute a new CBA to govern their relations upon the expiry of the
period of suspension and the resumption of normal operations. Issue: WON negotiations may be suspended for 10 years.

Did the Secretary act in excess of jurisdiction in imposing the wage increase and union shop Held: YES. CBA negotiations may be suspended for 10 years.
provision on the petitioner? We hold that he did not. While petitioner cannot be forced to The assailed PAL-PALEA agreement was the result of voluntary collective bargaining
abandon its suspension of operations even if said suspension be declared unjustified, illegal negotiations undertaken in the light of the severe financial situation faced by the employer,
and invalid, neither can petitioner evade its obligation to bargain with the union, using the with the peculiar and unique intention of not merely promoting industrial peace at PAL, but
cessation of its business as reason therefor. For, as already indicated above, the employer- preventing the latter’s closure.
employee relationship was merely suspended (and not terminated) for the duration of the
temporary suspension. Using the suspension as an excuse to evade the duty to bargain is There is no conflict between said agreement and Article 253-A of the Labor Code. CBA
further proof of its illegality. It shows abuse of this option and bad faith on the part of under Article 253-A of the Labor Code has a two-fold purpose. One is to promote industrial
petitioner. And since it refused to bargain, without valid and sufficient cause, the Secretary in stability and predictability. Inasmuch as the agreement sought to promote industrial peace, at
the exercise of his powers under Article 263(i) of the Labor Code to decide and resolve labor the PAL during its rehabilitation, said agreement satisfied the first purpose of said article. The
disputes, properly granted the wage increase and imposed the union shop provision. other purpose is to assign specific timetable, wherein negotiations become a matter of right
and requirement. Nothing in Article 253-A prohibits the parties from waiving or suspending
Notwithstanding that respondent Secretary did not act with grave abuse of discretion in the mandatory timetable and agreeing on the remedies to enforce the same.
issuing the challenged Orders, we cannot ignore the supervening event which occurred after
December 15, 1991, i.e., the subsequent permanent cessation of petition of petitioner on SMC-Union vs. Confesor, G.R. No. 111262, September 19, 1996
account of losses. Thus, despite the absence of grave abuse of discretion on the part of the
respondent Secretary, this Court cannot impose upon petitioner the directive to enter into a FACTS: On June 28, 1990, petitioner-union San Miguel Corporation Employees Union —
new CBA with the union for the very simple reason that to do so would be to compel PTGWO entered into a CBA with private respondent San Miguel Corporation (SMC) to take
petitioner to continue its business when it had already decided to close shop, and that would effect upon the expiration of the previous CBA or on June 30, 1989.
be judicial tyranny on our part.
This CBA provided, among others, that:
Sundowner Development Corp. vs. Drilon, G.R. No. 82341, December 6, 1989
ARTICLE XIV
FACTS: Hotel Mabuhay leased the premises belonging to Syjuco. However, due to non-
payment of rentals, a case for ejectment was filed and Hotel Mabuhay offered to amicably DURATION OF AGREEMENT
settle by surrendering the premises and to sell its assets and property to any interested party,
to which Syjuco acceded. Sec. 1. This Agreement which shall be binding upon the parties hereto and their respective
successors-in-interest, shall become effective and shall remain in force and effect until June
Mabuhay offered to sell its assets and personal properties in the premises to petitioner to 30, 1992.
which petitioner agreed. A deed of assignment of said assets and personal properties was
executed by Mabuhay on April 29,1987 in favor of petitioner. Sec. 2. In accordance with Article 253-A of the Labor Code as amended, the term of this
Agreement insofar as the representation aspect is concerned, shall be for five (5) years from
On same date Syjuco formally turned over the possession of the leased premises to July 1, 1989 to June 30, 1994. Hence, the freedom period for purposes of such
petitioner who actually took possession and occupied the same on May 1, 1987. representation shall be sixty (60) days prior to June 30, 1994.

On May 4, 1987, respondent National Union of Workers in Hotel, Restaurant and Allied Sec. 3. Sixty (60) days prior to June 30, 1992 either party may initiate negotiations of all
Services (NUWHRAIN for short) picketed the leased premises, barricaded the entrance to the provisions of this Agreement, except insofar as the representation aspect is concerned. If no
leased premises and denied petitioner's officers, employees and guests free access to and agreement is reached in such negotiations, this Agreement shall nevertheless remain in force
egress from said premises. Respondent NUWHRAIN on July 13, 1987 filed its position paper up to the time a subsequent agreement is reached by the parties.
alleging connivance between Mabuhay and petitioner in selling the assets and closing the
hotel to escape its obligations to the employees of Mabuhay and so it prays that petitioner Meanwhile, effective October 1, 1991, Magnolia and Feeds and Livestock Division were
accept the workforce of Mabuhay and pay backwages from April 15,1986 to April 28,1987, spun-off and became two separate and distinct corporations: Magnolia Corporation
the day Mabuhay stopped operation. (Magnolia) and San Miguel Foods, Inc. (SMFI). Notwithstanding the spin-offs, the CBA
remained in force and effect.
ISSUE: Whether or not the purchaser of the assets of an employer corporation can be
considered a successor employer of the latter's employees. After June 30, 1992, the CBA was renegotiated in accordance with the terms of the CBA and
Article 253-A of the Labor Code. Negotiations started sometime in July, 1992 with the two
HELD: The absorption of the employees of Hotel Mabuhay may not be imposed on parties submitting their respective proposals and counterproposals.
Sundowner, who has no liability whatsoever to the employees of Hotel Mabuhay and its
responsibility if at all, is only to consider them for re-employment in the operation of the During the negotiations, the petitioner-union insisted that the bargaining unit of SMC should
business in the same premises. There can be no implied acceptance of the employees of still include the employees of the spun-off corporations: Magnolia and SMFI; and that the
Hotel Mabuhay by petitioner as it is expressly provided in the agreement that petitioner has renegotiated terms of the CBA shall be effective only for the remaining period of two years or
no commitment or duty to absorb them. until June 30, 1994.
18

SMC, on the other hand, contended that the members/employees who had moved to nature of work, wages, hours of work and other conditions of employment. Interests of
Magnolia and SMFI, automatically ceased to be part of the bargaining unit at the SMC. employees in the different companies perforce differ. The nature of their products and scales
Furthermore, the CBA should be effective for three years in accordance with Art. 253-A of the of business may require different skills which must necessarily be commensurated by
Labor Code. different compensation packages. The different companies may have different volumes of
work and different working conditions. For such reason, the employees of the different
Unable to agree on these issues with respect to the bargaining unit and duration of the CBA, companies see the need to group themselves together and organize themselves into
petitioner-union declared a deadlock on September 29, 1990. distinctive and different groups. It would then be best to have separate bargaining units for
the different companies where the employees can bargain separately according to their
(Notice of strike…Secretary assumed jurisdiction) needs and according to their own working conditions.

Secretary’s decision: the CBA shall be effective for the period of 3 years from June 30, 1992; LMG Chemicals Corp vs. Secretary of Labor, G.R. No. 127422, April 17, 2001
and that such CBA shall cover only the employees of SMC and not of Magnolia and SMFI.
FACTS: LMG Chemicals Corp, (petitioner) is a domestic corp engaged in the manufacture
ISSUES: 1) Whether or not the duration of the renegotiated terms of the CBA is to be and sale of various kinds of chemical substances, including aluminum sulfate which is
effective for three years of for only two years; and 2) Whether or not the bargaining unit of essential in purifying water, and technical grade sulfuric acid used in thermal power plants.
SMC includes also the employees of the Magnolia and SMFI. Petitioner has three divisions, namely: the Organic Division, Inorganic Division and the
Pinamucan Bulk Carriers. There are two unions within petitioner’s Inorganic Division. One
HELD: We agree with the Secretary of Labor. union represents the daily paid employees and the other union represents the monthly paid
employees. Chemical Workers Union, respondent, is a duly registered labor organization
Pertinent to the first issue is Art. 253-A of the Labor Code as amended which reads: acting as the collective bargaining agent of all the daily paid employees of petitioner’s
Inorganic Division.
Art. 253-A. Terms of a CBA. — Any CBA that the parties may enter into shall, insofar as the
representation aspect is concerned, be for a term of 5 years. No petition questioning the Sometime in December 1995, the petitioner and the respondent started negotiation for a new
majority status of the incumbent bargaining agent shall be entertained and no certification CBA as their old CBA was about to expire. They were able to agree on the political provisions
election shall be conducted by the Department of Labor and Employment outside of the sixty- of the new CBA, but no agreement was reached on the issue of wage increase. The
day period immediately before the date of expiry of such five year term of the CBA. All other economic issues were not also settled.
provisions of the CBA shall be renegotiated not later than 3 years after its execution. Any
agreement on such other provisions of the CBA entered into within 6 months from the date of With the CBA negotiations at a deadlock (Strike…Secretary assumed jurisdiction)
expiry of the term of such other provisions as fixed in such CBA, shall retroact to the day
immediately following such date. If any such agreement is entered into beyond six months, Secretary of Labor and Employment granted an increase of P140 (higher than the offer of
the parties shall agree on the duration of retroactivity thereof. In case of a deadlock in the petitioner-company of P135). Also, as to the effectivity of the new CBA…Sec held:
renegotiation of the CBA, the parties may exercise their rights under this Code. (Emphasis
supplied.) 3. Effectivity of the new CBA

The “representation aspect” refers to the identity and majority status of the union that Article 253-A of the Labor Code, as amended, provides that when no new CBA is signed
negotiated the CBA as the exclusive bargaining representative of the appropriate bargaining during a period of six months from the expiry date of the old CBA, the retroactivity period
unit concerned. “All other provisions” simply refers to the rest of the CBA, economic as well shall be according to the parties’ agreement, Inasmuch as the parties could not agree on this
as non-economic provisions, except representation. issue and since this Office has assumed jurisdiction, then this matter now lies at the
discretion of the Secretary of labor and Employment. Thus the new Collective Bargaining
The law is clear and definite on the duration of the CBA insofar as the representation aspect Agreement which the parties will sign pursuant to this Order shall retroact to January 1, 1996.
is concerned, but is quite ambiguous with the terms of the other provisions of the CBA. It is a
cardinal principle of statutory construction that the Court must ascertain the legislative intent petitioner contends that public respondent committed grave abuse of discretion when he
for the purpose of giving effect to any statute. ordered that the new CBA which the parties will sign shall retroact to January 1, 1996

(as usual mahabang conversation ng mga framers) ISSUE: Whether or not the new CBA shall retroact?

Obviously, the framers of the law wanted to maintain industrial peace and stability by having HELD: Petitioner insists that public respondent’s discretion on the issue of the date of the
both management and labor work harmoniously together without any disturbance. Thus, no effectivity of the new CBA is limited to either: (1) leaving the matter of the date of effectivity of
outside union can enter the establishment within 5 years and challenge the status of the the new CBA is limited to either: (1) leaving the matter of the date of effectivity of the new
incumbent union as the exclusive bargaining agent. Likewise, the terms and conditions of CBA to the agreement of the parties or (2) ordering that the terms of the new CBA be
employment (economic and non-economic) can not be questioned by the employers or prospectively applied.
employees during the period of effectivity of the CBA. The CBA is a contract between the
parties and the parties must respect the terms and conditions of the agreement. Notably, the It must be emphasized that respondent Secretary assumed jurisdiction over the dispute
framers of the law did not give a fixed term as to the effectivity of the terms and conditions of because it is impressed with national interest. As noted by the Secretary, “the petitioner corp
employment. It can be gleaned from their discussions that it was left to the parties to fix the was then supplying the sulfate requirements of MWSS as well as the sulfuric acid of
period. NAPOCOR, and consequently, the continuation of the strike would seriously affect the water
supply of Metro Manila and the power supply of the Luzon Grid.” Such authority of the
The issue as to the term of the non-representation provisions of the CBA need not Secretary to assume jurisdiction carries with it the power to determine the retroactivity of the
belaboured. The parties, by mutual agreement, enter into a renegotiated contract with a term parties’ CBA.
of three (3) years or one which does not coincide with the said 5-year term, and said
agreement is ratified by majority of the members in the bargaining unit, the subject contract is It is well settled in our jurisprudence that the authority of the Secretary of Labor to assume
valid and legal and therefore, binds the contracting parties. jurisdiction over a labor dispute causing or likely to cause a strike or lockout in an industry
indispensable to national interest includes and extends to all questions and controversies
Thus, we do not find any grave abuse of discretion on the part of the Secretary of Labor in arising therefrom. The power is plenary and discretionary in nature to enable him to
ruling that the effectivity of the renegotiated terms of the CBA shall be for 3 years. effectively and efficiently dispose of the primary dispute.

II. Undeniably, the transformation of the companies was a management prerogative and This Court held in St. Luke’s Medical Center, Inc. vs. Torres:
business judgment which the courts can not look into unless it is contrary to law, public policy
or morals. Neither can we impute any bad faith on the part of SMC so as to justify the Therefore in the absence of the specific provision of law prohibiting retroactivity of the
application of the doctrine of piercing the corporate veil.18 Ever mindful of the employees’ effectivity of the arbitral awards issued by the Secretary of Labor pursuant to Article 263(g) of
interests, management has assured the concerned employees that they will be absorbed by the Labor Code, such as herein involved, public respondent is deemed vested with plenary
the new corporations without loss of tenure and retaining their present pay and benefits powers to determine the effectivity thereof.”
according to the existing CBAs. 19 They were advised that upon the expiration of the CBAs,
new agreements will be negotiated between the management of the new corporations and MERALCO vs. Quisumbing, G.R. No. 127598, January 27, 1999
the bargaining representatives of the employees concerned.
FACTS: The court directed the parties to execute a CBA incorporating the terms among
Indubitably, therefore, Magnolia and SMFI became distinct entities with separate juridical which are the following modifications among others: Wages: PhP 1,900 for 1995-1996;
personalities. Thus, they can not belong to a single bargaining unit. Retroactivity: December 28, 1996-Dec. 1999, etc. Dissatisfied, some members of the union
filed a motion for intervention/reconsideration. Petitioner warns that is the wage increase of
Moreover, in determining an appropriate bargaining unit, the test of grouping is mutuality or Php2,000.00 per month as ordered is allowed, it would pass the cost covering such increase
commonality of interests. The employees sought to be represented by the collective to the consumers through an increase rate of electricity. On the retroactivity of the CBA
bargaining agent must have substantial mutual interests in terms of employment and working arbitral award, the parties reckon the period as when retroaction shall commence.
conditions as evinced by the type of work they performed. 22 Considering the spin-offs, the
companies would consequently have their respective and distinctive concerns in terms of the
19

ISSUE: Whether or not retroactivity of arbitral awards shall commence at such time as
granted by Secretary. Later on, the UNION filed a notice of strike against BENGUET. UNION members who were
BENGUET employees in the mining camps at Acupan, Antamok and Balatoc, went on strike.
RULING: In St. Luke’s Medical vs Torres, a deadlock developed during CBA negotiations The strike was attended by violence, some of the workers and executives of the BENGUET
between management unions. The Secretary assumed jurisdiction and ordered the were prevented from entering the premises and some of the properties of the BENGUET
retroaction of the CBA to the date of expiration of the previous CBS. The Court ratiocinated were damaged as a result of the strike. Eventually, the parties agreed to end the dispute.
thus: In the absence of a specific provision of law prohibiting retroactive of the effectivity of BENGUET and UNION executed the AGREEMENT. PAFLU placed its conformity thereto.
arbitral awards issued by the Secretary pursuant to article 263(g) of the Labor Code, public About a year later or on January 29, 1964, a collective bargaining contract was finally
respondent is deemed vested with the plenary and discretionary powers to determine the executed between UNION-PAFLU and BENGUET.
effectivity thereof.
Meanwhile, BENGUET sued UNION, PAFLU and their Presidents to recover the amount the
In general, a CBA negotiated within six months after the expiration of the existing CBA former incurred for the repair of the damaged properties resulting from the strike. BENGUET
retroacts to the day immediately following such date and if agreed thereafter, the effectivity also argued that the UNION violated the CONTRACT which has a stipulation not to strike
depends on the agreement of the parties. On the other hand, the law is silent as to the during the effectivity thereof.
retroactivity of a CBA arbitral award or that granted not by virtue of the mutual agreement of
the parties but by intervention of the government. In the absence of a CBA, the Secretary’s Defendants unions and their presidents defended that: (1) they were not bound by the
determination of the date of retroactivity as part of his discretionary powers over arbitral CONTRACT which BBWU, the defeated union, had executed with BENGUET; (2) the strike
awards shall control. was due, among others, to unfair labor practices of BENGUET; and (3) the strike was lawful
and in the exercise of the legitimate rights of UNION-PAFLU under Republic Act 875.
Wherefore, the arbitral award shall retroact from December 1, 1995 to November 30, 1997;
and the award of wage is increased from Php1,900 to Php2,000. The trial court dismissed the complaint on the ground that the CONTRACT, particularly the
No-Strike clause, did not bind defendants. BENGUET interposed the present appeal.
FVC-Labor Union-PTGWO vs. SANAMA-FVC-SIGLO, G.R. No. 176249, Novber 27, 2009
ISSUE: Did the Collective Bargaining Contract executed between Benguet and BBWU on
Facts:On December 22, 1997, the petitioner FVCLU-PTGWO –the recognized bargaining June 23, 1959 and effective until December 23, 1963 automatically bind UNION-PAFLU upon
agent of the rank-and-fileemployees of the FVC Philippines, Incorporated – signed a its certification, on August 18, 1962, as sole bargaining representative of all BENGUET
five-year collective bargaining agreement with the company. The five-year CBA period employees
was from February 1, 1998 to January 30, 2003. At the end of the 3rd year of the five-year
term and pursuant to the CBA, FVCLU-PTGWO and the company entered into the RULING: NO. BENGUET erroneously invokes the so-called “Doctrine of Substitution”
renegotiation of the CBA and modified, among other provisions, the CBA’s duration. Article referred to in General Maritime Stevedore’s Union v. South Sea Shipping Lines where it was
XXV, Section 2 of the renegotiated CBA provides that “this re-negotiation ruled that:
agreement shall take effect beginning February 1, 2001 and until May 31, 2003” thus
extending the original five-year period of the CBA by four (4) months. On January 21, 2003, “We also hold that where the bargaining contract is to run for more than two years, the
nine (9) days before the January 30, 2003 expiration of the originally-agreed five-year principle of substitution may well be adopted and enforced by the CIR to the effect that after
CBA term (and four [4] months and nine [9] days away from theexpiration of the amended two years of the life of a bargaining agreement, a certification election may be allowed by the
CBA period), the respondent Sama-Samang Nagkakaisang Manggagawasa FVC-Solidarity CIR, that if a bargaining agent other than the union or organization that executed the
of Independent and General Labor Organizations (SANAMA-SIGLO) filed before contract, is elected, said new agent would have to respect said contract, but that it may
the Department of Labor and Employment (DOLE) a petition for certification election for the bargain with the management for the shortening of the life of the contract if it considers it too
same rank-and-file unit covered by the FVCLU-PTGWO CBA. FVCLU-PTGWO moved long, or refuse to renew the contract pursuant to an automatic renewal clause.”
to dismiss the petition on the ground that the certification election petition was filed
outside the freedom period or outside of the sixty (60) days before the expiration of the CBA BENGUET’s reliance upon the Principle of Substitution is totally misplaced. This principle,
on May 31, 2003. formulated by the NLRB as its initial compromise solution to the problem facing it when there
occurs a shift in employees’ union allegiance after the execution of a bargaining contract with
Issue:Was the certification election filed within the freedom period? their employer, merely states that even during the effectivity of a collective bargaining
agreement executed between employer and employees thru their agent, the employees can
Ruling:Yes. While the parties may agree to extend the CBA’s original five-year term change said agent but the contract continues to bind them up to its expiration date. They may
together with all other CBA provisions, any such amendment or term in excess of five bargain however for the shortening of said expiration date.
years will not carry with it a change in the union’s exclusive collective In formulating the “substitutionary” doctrine, the only consideration involved was the
bargaining status. By express provision of Article 253-A, the exclusive bargaining status employees‘ (principal) interest in the existing bargaining agreement. The agent’s (union)
cannot go beyond five years and the representation status is a legal matter not for interest never entered the picture. The majority of the employees, as an entity under the
the workplace parties to agree upon. In other words, despite an agreement for a CBA with a statute, is the true party in interest to the contract, holding rights through the agency of the
life of more than five years, either as an original provision or by amendment, the union representative. Thus, any exclusive interest claimed by the agent is defeasible at the
bargaining union’s exclusive bargaining status is effective only for five years and can be will of the principal. The “substitutionary” doctrine only provides that the employees cannot
challenged within sixty (60) days prior to the expiration of the CBA’s first five years. In the revoke the validly executed collective bargaining contract with their employer by the simple
present case, the CBA was originally signed for a period of five years, i.e., from expedient of changing their bargaining agent. And it is in the light of this that the phrase “said
February 1, 1998 to January 30, 2003, with a provision for the renegotiation of the CBA’s new agent would have to respect said contract” must be understood. It only means that the
other provisions at the end of the 3rd year of the five-year CBA term. Thus, prior to January employees, thru their new bargaining agent, cannot renege on their collective bargaining
30, 2001 the workplace parties sat down for renegotiation but instead of confining contract, except of course to negotiate with management for the shortening thereof.
themselves to the economic and non-economic CBA provisions, also extended the life of the
CBA for another four months, i.e., from the original expiry date on January 30, 2003 The “substitutionary” doctrine cannot be invoked to support the contention that a newly
to May 30, 2003. This negotiated extension of the CBA term has no legal effect on the certified collective bargaining agent automatically assumes all the personal undertakings —
FVCLU-PTGWO’s exclusive bargaining representation status which remained effective like the no-strike stipulation here — in the collective bargaining agreement made by the
only for five years ending on the original expiry date of January 30, 2003. Thus, sixty days deposed union. When BBWU bound itself and its officers not to strike, it could not have
prior to this date, or starting December 2, 2002, SANAMA-SIGLO could properly file a petition validly bound also all the other rival unions existing in the bargaining units in question. BBWU
for certification election. Its petition, filed on January 21, 2003 or nine (9) days before the was the agent of the employees, not of the other unions which possess distinct personalities.
expiration of the CBA and of FVCLU-PTGWO’s exclusive bargaining status, was seasonably
filed. UNION, as the newly certified bargaining agent, could always voluntarily assume all the
personal undertakings made by the displaced agent. But as the lower court found, there was
Benguet Consolidated, Inc., vs. BCI Employees and Workers’ Union, G.R. No. L-24711, April no showing at all that, prior to the strike, UNION formally adopted the existing CONTRACT as
30, 1968 its own and assumed all the liabilities imposed by the same upon BBWU. Defendants were
FACTS: On June 23, 1959, the Benguet-Balatoc Workers Union (“BBWU”), for and in behalf neither signatories nor participants in the CONTRACT.
of all Benguet Consolidated, Inc (BENGUET) employees in its mines and milling
establishment located at Balatoc, Antamok and Acupan, Mt. Province, entered into a Everything binding on a duly authorized agent, acting as such, is binding on the principal; not
Collective Bargaining Contract (CONTRACT) with BENGUET. The CONTRACT was vice-versa, unless there is mutual agency, or unless the agent expressly binds himself to the
stipulated to be effective for a period of 4-1/2 years, or from June 23, 1959 to December 23, party with whom he contracts. Here, it was the previous agent who expressly bound itself to
1963. It likewise embodied a No-Strike, No-Lockout clause. the other party, BENGUET. UNION, the new agent, did not assume this undertaking of
BBWU.
3 years later, or on April 6, 1962, a certification election was conducted by the Department of
Labor among all the rank and file employees of BENGUET in the same collective bargaining Since defendants were not contractually bound by the no-strike clause in the CONTRACT, for
units. BCI EMPLOYEES & WORKERS UNION (UNION) obtained more than 50% of the total the simple reason that they were not parties thereto, they could not be liable for breach of
number of votes, defeating BBWU. The Court of Industrial Relations certified the UNION as contract to plaintiff.
the sole and exclusive collective bargaining agent of all BENGUET employees as regards
rates of pay, wages, hours of work and such other terms and conditions of employment Capitol Medical Center Alliance of Concerned Employees vs. Laguesma, G.R. No. 118015,
allowed them by law or contract. Feb 4, 1997
20

FACTS: Respondent union filed petition for certification election. The Med-Arbiter granted the It is only just and equitable that the circumstances in this case should be considered as
petition for certification election. Respondent Capitol Medical Center (CMC) appealed to the similar in nature to a “bargaining deadlock” when no certification election could be held. This
Office of the Secretary. But the Order granting the certification election was affirmned. is also to make sure that no floodgates will be opened for the circumvention of the law by
unscrupulous employers to prevent any certified bargaining agent from negotiating a CBA.
On December 9, 1992, elections were held with respondent union garnering 204 votes, 168 Thus, Section 3, Rule V, Book V of the Implement Rules should be interpreted liberally so as
in favor of no union and 8 spoiled ballots out of a total of 380 votes cast. Med-Arbiter issued to include a circumstance, e.g. where a CBA could not be concluded due to the failure of one
an Order certifying respondent union as the sole and exclusive bargaining representative of party to willingly perform its duty to bargain collectively.
the rank and file employees at CMC.
Caltex Refinery Employees Assoc. vs. Brillantes, G.R. No. 123782, September 17, 1997
Respondent CMC again appealed to the Office of the Secretary of Labor the result of the FACTS: Anticipating the expiration of their CBA on July 31, 1995, petitioner and private
election, it was denied. MR also denied. Respondent CMC’s contention was the supposed respondent negotiated the terms and conditions of employment to be contained in a new
pendency of its petition for cancellation of respondent union’s certificate of registration. In the CBA. The negotiation between the two parties was participated in by the NCMB and the
said case, the Med-Arbiter therein issued an Order which declared respondent union’s Office of the Secretary of Labor and Employment. Some items in the new CBA were amicably
certificate of registration as null and void. However, this order was reversed on appeal by the arrived at and agreed upon, but others were unresolved.
Officer-in-Charge of the BLR in her Order. The said Order dismissed CMC’s motion for
cancellation of the certificate of registration of respondent union and declared that it was not To settle the unresolved issues, eight meetings between the parties were conducted.
only a bona fide affiliate or local of a federation, but a duly registered union as well. Because the parties failed to reach any significant progress in these meetings, petitioner
declared a deadlock. On July 24, 1995, petitioner filed a notice of strike. 6 conciliation
Respondent union, after being declared as the certified bargaining agent of the rank-and-file meetings conducted by the NCMB failed, failed. Marathon meetings at the plant level, but this
employees of respondent CMC, presented proposals for the negotiation of a CBA. However, remedy proved also unavailing.
CMC contended that CBA negotiations should be suspended in view of the Order declaring
the registration of respondent union as null and void. In spite of the refusal of respondent Secretary assumed jurisdiction and ordered “Accordingly, any strike or lockout, whether
CMC, respondent union still persisted in its demand for CBA negotiations, claiming that it has actual or intended, is hereby enjoined.”xxx But the members of petitioner defied them and
already been declared as the sole and exclusive bargaining agent of the rank-and-file continued their mass action (despite repeated orders)
employees of the hospital.
Thereafter, the contending parties filed their position papers pertaining to unresolved issues.
Due to respondent CMC’s refusal to bargain collectively, respondent union filed a notice of Because of the strike, private respondent terminated the employment of some officers of
strike and later staged a strike on April 15, 1993. The case was certified to the NLRC for petitioner union. The legality of these dismissals brought additional contentious issues.
compulsory arbitration.
Again, the parties tried to resolve their differences through conciliation. Failing to come to any
It is at this point that petitioner union, on March 24, 1994, filed a petition for certification substantial agreement, the parties decided to refer the problem to the secretary of labor and
election among the regular rank-and-file employees of the Capitol Medical Center Inc. It employment.
alleged in its petition that: 1) three hundred thirty one (331) out of the four hundred (400) total
rank-and-file employees of respondent CMC signed a petition to conduct a certification (guys, what the SC did in this case, is to decide upon the conflicting issues ng parties with
election; and 2) that the said employees are withdrawing their authorization for the said union regard dun sa CBA kaya nilagay ko na lang lahat)
to represent them as they have joined and formed the union Capitol Medical Center Alliance
of Concerned Employees (CMC-ACE). They also alleged that a certification election can now ISSUE: (1) petitioner questions public respondent’s resolution of five issues in the CBA,
be conducted as more that 12 months have lapsed since the last certification election was specifically on wage increase, union security clause, retirement benefits or application of the
held. new retirement plan, signing bonus and grievance and arbitration machineries; and
(2) Whether or not the Honorable Secretary of Labor and Employment committed grave
Respondent union opposed the petition and moved for its dismissal. It contended that it is the abuse of discretion in resolving the instant labor dispute.
certified bargaining agent of the rank-and-file employees of the Hospital, which was
confirmed by the Secretary of DOLE and by this Court. It also alleged that it was not HELD: The petition is partly meritorious.
negligent in asserting its right as the certified bargaining agent for it continuously demanded
the negotiation of a CBA with the hospital despite the latter’s avoidance to bargain (1) 1. Wage Increase. Petitioner maintains that the salaries of Shell Refinery employees be
collectively. used as a “reference point” in upgrading the compensation of private respondent’s
employees because these two companies are in the “same industry and their refineries are
May 12, 1994, Med-Arbiter Brigida Fadrigon, issued an Order granting the petition for both in Batangas.” Thus, the wage increase of petitioner’s members should be
certification election among the rank and file employees. On appeal by respondent union, the “15%/15%/15%.” Private respondent counters with a “proposed 9% 7% 7% increase for the
public respondent Laguesma reversed and favored the respondent union. Hence this petition. same period with automatic adjustment should the increase fall short of the inflation rate.

ISSUE: Was there a bargaining deadlock between CMC and respondent union. The alleged “similarity” in the situation of Caltex and Shell cannot be considered a valid
ground for a demand of wage increase, in the absence of a showing that the two companies
RULING: While it is true that one year had lapsed since the time of declaration of a final are also similar in “substantial aspects,” as discussed above.
certification result, and that there is no collective bargaining deadlock, public respondent did
not commit grave abuse of discretion when it ruled in respondent union’s favor since the True, union members have the right to demand wage increases through their collective force;
delay in the forging of the CBA could not be attributed to the fault of the latter. but it is equally cogent that they should also be able to justify an appreciable increase in
wages. We observe that private respondent’s detailed allegations on productivity are
After respondent union was certified as the bargaining agent of CMC, it invited the employer unrebutted. It is noteworthy that petitioner ignored this argument of private respondent and
hospital to the bargaining table by submitting its economic proposal for a CBA. However, based its demand for wage increase not on the ground that they were as productive as the
CMC refused to negotiate with respondent union and instead challenged the latter’s legal Shell employees. Thus, we cannot attribute grave abuse of discretion to public respondent.
personality through a petition for cancellation of the certificate of registration which eventually
reached this Court. The decision affirming the legal status of respondent union should have 2. Union Security Clause. Petitioner argues that in spite of the provisions on the “union
left CMC with no other recourse but to bargain collectively; but still it did not. Respondent security clause,” it may expel a member only on any of three grounds: non-payment of dues,
union was left with no other recourse but to file notice of strike against CMC for unfair labor subversion, or conviction for a crime involving moral turpitude. If the employee’s act does not
practice with the NCMB. This eventually led to a strike. constitute any of these three grounds, the member would continue to be employed by private
respondent. Thus, the disagreement between petitioner and private respondent on this issue
A “deadlock” is the counteraction of things producing entire stoppage; There is a deadlock is not only “procedural” but also “substantial.”
when there is a complete blocking or stoppage resulting from the action of equal and
opposed forces. The word is synonymous with the word impasse, which presupposes We agree with petitioner. The disagreement between petitioner and private respondent on
reasonable effort at good faith bargaining which, despite noble intentions, does not conclude the union security clause should have been definitively resolved by public respondent. The
in agreement between the parties. labor secretary should take cognizance of an issue which is not merely incidental to but
essentially involved in the labor dispute itself, or which is otherwise submitted to him for
Although there is no “deadlock” in its strict sense as there is no “counteraction” of forces resolution. The secretary of labor assumed jurisdiction over this labor dispute in an industry
present in this case nor “reasonable effort at good faith bargaining,” such can be attributed to indispensable to national interest, precisely to settle once and for all the disputes over which
CMC’s fault as the bargaining proposals of respondent union were never answered by CMC. he has jurisdiction at his level. In not performing his duty, the secretary of labor committed a
In fact, what happened in this case is worse than a bargaining deadlock for CMC employed grave abuse of discretion.
all legal means to block the certification of respondent union as the bargaining agent of the
rank-and-file; and use it as its leverage for its failure to bargain with respondent union. We 3. New Retirement Plan. Petitioner contends that “40 of its members who are still covered by
can only conclude that CMC was unwilling to negotiate and reach an agreement with the Old Retirement Plan because they were not able to exercise the option to shift to the New
respondent union. CMC has not at any instance shown willingness to discuss the economic Retirement Plan, for one reason or another, when such option was given in the past” are
proposals given by respondent union. included in the New Retirement Plan.
21

We hold that public respondent did not commit grave abuse of discretion in respecting the
free and voluntary decision of the employees in regard to the Provident Plan and the On petition for certiorari, the Court of Appeals rendered a Decision reinstating the LA’s
irrevocable one-time option provided for in the New Retirement Plan. Although the union has Decision by reason of technicality. This prompted UE to file the present petition.
every right to represent its members in the negotiation regarding the terms and conditions of
their employment, it cannot negate their wishes on matters which are purely personal and ISSUE: Whether or not Bueno and Pepanio were validly terminated from their employment?
individual to them. In this case, the forty employees freely opted to be covered by the Old
Plan; their decision should be respected. The company gave them every opportunity to HELD: Petition is granted. LABOR LAW: collective bargaining agreement
choose, and they voluntarily exercised their choice. The union cannot pretend to know better;
it cannot impose its will on them. The policy requiring postgraduate degrees of college teachers was provided in the Manual of
Regulations as early as 1992. Indeed, recognizing this, the 1994 CBA provided even then
4. Grievance Machinery and Arbitration. Petitioner contends that public respondent “derailed that UE was to extend only semester- to-semester appointments to college faculty staffs, like
the grievance and arbitration scheme proposed by the Union.” Petitioner’s recommendation Bueno and Pepanio, who did not possess the minimum qualifications for their positions.
for a “single arbitrator is based on the proposition that if voluntary arbitration should be
resorted to at all, this recourse should entail the least possible expense.” Besides, as the Court held in Escorpizo v. University of Baguio, a school CBA must be read
in conjunction with statutory and administrative regulations governing faculty qualifications.
No particular setup for a grievance machinery is mandated by law. Rather, Article 260 of the Such regulations form part of a valid CBA without need for the parties to make express
Labor Code, as incorporated by RA 6715, provides for only a single grievance machinery in reference to it. While the contracting parties may establish such stipulations, clauses, terms
the company to settle problems arising from “interpretation or implementation of their CBA and conditions, as they may see fit, the right to contract is still subject to the limitation that the
and those arising from the interpretation or enforcement of company personnel policies.” agreement must not be contrary to law or public policy.

We believe that the procedure described by public respondent sufficiently complies with the Here, UE gave Bueno and Pepanio more than ample opportunities to acquire the
minimum requirement of the law. Public respondent even provided for two steps in hearing postgraduate degree required of them. But they did not take advantage of such opportunities.
grievances prior to their referral to arbitration. The parties will decide on the number of Justice, fairness, and due process demand that an employer should not be penalized for
arbitrators who may hear a dispute only when the need for it arises. Even the law itself does situations where it had little or no participation or control.
not specify the number of arbitrators. . In this matter, cost is not the only consideration; full
deliberation on the issues is another, and it is best accomplished in a hearing conducted by Insular Life Assurance Employees Asso. v. Insular Life, G. R. No. L-25291, Jan 30, 1971
three arbitrators. In effect, the parties are afforded the latitude to decide for themselves the FACTS: The Insular Life Assurance Co., Ltd., Employees Association-NATU, FGU Insurance
composition of the grievance machinery as they find appropriate to a particular situation. At Group Workers & Employees Association-NATU, and Insular Life Building Employees
bottom, we cannot really impute grave abuse of discretion to public respondent on this issue. Association-NATU (hereinafter referred to as the Unions), while still members of the
Federation of Free Workers (FFW), entered into separate CBAs with the Insular Life
5. Signing Bonus. Petitioner asseverates that the “signing bonus is an existing benefit Assurance Co., Ltd. and the FGU Insurance Group (hereinafter referred to as the
embodied in the old CBA.” 42 It explains that public respondent erred in removing the award Companies).
of a signing bonus xxx
Two of the lawyers of the Unions then were Felipe Enaje and Ramon Garcia; the latter was
Although proposed by petitioner, 45 the signing bonus was not accepted by private formerly the secretary-treasurer of the FFW and acting president of the Insular Life/FGU
respondent. 46 Besides, a signing bonus is not a benefit which may be demanded under the unions and the Insular Life Building Employees Association. Garcia, as such acting president,
law. Rather, it is now claimed by petitioner under the principle of “maintenance of existing in a circular issued in his name and signed by him, tried to dissuade the members of the
benefits” of the old CBA. However, as clearly explained by private respondent, a signing Unions from disaffiliating with the FFW and joining the National Association of Trade Unions
bonus may not be demanded as a matter of right. If it is not agreed upon by the parties or (NATU), to no avail.
unilaterally offered as an additional incentive by private respondent, the condition for
awarding it must be duly satisfied. In the present case, the condition sine qua non for its grant Enaje and Garcia soon left the FFW and secured employment with the Anti-Dummy Board of
— a non-strike — was not complied with. In fact, private respondent categorically sated in its the Department of Justice. Thereafter, the Companies hired Garcia in the latter part of 1956
counter-proposal — to the exclusion of those agreed upon before — that the new CBA would as assistant corporate secretary and legal assistant in their Legal Department. Enaje was
constitute the only agreement between the parties. hired as personnel manager of the Companies, and was likewise made chairman of the
negotiating panel for the Companies in the collective bargaining with the Unions.
(2). In the present case, the foregoing requirement has been sufficiently met. Petitioner’s
claim of grave abuse of discretion is anchored on the simple fact that public respondent Unions jointly submitted proposals to the Companies; negotiations were conducted on the
adopted largely the proposals of private respondent. It should be understood that bargaining Union’s proposals, but these were snagged by a deadlock on the issue of union shop, as a
is not equivalent to an adversarial litigation where rights and obligations are delineated and result of which the Unions filed on January 27, 1958 a notice of strike for “deadlock on
remedies applied. It is simply a process of finding a reasonable solution to a conflict and collective bargaining.” The issue was dropped subsequently (in short, nagkasundo). But, the
harmonizing opposite positions into a fair and reasonable compromise. When parties agree parties negotiated on the labor demands but with no satisfactory result due to a stalemate on
to submit unresolved issues to the secretary of labor for his resolution, they should not expect the matter of salary increases.
their positions to be adopted in toto. It is understood that they defer to his wisdom and
objectivity in insuring industrial peace. And unless they can clearly demonstrate bias, Meanwhile, 87 unionists were reclassified as supervisors without increase in salary nor in
arbitrariness, capriciousness or personal hostility on the part of such public officer, the Court responsibility while negotiations were going on in the Department of Labor after the notice to
will not interfere or substitute the said officer’s judgment with its own. strike was served on the Companies. These employees resigned from the Unions.

University of the East vs. Pepanio, G.R. No. 193897, January 13, 2013 On May 21, 1958 the Companies through their acting manager and president, sent to each of
FACTS: In 1992, DECS issued the Revised Manual of Regulations for Private Schools, which the strikers a letter (exhibit A) quoted verbatim as follows:
requires college faculty members to have a master's degree as a minimum educational
qualification for acquiring regular status. We recognize it is your privilege both to strike and to conduct picketing.

University of the East hired respondent Mariti D. Bueno (Bueno) in 1997 and respondent However, if any of you would like to come back to work voluntarily, you may:
Analiza F. Pepanio (Pepanio) in 2000, both on a semester-to-semester basis to teach in its
college. During this time, the 1994 CBA was still in force. It provided that UE shall extend only Advise the nearest police officer or security guard of your intention to do so.
semester-to-semester appointments to college faculty staffs who did not possess the Take your meals within the office.
minimum qualifications. Meantime, DECS-CHED-TESDA-DOLE Joint Order 1 was issued Make a choice whether to go home at the end of the day or to sleep nights at the
which provides that “teaching or academic personnel who do not meet the minimum office where comfortable cots have been prepared.
academic qualifications shall not acquire tenure or regular status.” Enjoy free coffee and occasional movies.
Be paid overtime for work performed in excess of eight hours.
Then in 2001, UE and the faculty union entered into a new CBA that would have the school Be sure arrangements will be made for your families.
extend probationary full-time appointments to full-time faculty members who did not yet have The decision to make is yours — whether you still believe in the motives of the
the required postgraduate degrees provided that the latter would obtain such requirement strike or in the fairness of the Management.
during their probationary period. Hence, UE extended probationary appointments to Bueno
and Pepanio. The two, however, failed to obtain post-graduate degrees.
Unions, however, continued on strike, with the exception of a few unionists who were
convinced to desist by the aforesaid letter
UE informed Bueno and Pepanio that their probationary status is about to expire since they
lack the required post-graduate qualification. However, Bueno and Pepanio demanded that
From the date the strike was called on May 21, 1958, until it was called off on May 31, 1958,
they should be considered as regular employees since they were hired in 1997 and 2000,
some management men tried to break thru the Unions’ picket lines xxx succeeded in
when what was in force was the 1994 CBA which did not require a master’s degree before
penetrating the picket lines in front of the Insular Life Building, thus causing injuries to the
attaining regular status. UE did not heed to their demands.
picketers and also to the strike-breakers due to the resistance offered by some picketers.
Thus, they filed a case for illegal dismissal before the Labor Arbiter. The LA ruled in their
favor. Dissatisfied, UE appealed to the NLRC. The NLRC reversed the LA’s ruling.
22

Alleging that some non-strikers were injured and with the use of photographs as evidence, by a union, since the employees thus offered reinstatement are unable to determine what the
the Companies then filed criminal charges against the strikers with the City Fiscal’s Office of consequences of returning to work would be.
Manila.xxx
ULP also: (super short cut na to) Hiring of Enage and Garcia with attractive compensations;
Another letter was sent by the company to the individual strikers: respondents reclassified 87 employees as supervisors without increase in salary or in
responsibility, in effect compelling these employees to resign from their unions; respondents,
The first day of the strike was last 21 May 1958. thru their president and manager, respondent Jose M. Olbes, brought three truckloads of
non-strikers and others, escorted by armed men, who, despite the presence of eight
Our position remains unchanged and the strike has made us even more entrances to the three buildings occupied by the Companies, entered thru only one gate less
convinced of our decision. than two meters wide and in the process, crashed thru the picket line posted in front of the
premises of the Insular Life Building. This resulted in injuries on the part of the picketers and
We do not know how long you intend to stay out, but we cannot hold your the strike-breakers; respondents brought against the picketers criminal charges, only three of
positions open for long. We have continued to operate and will continue to do so which were not dismissed, and these three only for slight misdemeanors. As a result of these
with or without you. criminal actions, the respondents were able to obtain an injunction from the court of first
instance restraining the strikers from stopping, impeding, obstructing, etc. the free and
If you are still interested in continuing in the employ of the Group Companies, peaceful use of the Companies’ gates, entrance and driveway and the free movement of
and if there are no criminal charges pending against you, we are giving you until persons and vehicles to and from, out and in, of the Companies’ buildings.
2 June 1958 to report for work at the home office. If by this date you have not yet
reported, we may be forced to obtain your replacement. Verily, the above actuations of the respondents before and after the issuance of the letters,
exhibit A and B, yield the clear inference that the said letters formed of the respondents
Before, the decisions was yours to make. scheme to preclude if not destroy unionism within them.

So it is now. II. The respondents did not merely discriminate against all the strikers in general. They
separated the active from the less active unionists on the basis of their militancy, or lack of it,
Incidentally, all of the more than 120 criminal charges filed against the members of the on the picket lines. Unionists belonging to the first category were refused readmission even
Unions, except 3, were dismissed by the fiscal’s office and by the courts. These three cases after they were able to secure clearances from the competent authorities with respect to the
involved “slight physical injuries” against one striker and “light coercion” against two others. criminal charges filed against them.

At any rate, because of the issuance of the writ of preliminary injunction against them as well It is noteworthy that — perhaps in an anticipatory effort to exculpate themselves from charges
as the ultimatum of the Companies giving them until June 2, 1958 to return to their jobs or of discrimination in the readmission of strikers returning to work — the respondents delegated
else be replaced, the striking employees decided to call off their strike and to report back to the power to readmit to a committee.
work on June 2, 1958.
III. Anent the third assignment of error, the record shows that not a single dismissed striker
*However, before readmitting the strikers, the Companies required them not only to secure was given the opportunity to defend himself against the supposed charges against him. As
clearances from the City Fiscal’s Office of Manila but also to be screened by a management earlier mentioned, when the striking employees reported back for work on June 2, 1958, the
committee among the members of which were Enage and Garcia. The screening committee respondents refused to readmit them unless they first secured the necessary clearances; but
initially rejected 83 strikers with pending criminal charges. However, all non-strikers with when all, except three, were able to secure and subsequently present the required
pending criminal charges which arose from the breakthrough incident were readmitted clearances, the respondents still refused to take them back.
immediately by the Companies without being required to secure clearances from the fiscal’s
office. Subsequently, when practically all the strikers had secured clearances from the fiscal’s Indeed, the individual cases of dismissed officers and members of the striking unions do not
office, the Companies readmitted only some but adamantly refused readmission to 34 indicate sufficient basis for dismissal.
officials and members of the Unions who were most active in the strike, on the ground that
they committed “acts inimical to the interest of the respondents,” without however stating the Standard Chartered Bank Employees Union vs. Confesor, GR No. 114974, Jun 16, 2004
specific acts allegedly committed. Some 24 of the above number were ultimately notified FACTS: Before the commencement of the negotiation for the new CBA between the bank
months later that they were being dismissed retroactively as of June 2, 1958 and given and the Union, the Union, through Divinagracia, suggested to the Bank’s Human Resource
separation pay checks computed under Rep. Act 1787, while others (ten in number) up to Manager and head of the negotiating panel, Cielito Diokno, that the bank lawyers should be
now have not been readmitted although there have been no formal dismissal notices given to excluded from the negotiating team. The Bank acceded. Meanwhile, Diokno(head of the
them. negotiating team for the bank) suggested to Divinagracia that Jose P. Umali, Jr., the
President of the National Union of Bank Employees (NUBE), the federation to which the
CIR prosecutor filed a complaint for unfair labor practice against the Companies under Union was affiliated, be excluded from the Union’s negotiating panel. However, Umali was
Republic Act 875. The complaint specifically charged the Companies with (1) interfering with retained as a member thereof.
the members of the Unions in the exercise of their right to concerted action, by sending out
individual letters to them urging them to abandon their strike and return to work, with a There was deadlock in the negotiations. Both parties alleged ULP. Bank alleged that the
promise of comfortable cots, free coffee and movies, and paid overtime, and, subsequently, Union violated its no strike- no lockout clause by filing a notice of strike before the NCMB.
by warning them that if they did not return to work on or before June 2, 1958, they might be Considering that the filing of notice of strike was an illegal act, the Union officers should be
replaced; and (2) discriminating against the members of the Unions as regards readmission dismissed. Union alleged unfair labor practice when the bank allegedly interfered with the
to work after the strike on the basis of their union membership and degree of participation in Union’s choice of negotiator. It argued that, Diokno’s suggestion that the negotiation be
the strike. limited as a “family affair” was tantamount to suggesting that Federation President Jose
Umali, Jr. be excluded from the Union’s negotiating panel. It further argued that, damage or
ISSUE: Whether or not respondent company is guilty of ULP injury to the public interest need not be present in order for unfair labor practice to prosper.
The Union also contended that the Bank merely went through the motions of collective
HELD: YES. The act of an employer in notifying absent employees individually during a strike bargaining without the intent to reach an agreement
following unproductive efforts at collective bargaining that the plant would be operated the
next day and that their jobs were open for them should they want to come in has been held to ISSUE: (1) WON there was interference; (2) WON the bank committed “surface bargaining”
be an unfair labor practice, as an active interference with the right of collective bargaining
through dealing with the employees individually instead of through their collective bargaining HELD: (1) NONE. Article 248(a) of the Labor Code, considers it an unfair labor practice when
representatives. an employer interferes, restrains or coerces employees in the exercise of their right to self-
organization or the right to form association. The right to self-organization necessarily
Although the union is on strike, the employer is still under obligation to bargain with the union includes the right to collective bargaining. Parenthetically, if an employer interferes in the
as the employees’ bargaining representative. selection of its negotiators or coerces the Union to exclude from its panel of negotiators a
representative of the Union, and if it can be inferred that the employer adopted the said act to
Individual solicitation of the employees or visiting their homes, with the employer or his yield adverse effects on the free exercise to right to self-organization or on the right to
representative urging the employees to cease union activity or cease striking, constitutes collective bargaining of the employees, ULP under Article 248(a) in connection with Article
unfair labor practice. All the above-detailed activities are unfair labor practices because they 243 of the Labor Code is committed.
tend to undermine the concerted activity of the employees, an activity to which they are
entitled free from the employer’s molestation. In order to show that the employer committed ULP under the Labor Code, substantial
evidence is required to support the claim. Substantial evidence has been defined as such
Indeed, when the respondents offered reinstatement and attempted to “bribe” the strikers with relevant evidence as a reasonable mind might accept as adequate to support a conclusion. In
“comfortable cots,” “free coffee and occasional movies,” “overtime” pay for “work performed in the case at bar, the Union bases its claim of interference on the alleged suggestions of
excess of eight hours,” and “arrangements” for their families, so they would abandon the Diokno to exclude Umali from the Union’s negotiating panel.
strike and return to work, they were guilty of strike-breaking and/or union-busting and,
consequently, of unfair labor practice. It is equivalent to an attempt to break a strike for an The circumstances that occurred during the negotiation do not show that the suggestion
employer to offer reinstatement to striking employees individually, when they are represented made by Diokno to Divinagracia is an anti-union conduct from which it can be inferred that
the Bank consciously adopted such act to yield adverse effects on the free exercise of the
23

right to self-organization and collective bargaining of the employees, especially considering ISSUE: WON the CA committed grave abuse of discretion in finding petitioner guilty of illegal
that such was undertaken previous to the commencement of the negotiation and dismissal and UPL?
simultaneously with Divinagracia’s suggestion that the bank lawyers be excluded from its
negotiating panel. RULING: No. The special civil action for Certiorari is intended for the correction of errors of
jurisdiction only or grave abuse of discretion amounting to lack or excess of jurisdiction. Its
The records show that after the initiation of the collective bargaining process, with the principal office is only to keep the inferior court within the parameters of its jurisdiction or to
inclusion of Umali in the Union’s negotiating panel, the negotiations pushed through. The prevent it from committing such a grave abuse of discretion amounting to lack or excess of
complaint was made only on August 16, 1993 after a deadlock was declared by the Union on jurisdiction. The essential requisites for a Petition for Certiorari under Rule 65 are:
June 15, 1993.
(1) the writ is directed against a tribunal, a board, or an officer exercising judicial
It is clear that such ULP charge was merely an afterthought. The accusation occurred after or quasi-judicial function;
the arguments and differences over the economic provisions became heated and the parties (2) such tribunal, board, or officer has acted without or in excess of jurisdiction,
had become frustrated. It happened after the parties started to involve personalities. As the or with grave abuse of discretion amounting to lack or excess of jurisdiction; and
public respondent noted, passions may rise, and as a result, suggestions given under less (3) there is no appeal or any plain, speedy, and adequate remedy in the ordinary
adversarial situations may be colored with unintended meanings. Such is what appears to course of law.
have happened in this case.
In the case, petitioners failed to meet the third requisite for the proper invocation of Petition
(2) NO. Surface bargaining is defined as “going through the motions of negotiating” without for Certiorari that there is no appeal or any plain, speedy, and adequate remedy in the
any legal intent to reach an agreement.” ordinary course of law. They simply alleged that the Court of Appeals gravely abuse its
discretion which amount to lack or excess of jurisdiction in rendering the assailed Decision
The Union alleges that the Bank violated its duty to bargain; hence, committed ULP under and Resolution. They did not bother to explain why an appeal cannot possibly cure the errors
Article 248(g) when it engaged in surface bargaining. It alleged that the Bank just went committed by the appellate court. Where the issue or question involves or affects the wisdom
through the motions of bargaining without any intent of reaching an agreement, as evident in or legal soundness of the decision, and not the jurisdiction of the court to render said
the Bank’s counter-proposals. It explained that of the 34 economic provisions it made, the decision, the same is beyond the province of a petition for certiorari. In any event, granting
Bank only made 6 economic counterproposals. Further, as borne by the minutes of the arguendo, that the present petition is proper, still it is dismissible. The Court of Appeals
meetings, the Bank, after indicating the economic provisions it had rejected, accepted, cannot be said to have acted with grave abuse of discretion amounting to lack or excess of
retained or were open for discussion, refused to make a list of items it agreed to include in jurisdiction in annulling the Decision of the NLRC because the findings of the Court of
the economic package. Appeals that private respondent Tamondong was indeed a supervisory employee and not a
managerial employee, thus, eligible to join or participate in the union activities of private
The minutes of meetings from March 12, 1993 to June 15, 1993 do not show that the Bank respondent CUSE, were supported by evidence on record. Tamondong may have possessed
had any intention of violating its duty to bargain with the Union. Records show that after the enormous powers and was performing important functions that goes with the position of
Union sent its proposal to the Bank on February 17, 1993, the latter replied with a list of its Personnel Superintendent, nevertheless, there was no clear showing that he is at liberty, by
counter-proposals on February 24, 1993. Thereafter, meetings were set for the settlement of using his own discretion and disposition, to lay down and execute major business and
their differences. The minutes of the meetings show that both the Bank and the Union operational policies for and in behalf of CAPASCO.
exchanged economic and non-economic proposals and counter-proposals.
Cainta Catholic Sch. vs. Cainta Catholic Sch. Emp. Union, G.R. No. 15102, May 2, 2006
The Union has not been able to show that the Bank had done acts, both at and away from FACTS: On 15 October 1993, petitioner school retired Llagas and Javier, President and Vice-
the bargaining table, which tend to show that it did not want to reach an agreement with the president of respondent union, respectively, who had rendered more than twenty (20) years
Union or to settle the differences between it and the Union. Admittedly, the parties were not of continuous service, pursuant to Section 2, Article X of the CBA, to wit:
able to agree and reached a deadlock. However, it is herein emphasized that the duty to
bargain “does not compel either party to agree to a proposal or require the making of a An employee may be retired, either upon application by the employee himself or by the
concession.” decision of the Director of the School, upon reaching the age of sixty (60) or after having
rendered at least twenty (20) years of service to the School the last three (3) years of which
Hence, the parties’ failure to agree did not amount to ULP under Article 248(g) for violation of must be continuous.
the duty to bargain.
Because of the foregoing, the union filed a Notice of Strike with the NCMB and later staged a
NOTE: (on the allegation of the bank’s refusal to give certain information) The Union, did not, strike and picketed in the school’s entrance. Later, the union filed a complaint for unfair labor
as the Labor Code requires, send a written request for the issuance of a copy of the data practice against petitioner school before the NLRC.
about the Bank’s rank and file employees. Moreover, as alleged by the Union, the fact that
the Bank made use of the aforesaid guestimates, amounts to a validation of the data it had The School avers that the retirement of Llagas and Javier was clearly in accordance with a
used in its presentation. specific right granted under the CBA. The School justifies its actions by invoking our rulings
in Pantranco North Express, Inc. v. NLRC and Bulletin Publishing Corporation v. Sanchez
Cathay Pacific Steel vs. CA, G. R. No. 164561, August 30, 2006 that no unfair labor practice is committed by management if the retirement was made in
FACTS: Cathay Pacific Steel Corporation (CAPASCO), a domestic corporation engaged in accord with management prerogative or in case of voluntary retirement, upon approval of
the business of manufacturing steel products. Tomondong was the Assistant to the Personnel management.
Manager for its Cainta. Thereafter, he was promoted to the position of
Personnel/Administrative Officer, and later to that of Personnel Superintendent. The The Union, on the other hand, argues that the retirement of the two union officers is a mere
supervisory personnel of CAPASCO launched a move to organize a union among their ranks, subterfuge to bust the union.
later known as private respondent CUSE. Private respondent Tamondong actively involved
himself in the formation of the union and was even elected as one of its officers after its ISSUE: Whether or not the retirement of Llagas and Javier is legal.
creation. Consequently, CAPASCO sent a memo to Tamondong requiring him to explain and
to discontinue from his union activities, with a warning that a continuance thereof shall HELD: The SC held that the termination of employment of Llagas and Javier was valid,
adversely affect his employment in the company. Tamondong ignored said warning and arising as it did from a management prerogative granted by the mutually-negotiated CBA
made a reply letter invoking his right as a supervisory employee to join and organize a labor between the School and the Union.
union. Thereafter CAPASCO terminated the employment of Tamondong on the ground of
loss of trust and confidence, citing his union activities as acts constituting serious disloyalty to Pursuant to the existing CBA, the School has the option to retire an employee upon reaching
the company. Tamondong challenged his dismissal for being illegal and as an act involving the age limit of sixty (60) or after having rendered at least twenty (20) years of service to the
unfair labor practice by filing a Complaint for Illegal Dismissal and Unfair Labor Practice School, the last three (3) years of which must be continuous. Retirement is different specie of
before the NLRC. Though he admitted his active role in the formation of a union composed of termination of employment from dismissal for just or authorized causes under Articles 282
supervisory personnel in the company, he claimed that such was not a valid ground to and 283 of the Labor Code. While in all three cases, the employee to be terminated may be
terminate his employment because it was a legitimate exercise of his constitutionally unwilling to part from service, there are eminently higher standards to be met by the employer
guaranteed right to self-organization. CAPASCO contended that Tamondong's position as validly exercising the prerogative to dismiss for just or authorized causes. In those two
Personnel Superintendent and the functions actually performed by him in the company instances, it is indispensable that the employer establish the existence of just or authorized
makes him a managerial employee, thus, under the law he was prohibited from joining a causes for dismissal as spelled out in the Labor Code. Retirement, on the other hand, is the
union as well as from being elected as one of its officers. The LA ruled in favor of Tamonding result of a bilateral act of the parties, a voluntary agreement between the employer and the
finding CAPASCO guilty of UPL and Illegal dismissal. On appeal, the NLRC only modified the employee whereby the latter after reaching a certain age agrees and/or consents to sever his
ruling by dismissing the complaints for UPL and Illegal dismissal and also the award for employment with the former.
damages but ordered the payment of backwages to Tamondong. Petitioners filed a Motion
for Clarification and Partial Reconsideration, while, Tamondong filed a Motion for Article 287 of the Labor Code, as amended, governs retirement of employees, stating:
Reconsideration of the said NLRC Decision, but the NLRC affirmed its original Decision. On
petititon, the Court of Appeals granted the nullification of the decision of the NLR. Hence, this ART. 287. Retirement. – Any employee may be retired upon reaching the
present Petition for Certiorari. retirement age established in the collective bargaining agreement or other
applicable employment contract.
24

In case of retirement, the employee shall be entitled to receive such that such dissolution was a valid exercise of a management prerogative. Thus this appeal is
retirement benefits as he may have earned under existing laws and any taken.
collective bargaining agreement and other agreements: Provided, however, That
an employee’s retirement benefits under any collective bargaining agreement Petitioner argued that the 18 security guards affected are part of the bargaining unit and
and other agreements shall not be less than those provided herein. covered by the existing collective bargaining contract, as such, their transfers and eventual
In the absence of a retirement plan or agreement providing for dismissals are illegal being done in violation of the existing contract. The Company
retirement benefits of employees in the establishment, an employee upon maintained that in contracting out the security service and redeploying the 18 security guards
reaching the age of sixty (60) years or more, but not beyond sixty-five (65) years affected, it was merely performing its legitimate prerogative to adopt the most efficient and
which is hereby declared the compulsory retirement age, who has served at least economical method of operation, that said action was motivated by business consideration in
five (5) years in the said establishment, may retire and shall be entitled to line with past established practice and made after notice to and discussion with the Union,
retirement pay equivalent to at least one-half (1/2) month salary for every year of that the 18 guards concerned were dismissed for wilfully refusing to obey the transfer order,
service, a fraction of at least six (6) months being considered as one whole year. and that the strike staged by the Union is illegal.

By their acceptance of the CBA, the Union and its members are obliged to abide by the ISSUE: Whether the existing collective bargaining contract on maintaining security guard
commitments and limitations they had agreed to cede to management. The questioned section, among others, constitute a bar to the decision of the management to contract out
retirement provisions cannot be deemed as an imposition foisted on the Union, which very security guards.
well had the right to have refused to agree to allowing management to retire retire
employees with at least 20 years of service. RULING: YES. The strike was legal because there was a violation of the collective bargaining
agreement by Company. It was part of the CBA that the Security Guard Section will remain.
It should not be taken to mean that retirement provisions agreed upon in the CBA are Yet, the Company did not comply with the stipulation in CBA. It was thus an assurance of
absolutely beyond the ambit of judicial review and nullification. A CBA, as a labor contract, is security of tenure, at least, during the lifetime of the agreement. For what is involved is the
not merely contractual in nature but impressed with public interest. If the retirement integrity of the agreement reached, the terms of which should be binding on both parties
provisions in the CBA run contrary to law, public morals, or public policy, such provisions may
very well be voided. Certainly, a CBA provision or employment contract that would allow The stand of Shell Company as to the scope of management prerogative is not devoid of
management to subvert security of tenure and allow it to unilaterally “retire” employees after plausibility, management prerogative of the Company would have been valid if it were not
one month of service cannot be upheld. Neither will the Court sustain a retirement clause that bound by what was stipulated in CBA. The freedom to manage the business remains with
entitles the retiring employee to benefits less than what is guaranteed under Article 287 of the management. It cannot be denied the faculty of promoting efficiency and attaining economy
Labor Code, pursuant to the provision’s express proviso thereto in the provision. by a study of what units are essential for its operation. To it belongs the ultimate
determination of whether services should be performed by its personnel or contracted to
Yet the CBA in the case at bar contains no such infirmities which must be stricken down. outside agencies. However, while management has the final say on such matter, the labor
Twenty years is a more than ideal length of service an employee can render to one employer. union is not to be completely left out.
Under ordinary contemplation, a CBA provision entitling an employee to retire after 20 years
of service and accordingly collect retirement benefits is “reward for services rendered since it An unfair labor practice is committed by a labor union or its agent by its refusal ‘to bargain
enables an employee to reap the fruits of his labor — particularly retirement benefits, whether collectively with the employer’. Collective bargaining does not end with the execution of an
lump-sum or otherwise — at an earlier age, when said employee, in presumably better agreement, being a continuous process, the duty to bargain necessarily imposing on the
physical and mental condition, can enjoy them better and longer.” parties the obligation to live up to the terms of such a collective bargaining agreement if
entered into, it is undeniable that non-compliance therewith constitutes an unfair labor
A CBA may validly accord management the prerogative to optionally retire an employee practice.
under the terms and conditions mutually agreed upon by management and the bargaining
union, even if such agreement allows for retirement at an age lower than the optional The right to self-organization guarded by the Industrial Peace Act explicitly includes the right
retirement age or the compulsory retirement age. “to engage in concerted activities for the purpose of collective bargaining and to the mutual
aid or protection.” The employee, tenant or laborer is inhibited from striking or walking out of
Purefoods vs. Nagkakaisang Samahang Manggagawa ng Purefoods, G.R. No. 150896, his employment only when so enjoined by the CIR and after a dispute has been submitted
August 28, 2008 thereto and pending award or decision by the court of such dispute.
Facts: Three labor organizations and a federation are respondents in this case NAGSAMA-
Purefoods, the exclusive bargaining agent of the rank-and-file workers of Purefoods, STFWU, In the present case, the employees or laborers may strike before being ordered not to do so
(Sto. Tomas, Batangas); and PGFWU (Sta. Rosa, Laguna). These organizations were and before an industrial dispute is submitted to the CIR, subject to the power of the latter,
affiliates of the respondent federation, Purefoods Unified Labor Organization (PULO). The after hearing when public interest so requires or when the dispute cannot, in its opinion, be
three labor organizations manifested their desire to re-negotiate the collective bargaining promptly decided or settled, to order them to return to work, with the consequence that if the
agreement, submitting their respective demands and proposals and authorizing a negotiating strikers fail to return to work, when so ordered, the court may authorize the employer to
panel which included among others a PULO representative. While Purefoods formally accept other employees or laborers.” Thus a strike may not be staged only when, during the
acknowledged receipt of the union’s proposals, but refused to negotiate with the unions pendency of an industrial dispute, the CIR has issued the proper injunction against the
should a PULO representative be in the panel which resulted in a deadlock. However, the laborers (section 19, Commonwealth Act No. 103, as amended).
petitioner company concluded a new CBA with another union in its farm in Malvar, Batangas ————-
and terminated the service of regular rank-and file workers in Sto Tomas. The farm manager, **NOTE:
supervisors and electrical workers of the Sto. Tomas farm, who were members of another
union, were nevertheless retained by the company in its employ. The 4 respondent labor BELIEF IN GOOD FAITH THAT EMPLOYER COMMITTED UNFAIR LABOR PRACTICE
organizations jointly instated a complaint for Unfair Labor Practice, illegal lockout/dismissal RENDERS STRIKE LEGAL:
and damages.
It is not even required that there be in fact an unfair labor practice committed by the
Issue: WON the refusal of Purefoods to recognized PULO as a labor organizations’ affiliation employer. It suffices, if such a belief in good faith is entertained by labor, as the inducing
constituted undue interference in, and restraint on the exercise of the employees’ right to self- factor for staging a strike. So it was declared: “As a consequence, we hold that the strike in
organization and free collective bargaining question had been called to offset what petitioners were wanted in believing in good faith to
be unfair labor practices on the part of Management, that petitioners were not bound,
Held: Yes! It is crystal clear that the closure of the Sto. Tomas farm was made in bad faith. therefore, to wait for the expiration of thirty (30) days from notice of strike before staging the
Badges of bad faith are evident from the following acts of the petitioner: it unjustifiably same, that said strike was not, accordingly, illegal and that the strikers had not thereby lost
refused to recognize the STFWU’s and the other unions’ affiliation with PULO; it concluded a their status as employees of respondents herein.”
new CBA with another union in another farm during the agreed indefinite suspension of the
collective bargaining negotiations; it surreptitiously transferred and continued its business in a BPI Employees Union-Davao City-FUBU vs. BPI, G.R. No. 174912, July 24, 2013
less hostile environment; and it suddenly terminated the STFWU members, but retained and FACTS: BPI Operations Management Corporation (BOMC), which was created pursuant to
brought the non-members to the Malvar farm. Petitioner presented no evidence to support Central Bank Circular No. 1388, Series of 1993 (CBP Circular No. 1388, 1993), and primarily
the contention that it was incurring losses or that the subject farm’s lease agreement was pre- engaged in providing and/or handling support services for banks and other financial
terminated. Ineluctably, the closure of the Sto. Tomas farm circumvented the labor institutions, is a subsidiary of the Bank of Philippine Islands (BPI) operating and functioning
organization’s right to collective bargaining and violated the members’ right to security of as an entirely separate and distinct entity.
tenure.
A service agreement between BPI and BOMC was initially implemented in BPIs Metro Manila
Shell Oil Workers Union vs. Shell Oil Company, G.R. No. L-28607, May 31, 1971 branches. In this agreement, BOMC undertook to provide services such as check clearing,
FACTS: Respondent Shell Company of the Philippines (COMPANY) dissolved its security delivery of bank statements, fund transfers, card production, operations accounting and
guard section stationed at its Pandacan Installation, notwithstanding its (guard section) control, and cash servicing, conformably with BSP Circular No. 1388. Not a single BPI
continuance and that such is assured by an existing collective bargaining contract. The employee was displaced and those performing the functions, which were transferred to
respondent company transferred 18 security guards to its other department and consequently BOMC, were given other assignments.
hired a private security agency to undertake the work of said security guards. This resulted in
a strike called by petitioner Shell Oil Workers’ Union (UNION), The President certified it to The Manila chapter of BPI Employees Union (BPIEU-Metro ManilaFUBU) then filed a
respondent Court of Industrial Relations (CIR). CIR declared the strike illegal on the ground complaint for unfair labor practice (ULP). The Labor Arbiter (LA) decided the case in favor of
25

the union. The decision was, however, reversed on appeal by the NLRC. BPIEU-Metro
Manila-FUBU filed a petition for certiorari before the CA which denied it, holding that BPI Although the cited law pertains to the specific employee who filed a case or given a testimony
transferred the employees in the affected departments in the pursuit of its legitimate against the employer, it should be construed in line with the spirit and purpose of said Section
business. 4 and of the legislation of which it forms part — namely, to assure absolute freedom of the
employees and laborers to establish labor organizations and unions, as well as to prefer
The service agreement was likewise implemented in Davao City. Later, a merger between charges before the proper organs of the Government for violations of our labor laws.
BPI and Far East Bank and Trust Company (FEBTC) took effect on April 10, 2000 with BPI
as the surviving corporation. Thereafter, BPIs cashiering function and FEBTCs cashiering, If the dismissal of an employee due to the filing by him of said charges would be and is an
distribution and bookkeeping functions were handled by BOMC. Consequently, twelve (12) undue restraint upon said freedom, the dismissal of his brother owing to the non-withdrawal
former FEBTC employees were transferred to BOMC to complete the latters service of the charges of the former, would be and constitute as much a restraint upon the same
complement. freedom. In fact, it may be greater and more effective restraint thereto. Indeed, a complainant
may be willing to risk the hazards of a possible and even probable retaliatory action by the
BPI Davaos rank and file collective bargaining agent, BPI Employees Union-Davao City- employer in the form of a dismissal or another discriminatory act against him personally,
FUBU (Union), objected to the transfer of the functions and the twelve (12) personnel to considering that nobody is perfect, that everybody commits mistakes and that there is always
BOMC contending that the functions rightfully belonged to the BPI employees and that the a possibility that the employer may find in the records of any employee, particularly if he has
Union was deprived of membership of former FEBTC personnel who, by virtue of the merger, long been in the service, some act or omission constituting a fault or negligence which may
would have formed part of the bargaining unit represented by the Union pursuant to its union be an excuse for such dismissal or discrimination. Yet, such complainant may not withstand
shop provision in the CBA. the pressure that would result if his brother or another member of his immediate family were
threatened with such action unless the charges in question were withdrawn.
ISSUE: Whether or not the act of BPI to outsource the cashiering, distribution and
bookkeeping functions to BOMC is in conformity with the law and the existing CBA. What is prohibited to be done directly shall not be allowed to be accomplished indirectly.
Thus in the Matter of Quidnick Dye Works Inc. and Federation of Dyers, Finishers, Printers
HELD: Yes. Labor Law- only gross violations of the economic provisions of the CBA are and Bleachers of America (2 NLRB 963) it was held that the dismissal of a laborer on account
treated as ULP. Otherwise, they are mere grievances. of union activities of his brother constituted an ULP.

In the present case, the alleged violation of the union shop agreement in the CBA, even The discharge of relatives of an employee who has himself been discriminately discharged,
assuming it was malicious and flagrant, is not a violation of an economic provision in the for no other reason than the relation, is itself a discriminatory discharge, in violation of the
agreement. The provisions relied upon by the Union were those articles referring to the Act. An illustration is Memphis Furniture Co. (3 NLRB 26 [1937]), “where the evidence
recognition of the union as the sole and exclusive bargaining representative of all rank-and- indicated that the sole reason for the dismissal of a female employee was that she was the
file employees, as well as the articles on union security, specifically, the maintenance of wife of an employee who has been discharged. It was held that the discharge under the
membership in good standing as a condition for continued employment and the union shop circumstances was discriminatory and a violation of the Act, even though discharged female
clause. It failed to take into consideration its recognition of the banks exclusive rights and employee was not herself a member of any union. The respondent thus made union
prerogatives, likewise provided in the CBA, which included the hiring of employees, membership and activities a bar to the employment not only of the union member himself but
promotion, transfers, and dismissals for just cause and the maintenance of order, discipline of members of his family as well. A more effective mode of discouraging of union affiliation
and efficiency in its operations. could hardly be found then the knowledge that such activities put not merely the union
member’s employment but that of those closely related to him in jeopardy.“
The Union, however, insists that jobs being outsourced to BOMC were included in the
existing bargaining unit, thus, resulting in a reduction of a number of positions in such unit. In addition to violating Section 4(a) (5) of Republic Act No. 875, the discharge of Apolonio
The reduction interfered with the employees right to self-organization because the power of a San Jose is, therefore, an ULP.
union primarily depends on its strength in number.
Rizal Cement Workers vs. Madrigal, G.R. No. L-19767, April 30, 1964
It is incomprehensible how the "reduction of positions in the collective bargaining unit" Facts: The petitioner Union staged a strike at the plant of the respondent Rizal Cement Co.,
interferes with the employees right to self-organization because the employees themselves Inc. in Binangonan, Rizal. In the early morning of the following day, that is, on May 28, 1956,
were neither transferred nor dismissed from the service. In the case at hand, the union has Candido de Leon warehouseman-encargado at the Bodega Tanque, received a telephone
not presented even an iota of evidence that petitioner bank has started to terminate certain call from one Johnny de Leon, manager of the respondent Rizal Cement Co., Inc., with the
employees, members of the union. In fact, what appears is that the Bank has exerted utmost information that the Union staged a strike against the company on the previous day, May 27,
diligence, care and effort to see to it that no union member has been terminated. In the 1956, in Binangonan, Rizal De Leon further informed him that he should take precautionary
process of the consolidation or merger of the two banks which resulted in increased measures in protecting the properties of the company stored at the Bodega Tanque because
diversification of functions, some of these non-banking functions were merely transferred to of the strikers caused damage to the factory in Binangonan and sabotage might occur. For
the BOMC without affecting the union membership. this reason, he was advised by the manager to request the members of the Union to stay
meanwhile outside the premises of the Bodega Tanque. What he did in the morning of May
It is to be emphasized that contracting out of services is not illegal per se. It is an exercise of 28, 1956 was to station himself at the gate of the compound. When the workers arrived for
business judgment or management prerogative. Absent proof that the management acted in work at 7:00 a.m., he did not allow the 21 complaining workers who are members of the
a malicious or arbitrary manner, the Court will not interfere with the exercise of judgment by Union to enter the gate and allowed only those who are not members of said Union.
an employer. In this case, bad faith cannot be attributed to BPI because its actions were
authorized by CBP Circular No. 1388, Series of 1993 issued by the Monetary Board of the Upon refusal of Candido de Leon to allow the complaining workers to work on that day, the
then Central Bank of the Philippines (now Bangko Sentral ng Pilipinas). Union, sent a letter to the manager of the Bodega alleging discrimination. The manager sent
a reply denying such allegation.
Philippine American Cigar and Cigarette Factory Workers vs. Philippine American Cigar, G.R.
No.L-18364, February 28, 1963 Issue: WON there was discrimination against the employees who are not allowed to work in
FACTS: October 23, 1958, Apolonio San Jose’s brother, Francisco San Jose, who is also a the Bodega.
regular worker of the respondent and a member of the complainant union, filed a charge for
ULP against herein respondent, which case is still pending. Held: It is not herein controverted that the complainants were locked out or denied work by
the respondent Company. However, for the discrimination by reason of union membership to
Subsequent to the filing of the said charge, the respondent by its manager Chue Yiong, be considered an unfair labor practice, the same must have been committed to courage or
summoned and advised union president Lazaro Peralta that if Francisco San Jose will not discourage such membership in the union.
withdraw his charge against the company, the company will also dismiss his brother Apolonio
San Jose, to which the union president replied that should not be the attitude of the company This cannot be said of the act of the Company complained of. As clearly established by the
because Apolonio has nothing to do with his brother’s case. evidence, its refusal to all complainants to work and requirement that the latter stay out of the
premises in the meantime was borne out of the Company’s justified apprehension and fear
On January 24, 1959, respondent did dismiss Apolonio San Jose without just and valid cause that sabotage might be committed in the warehouse where the products machinery and spare
and in gross violation of the operative CBA between the complainant union and respondent parts were stored, as has been the case in Binangonan. It has never been shown that the act
corporation. of the Company was intended to induce the complainants to renounce their union-
membership or as a deterrent for non-members to affiliate therewith, nor as a retaliatory
ISSUE: Whether the dismissal of a relative of an employee who filed an action against the measure for activities in the union or in furtherance of the cause of the union.
employer is an ULP.
PICOP vs. Taneca, G.R. No. 160828, August 9, 2010
RULING: YES. Section 4(a) (5) of Republic Act No. 875, provides that : Facts: Respondents were regular rank-and-file employees of PRI and bona fide members of
Nagkahiusang Mamumuo sa PRI Southern Philippines Federation of Labor (NAMAPRI-
“(a) It shall be ULP for an employer: SPFL), which is the collective bargaining agent for the rank-and-file employees of petitioner
PRI. PRI has a CBA with NAMAPRI-SPFL. The CBA contained the following union security
… (5) To dismiss, discharge, or otherwise prejudice or discriminate against an provisions:
employee for having filed charges or for having given or being about to give testimony under
this Act.” Article II- Union Security and Check-Off
26

two years, effective 1 July 2005 to 30 June 2007. The parties signed the Memorandum of
Section 6. Maintenance of membership. Agreement on 20 May 2005 and the employees ratified it on 27 May 2005.

6.1 All employees within the appropriate bargaining unit who are members of the UNION at On 21 June 2005, NUWHRAIN was accorded by the Labor Relations Division of the
the time of the signing of this AGREEMENT shall, as a condition of continued employment by Department of Labor and Employment (DOLE) the status of a legitimate labor organization.
the COMPANY, maintain their membership in the UNION in good standing during the Thereafter, NUWHRAIN exercised the right to challenge the majority status of the incumbent
effectivity of this AGREEMENT. union, HIMPHLU, by filing a Petition for Certification Election on 28 June 2005.

6.3 The COMPANY, upon the written request of the UNION and after compliance with the On 5 July 2007, the Industrial Relations Division of the DOLE allowed the registration of the
requirements of the New Labor Code, shall give notice of termination of services of any Memorandum of Agreement executed between HIMPHLU and the Hotel, extending the
employee who shall fail to fulfill the condition provided in Section 6.1 and 6.2 of this Article effectivity of the existing Collective Bargaining Agreement for another two years.

Atty. Fuentes sent a letter to the management of PRI demanding the termination of After the lapse of the 60-day freedom period, but pending the disposition of the Petition for
employees who allegedly campaigned for, supported and signed the Petition for Certification Certification Election filed by NUWHRAIN, HIMPHLU served the Hotel with a written demand
Election of the Federation of Free Workers Union (FFW) during the effectivity of the CBA. dated 28 July 200510 for the dismissal of 36 employees following their expulsion from
NAMAPRI-SPFL considered said act of campaigning for and signing the petition for HIMPHLU for alleged acts of disloyalty and violation of its Constitution and by-laws. An
certification election of FFW as an act of disloyalty and a valid basis for termination for a Investigation Report11 was attached to the said written demand, stating that the 36
cause in accordance with its Constitution and By-Laws, and the terms and conditions of the employees, who were members of HIMPHLU, joined NUWHRAIN, in violation of Section 2,
CBA, specifically Article II, Sections 6.1 and 6.2 on Union Security Clause. Article IV of the Collective Bargaining Agreement, which provided for a union security
clause.12 On 1 August 2005, the Hotel issued Disciplinary Action Notices13 (Notices) to the
On October 16, 2000, PRI served notices of termination for causes to employees whom 36 employees identified in the written demand of HIMPHLU. The Notices directed the 36
NAMAPRIL-SPFL sought to be terminated on the ground of “acts of disloyalty” committed employees to submit a written explanation for their alleged acts of disloyalty and violation of
against it when respondents allegedly supported and signed the Petition for Certification the union security clause for which HIMPHLU sought their dismissal.
Election of FFW before the “freedom period” during the effectivity of the CBA. A Notice dated
October 21, 2000 was also served on the DOLE, Caraga Region. NLRC decided that there was no unfair labor practice.NUWHRAIN filed a Motion for
Reconsideration of the foregoing NLRC Resolution. It was denied by the NLRC in another
Respondents then accused PRI of ULP. Resolution dated 30 June 2006.20 Thus, NUWHRAIN filed a Petition for Certiorari before the
Court of Appeals, docketed as C.A. G.R. SP No. 96171.
Issue: WON respondents were validly terminated.
In the meantime, on 16 June 2006, the Certification Election for regular rank and file
Held: “Union security” is a generic term, which is applied to and comprehends “closed shop,” employees of the Hotel was held, which HIMPHLU won. It was accordingly certified as the
“union shop,” “maintenance of membership,” or any other form of agreement which imposes exclusive bargaining agent for rank and file employees of the Hotel. On 30 May 2007, the
upon employees the obligation to acquire or retain union membership as a condition affecting Court of Appeals promulgated its Decision2 upholding the Resolution of the NLRC. It
employment. There is union shop when all new regular employees are required to join the declared that the Hotel had acted prudently when it issued the Notices to the 36 employees
union within a certain period as a condition for their continued employment. There is after HIMPHLU demanded their dismissal. It clarified that these Notices did not amount to the
maintenance of membership shop when employees, who are union members as of the termination of the employees concerned but merely sought their explanation on why the
effective date of the agreement, or who thereafter become members, must maintain union union security clause should not be applied to them. The appellate court also gave credence
membership as a condition for continued employment until they are promoted or transferred to the denial by the officers of the respondent and the Hotel that they made statements
out of the bargaining unit, or the agreement is terminated. A closed shop may be defined as favoring HIMPHLU over NUWHRAIN during the reconciliatory conferences. The Court of
an enterprise in which, by agreement between the employer and his employees or their Appeals further noted that the unhampered organization and registration of NUWHRAIN
representatives, no person may be employed in any or certain agreed departments of the negated its allegation that the Hotel required its employees not to join a labor organization as
enterprise unless he or she is, becomes, and, for the duration of the agreement, remains a a condition for their employment.
member in good standing of a union entirely comprised of or of which the employees in
interest are a part. NUWHRAIN’s Motion for Reconsideration of the aforementioned Decision of the Court of
Appeals was denied by the same court in a Resolution. Hence, the present Petition, in which
However, in terminating the employment of an employee by enforcing the union security NUWHRAIN makes the following assignment of errors:
clause, the employer needs to determine and prove that: (1) the union security clause is
applicable; (2) the union is requesting for the enforcement of the union security provision in ISSUE: Whether the dismissal of the subject employees in accordance with CBA’s Union
the CBA; and (3) there is sufficient evidence to support the decision of the union to expel the Security Clause deemed unfair labor practice.
employee from the union. These requisites constitute just cause for terminating an employee
based on the union security provision of the CBA. HELD: No. NUWHRAIN maintains that the respondent committed unfair labor practice when
(1) the Hotel issued the Notices to the 36 employees, former members of HIMPHLU, who
As to the first requisite, there is no question that the CBA between PRI and respondents switched allegiance to NUWHRAIN; and (2) the officers of the respondent and the Hotel
included a union security clause. Secondly, it is likewise undisputed that NAMAPRI-SPFL, in allegedly uttered statements during the reconciliatory conferences indicating their preference
two (2) occasions demanded from PRI, in their letters dated May 16 and 23, 2000, to for HIMPHLU and their disapproval of NUWHRAIN. This argument is specious.
terminate the employment of respondents due to their acts of disloyalty to the Union.
However, as to the third requisite, we find that there is no sufficient evidence to support the “Union security” is a generic term which is applied to and comprehends “closed shop,” “union
decision of PRI to terminate the employment of the respondents. shop,” “maintenance of membership” or any other form of agreement which imposes upon
employees the obligation to acquire or retain union membership as a condition affecting
The mere signing of the authorization in support of the Petition for Certification Election of employment. Article 248(e) of the Labor Code recognizes the effectivity of a union shop
FFW on March 19, 20 and 21, or before the “freedom period,” is not sufficient ground to clause:
terminate the employment of respondents inasmuch as the petition itself was actually filed
during the freedom period. Nothing in the records would show that respondents failed to The law allows stipulations for “union shop” and “closed shop” as a means of encouraging
maintain their membership in good standing in the Union. Respondents did not resign or workers to join and support the union of their choice in the protection of their rights and
withdraw their membership from the Union to which they belong. Respondents continued to interests vis-à-vis the employer. By thus promoting unionism, workers are able to negotiate
pay their union dues and never joined the FFW. with management on an even playing field and with more persuasiveness than if they were to
individually and separately bargain with the employer. In Villar v. Inciong, this Court held that
NUWHRAIN vs. NLRC, G.R. No. 179402, September 30, 2008 employees have the right to disaffiliate from their union and form a new organization of their
FACTS: Petitioner NUWHRAIN is a legitimate labor organization composed of rank-and-file own; however, they must suffer the consequences of their separation from the union under
employees of the Hotel, while respondent Acesite Philippines Hotel Corporation is the owner the security clause of the Collective Bargaining Agreement.
and operator of said Hotel.
In the present case, the Collective Bargaining Agreement includes a union security provision.
The Hotel entered into a Collective Bargaining Agreement with HI-MANILA PAVILION To avoid the clear possibility of liability for breaching the union security clause of the
HOTEL LABOR UNION (HIMPHLU), the exclusive bargaining agent of the rank-and-file Collective Bargaining Agreement and to protect its own interests, the only sensible option left
employees of the Hotel. Both parties consented that the representation aspect and other non- to the Hotel, upon its receipt of the demand of HIMPHLU for the dismissal of the 36
economic provisions of the Collective Bargaining Agreement were to be effective for five employees, was to conduct its own inquiry so as to make its own findings on whether there
years or until 30 June 2005; and the economic provisions of the same were to be effective for was sufficient ground to dismiss the said employees who defected from HIMPHLU. The
three years or until 30 June 2003. The parties subsequently re-negotiated the economic issuance by the respondent of the Notices requiring the 36 employees to submit their
provisions of the Collective Bargaining Agreement and extended the term of their effectivity explanations to the charges against them was the reasonable and logical first step in a fair
for another two years or until 30 June 2005.5 investigation. It is important to note that the Hotel did not take further steps to terminate the
36 employees. Instead, it arranged for reconciliatory conferences between the contending
During the 60-day freedom period which preceded the expiration of the Collective Bargaining unions in order to avert the possibility of dismissing the 36 employees for violation of the
Agreement, starting on 1 May 2005 and ending on 30 June 2005, the Hotel and HIMPHLU union security clause of the Collective Bargaining Agreement.
negotiated the extension of the provisions of the existing Collective Bargaining Agreement for
27

This Court, in Malayang Samahan ng Manggagawa sa M. Greenfield v. Ramos clearly stated The mere fact that one or more corporations are owned or controlled by the same or single
the general rule: the dismissal of an employee by the company pursuant to a labor union’s stockholder is not a sufficient ground for disregarding separate corporate personalities. Mere
demand in accordance with a union security agreement does not constitute unfair labor ownership by a single stockholder or by another corporation of all or nearly all of the capital
practice. An employer is not considered guilty of unfair labor practice if it merely complied in stock of a corporation is not of itself sufficient ground for disregarding the separate corporate
good faith with the request of the certified union for the dismissal of employees expelled from personality.
the union pursuant to the union security clause in the Collective Bargaining Agreement. In the
case at bar, there is even less possibility of sustaining a finding of guilt for unfair labor At first glance after reading the decision a quo, it would seem that the closure of respondent’s
practice where respondent did not dismiss the 36 employees, despite the insistence of operation is not justified. However, a deeper examination of the records along with the
HIMPHLU, the sole bargaining agent for the rank and file employees of the Hotel, on the evidence, would show that the closure, although it was done abruptly as there was no
basis of the union security clause of the Collective Bargaining Agreement. The only act compliance with the 30-day prior notice requirement, said closure was not intended to
attributed to the respondent is its issuance of the Notices which, contrary to being an unfair circumvent the provisions of the Labor Code on termination of employment. The closure of
labor practice, even afforded the employees involved a chance to be heard. operation by Complex on April 7, 1992 was not without valid reasons. Customers of
respondent alarmed by the pending labor dispute and the imminent strike to be foisted by the
In all, respondent had not committed any act which would constitute unfair labor practice. union, as shown by their strike vote, directed respondent Complex to pull-out its equipment,
machinery and materials to other safe bonded warehouse. Respondent being mere
BPI vs. BPI Employees Union-Davao Chapter, G.R. No. 164301, August 10, 2010 consignees of the equipment, machinery and materials were without any recourse but to
FACTS: In 2000, Far East Bank (FEB) was absorbed by the Bank of the Philippine Islands oblige the customers’ directive. The pull-out was effected on April 6, 1992. We can see here
(BPI). Now BPI has an existing Union Shop Clause agreement with the BPI Employees that Complex’s action, standing alone, will not result in illegal closure that would cause the
Union-Davao Chapter-Federation of Unions in BPI Unibank (BPI Union) whereby it is a pre- illegal dismissal of the complainant workers. Hence, the Labor Arbiter’s conclusion that since
condition that new employees must join the union before they can be regularized otherwise there were only 2 of respondent’s customers who have expressed pull-out of business from
they will not have a continued employment. By reason of the failure of the FEB employees to respondent Complex while most of the customer’s have not and, therefore, it is not justified to
join the union, BPI Union recommended to BPI their dismissal. BPI refused. The issue went close operation cannot be upheld. The determination to cease operation is a prerogative of
to voluntary arbitration where BPI won but the Court of Appeals reversed the Voluntary management that is usually not interfered with by the State as no employer can be required
Arbitrator. BPI appealed to the Supreme Court. to continue operating at a loss simply to maintain the workers in employment. That would be
taking of property without due process of law which the employer has the right to resist.
ISSUE: Whether or not the Union Shop agreement violated the constitutional right of security
of tenure of the FEB employees absorbed by BPI. Octavio vs. PLDT, G.R. No. 175492, February 27, 2013
Facts: PLDT and Gabay ng Unyon sa Telekominaksyon ng mga Superbisor (GUTS) entered
HELD: No. As a general rule, the State protects the workers right to security of tenure. An into a CBA covering the period January 1, 1999 to December 31, 2001 (CBA of 1999-2001).
employee’s services can only be terminated upon just and authorized causes. In this case, On October 1, 2000, PLDT hired Octavio as Sales System Analyst I on a probationary status.
the presence of a Union Shop Clause in the CBA between BPI and BPI Union must be He became a member of GUTS.
respected. Failure of an employee to join the union pursuant to the clause is an authorized
cause for BPI not to continue employing the employee concerned – and BPI must respect On May 31, 2002, PLDT and GUTS entered into another CBA covering the period January 1,
that provision of the CBA. In the hierarchy of labor rights, unionism is favored over security of 2002 to December 31, 2004 (CBA of 2002-2004). Claiming that he was not given the salary
tenure. A contrary interpretation of the Union Shop Clause would dilute its efficacy and put increases of P2,500.00 effective January 1, 2001 and P2,000.00 effective January 1, 2002,
the certified union that is supposedly being protected thereby at the mercy of management. Octavio wrote the President of GUTS, Adolfo Fajardo (Fajardo). Acting thereon and on similar
Nevertheless, the FEB employees are still entitled to the twin notice rule – this is to afford grievances from other GUTS members, Fajardo wrote the PLDT Human Resource Head to
them ample opportunity to whether or not join the union. inform management of the GUTS members’ claim for entitlement to the across-the-board
salary increases.
Complex Electronics Employees Association vs. Complex Electronics, G.R. No. 122136, July
19, 1999 The committee denied the claims of Octavio. Octavio filed before the Arbitration Branch of the
FACTS: Due to losses on production of the petitioner, it was constrained to cease operations. NLRC a Complaint for payment of said salary increases.
In the evening of April 6, 1992, the machinery, equipment and materials being used for
production at Complex were pulled-out from the company premises and transferred to the PLDT countered that the issues advanced by Octavio had already been resolved by the
premises of Ionics Circuit, Inc. (Ionics) at Cabuyao, Laguna. The following day, a total Union-Management Grievance Committee when it denied his claims through the Committee
closure of company operation was effected at Complex. Resolution. Moreover, the grant of across-the board salary increase for those who were
regularized starting January 1, 2002 and the exclusion thereto of those who were regularized
A complaint was, thereafter, filed with the Labor Arbitration Branch of the NLRC for unfair on January 1, 2001, do not constitute an act of unfair labor practice as would result in any
labor practice, illegal closure/illegal lockout, money claims for vacation leave, sick leave, discrimination or encourage or discourage membership in a labor organization. In fact, when
unpaid wages, 13th month pay, damages and attorney’s fees. The Union alleged that the the Union-Management Grievance Committee came up with the Committee Resolution, they
pull-out of the machinery, equipment and materials from the company premises, which considered the same as the most practicable and reasonable solution for both management
resulted to the sudden closure of the company was in violation of Section 3 and 8, Rule XIII, and union. At any rate, the said Committee Resolution had already become final and
Book V of the Labor Code of the Philippines and the existing CBA. Ionics was impleaded as a conclusive between the parties for failure of Octavio to elevate the same to the proper forum.
party defendant because the officers and management personnel of Complex were also In addition, PLDT claimed that the NLRC has no jurisdiction to hear and decide Octavio’s
holding office at Ionics with Lawrence Qua as the President of both companies. claims.

The Union anchors its position on the fact that Lawrence Qua is both the president of Issue: WON the decision of the Grievance Committee is binding.
Complex and Ionics and that both companies have the same set of Board of Directors. It
claims that business has not ceased at Complex but was merely transferred to Ionics, a Held: Yes. It is settled that "when parties have validly agreed on a procedure for resolving
runaway shop. To prove that Ionics was just a runaway shop, petitioner asserts that out of grievances and to submit a dispute to voluntary arbitration then that procedure should be
the 80,000 shares comprising the increased capital stock of Ionics, it was Complex that owns strictly observed." Moreover, we have held time and again that "before a party is allowed to
majority of said shares with P1,200,000.00 as its capital subscription and P448,000.00 as its seek the intervention of the court, it is a precondition that he should have availed of all the
paid up investment, compared to P800,000.00 subscription andP324,560.00 paid-up owing to means of administrative processes afforded him. Hence, if a remedy within the administrative
the other stockholders, combined. Thus, according to the Union, there is a clear ground to machinery can still be resorted to by giving the administrative officer concerned every
pierce the veil of corporate fiction. opportunity to decide on a matter that comes within his jurisdiction, then such remedy should
be exhausted first before the court’s judicial power can be sought. The premature invocation
ISSSUE: WON Ionics is merely a runaway shop of the court’s judicial intervention is fatal to one’s cause of action." "The underlying principle
of the rule on exhaustion of administrative remedies rests on the presumption that when the
HELD: NO. A “runaway shop” is defined as an industrial plant moved by its owners from one administrative body, or grievance machinery, is afforded a chance to pass upon the matter, it
location to another to escape union labor regulations or state laws, but the term is also used will decide the same correctly."
to describe a plant removed to a new location in order to discriminate against employees at
the old plant because of their union activities. It is one wherein the employer moves its By failing to question the Committee Resolution through the proper procedure prescribed in
business to another location or it temporarily closes its business for anti-union purposes. A the CBA, that is, by raising the same before a Board of Arbitrators, Octavio is deemed to
“runaway shop” in this sense, is a relocation motivated by anti-union animus rather than for have waived his right to question the same. Clearly, he departed from the grievance
business reasons. procedure mandated in the CBA and denied the Board of Arbitrators the opportunity to pass
upon a matter over which it has jurisdiction. Hence, and as correctly held by the CA,
In this case, however, Ionics was not set up merely for the purpose of transferring the Octavio’s failure to assail the validity and enforceability of the Committee Resolution makes
business of Complex. At the time the labor dispute arose at Complex, Ionics was already the same binding upon him. On this score alone, Octavio’s recourse to the labor tribunals
existing as an independent company. As earlier mentioned, it has been in existence since below, as well as to the CA, and, finally, to this Court, must therefore fail.
July 5, 1984 (8 years prior to the dispute). It cannot, therefore, be said that the temporary
closure in Complex and its subsequent transfer of business to Ionics was for anti-union Goya Inc. vs Goya Employees’ Union, G.R. No. 170054, January 21, 2013
purposes. The Union failed to show that the primary reason for the closure of the FACTS: Sometime in January 2004, petitioner Goya, Inc. (Company) hired contractual
establishment was due to the union activities of the employees. employees from PESO Resources Development Corporation (PESO) to perform temporary
and occasional services in its factory. This prompted respondent Goya, Inc. Employees
28

Union–FFW (Union) to request for a grievance conference on the ground that the contractual On appeal, the CA Eight Division denied the appeal and affirmed with modification the
workers do not belong to the categories of employees stipulated in the existing Collective Voluntary Arbitrators’ decision. While the CA agreed that the cash conversion is a fringe
Bargaining Agreement (CBA).The Union asserted that the hiring of contractual employees benefit, it does not necessarily mean that it is subject to the fringe benefit tax, as it explained
from PESO is not a management prerogative and in gross violation of the CBA tantamount to that Section 33 (A) of the National Internal Revenue Code (NIRC) of 1997 imposed a fringe
unfair labor practice (ULP). It noted that the contractual workers engaged have been benefit tax, effective January 1, 2000 and thereafter, on the grossed-up monetary value of
assigned to work in positions previously handled by regular workers and Union members, in fringe benefit furnished or granted to the employee (except rank-and-file employees) by the
effect violating Section 4, Article I of the CBA, which provides for three categories of employer (unless the fringe benefit is required by the nature of, or necessary to the trade,
employees in the Company. business or profession of the employer, or when the fringe benefit is for the convenience or
advantage of the employer). Since the gasoline allowance was mainly for the benefit of the
During the hearing on July 1, 2004, the Company and the Union manifested before Voluntary company, it is not subject to fringe benefit tax.
Arbitrator (VA) Bienvenido E. Laguesma that amicable settlement was no longer possible;
hence, they agreed to submit for resolution the solitary issue of "[w]hether or not the The company elevated the case to the Supreme Court. It assails the finding of both the
Company is guilty of unfair labor acts in engaging the services of PESO, a third party service arbitrators and the CA that the cash conversion of the unused portion of gasoline allowance
provider, under the existing CBA, laws, and jurisprudence." is a fringe benefit, not a part of compensation income. Even assuming the same is a fringe
benefit, the union has no cause of action for the refund of tax withheld and paid to the BIR.
On October 26, 2004, VA Laguesma dismissed the Union’s charge of ULP for being purely Citing Section 204 of the NIRC, the company contends that an action for the refund of an
speculative and for lacking in factual basis, but the Company was directed to observe and erroneous withholding and payment of taxes should be in the nature of a tax refund claim
comply with its commitment under the CBA. with the BIR. It further contends that when it withheld the income tax due from the cash
conversion of the unused gasoline allowance of the union members, it was simply acting as
ISSUE: WON the VA professed an order was not covered by the sole issue submitted for an agent of the government for the collection and payment of taxes due from the members.
voluntary arbitration
Issue: Whether or not the union has a cause of action for refund to tax withheld by the
HELD: No. The VA ruled on a matter that is covered by the sole issue submitted for voluntary company on the cash conversion of the unused portion of the gasoline allowance of its
arbitration. Resultantly, the CA did not commit serious error when it sustained the ruling that members.
the hiring of contractual employees from PESO was not in keeping with the intent and spirit of
the CBA. Indeed, the opinion of the VA is germane to, or, in the words of the CA, "interrelated Ruling: We partly grant the petition. The Voluntary Arbitrator has no jurisdiction to settle tax
and intertwined with," the sole issue submitted for resolution by the parties. matters

Generally, the arbitrator is expected to decide only those questions expressly delineated by The Labor Code vests the Voluntary Arbitrator original and exclusive jurisdiction to hear and
the submission agreement. Nevertheless, the arbitrator can assume that he has the decide all unresolved grievances arising from the interpretation or implementation of the
necessary power to make a final settlement since arbitration is the final resort for the Collective Bargaining Agreement and those arising from the interpretation or enforcement of
adjudication of disputes. The succinct reasoning enunciated by the CA in support of its company personnel policies[ LABOR CODE, Article 261.]. Upon agreement of the parties, the
holding, that the Voluntary Arbitrator in a labor controversy has jurisdiction to render the Voluntary Arbitrator shall also hear and decide all other labor disputes, including unfair labor
questioned arbitral awards, deserves our concurrence, thus: practices and bargaining deadlocks.1

In general, the arbitrator is expected to decide those questions expressly stated and limited in In short, the Voluntary Arbitrator’s jurisdiction is limited to labor disputes. Labor dispute
the submission agreement. However, since arbitration is the final resort for the adjudication of means “any controversy or matter concerning terms and conditions of employment or the
disputes, the arbitrator can assume that he has the power to make a final settlement. Thus, association or representation of persons in negotiating, fixing, maintaining, changing, or
assuming that the submission empowers the arbitrator to decide whether an employee was arranging the terms and conditions of employment, regardless of whether the disputants
discharged for just cause, the arbitrator in this instance can reasonably assume that his stand in the proximate relation of employer and employee.”2
powers extended beyond giving a yes-or-no answer and included the power to reinstate him
with or without back pay. The issues raised before the Panel of Voluntary Arbitrators are: (1) whether the cash
conversion of the gasoline allowance shall be subject to fringe benefit tax or the graduated
In one case, the Supreme Court stressed that "xxx the Voluntary Arbitrator had plenary income tax rate on compensation; and (2) whether the company wrongfully withheld income
jurisdiction and authority to interpret the agreement to arbitrate and to determine the scope of tax on the converted gas allowance.
his own authority subject only, in a proper case, to the certiorari jurisdiction of this Court. The
Arbitrator, as already indicated, viewed his authority as embracing not merely the The Voluntary Arbitrator has no competence to rule on the taxability of the gas allowance and
determination of the abstract question of whether or not a performance bonus was to be on the propriety of the withholding of tax. These issues are clearly tax matters, and do not
granted but also, in the affirmative case, the amount thereof. involve labor disputes. To be exact, they involve tax issues within a labor relations setting as
they pertain to questions of law on the application of Section 33 (A) of the NIRC. They do not
By the same token, the issue of regularization should be viewed as two-tiered issue. While require the application of the Labor Code or the interpretation of the MOA and/or company
the submission agreement mentioned only the determination of the date or regularization, law personnel policies. Furthermore, the company and the union cannot agree or compromise on
and jurisprudence give the voluntary arbitrator enough leeway of authority as well as the taxability of the gas allowance. Taxation is the State’s inherent power; its imposition
adequate prerogative to accomplish the reason for which the law on voluntary arbitration was cannot be subject to the will of the parties.
created – speedy labor justice. It bears stressing that the underlying reason why this case
arose is to settle, once and for all, the ultimate question of whether respondent employees Under paragraph 1, Section 4 of the NIRC, the CIR shall have the exclusive and original
are entitled to higher benefits. To require them to file another action for payment of such jurisdiction to interpret the provisions of the NIRC and other tax laws, subject to review by the
benefits would certainly undermine labor proceedings and contravene the constitutional Secretary of Finance. Consequently, if the company and/or the union desire/s to seek
mandate providing full protection to labor. clarification of these issues, it/they should have requested for a tax ruling3 from the Bureau of
Internal Revenue (BIR). Any revocation, modification or reversal of the CIR’s ruling shall not
Honda Cars vs Honda Cars Technical Specialist, G.R. No. 204142, November 19, 2014 be given retroactive application if the revocation, modification or reversal will be prejudicial to
Facts: Honda Cars Inc., and the company’s supervisors and technical specialists’ union, the taxpayers, except in the following cases:
Honda Cars Technical Specialists and Supervisory Union entered into a collective bargaining
agreement for the period April 1, 2006 to March 31, 2011. Prior to the execution of the CBA, (a) Where the taxpayer deliberately misstates or omits material facts from his return or any
the union members were receiving P3,300.00 a month as transportation allowance; on document required of him by the BIR;
September3, 2005, they entered into a Memorandum of Agreement converting the
transportation allowance into a monthly gasoline allowance at 125 liters each for official (b) Where the facts subsequently gathered by the BIR are materially different from the facts
business purposes and travel from home to office. Claiming that the gasoline allowance is on which the ruling is based; or
similar to the company policy for manages and assistant vice-presidents that “in the event the
gas allowance is not fully consumed, the gasoline not used may be converted to cash, (c) Where the taxpayer acted in bad faith.4
subject to whatever tax is applicable”, the company deducted from union members
withholding tax corresponding to the conversion to cash of their unused gasoline allowance, On the other hand, if the union disputes the withholding of tax and desires a refund of the
since the company considered it as part of compensation subject to income tax. The union withheld tax, it should have filed an administrative claim for refund with the CIR. Paragraph 2,
on the other hand claimed that the gasoline allowance is a “negotiated item” under Article XV Section 4 of the NIRC expressly vests the CIR original jurisdiction over refunds of internal
Section 15 of the new CBA on fringe benefits, thus it resulted to a grievance which was not revenue taxes, fees or other charges, penalties imposed in relation thereto, or other tax
resolved by the CBA grievance procedure. They submitted the issue to a panel of voluntary matters.
arbitrators as required by the CBA.
The union has no cause of action against the company
The Panel of Voluntary Arbitrators then rendered a decision declaring that the cash
conversion of the unused gasoline allowance is a fringe benefit subject to the fringe benefit Under the withholding tax system, the employer as the withholding agent acts as both the
tax, not to income tax, thus the deductions made by the company shall be considered as government and the taxpayer’s agent. Except in the case of a minimum wage earner, every
advances subject to refund in future remittances of withholding tax. employer has the duty to deduct and withhold upon the employee’s wages a tax determined
in accordance with the rules and regulations to be prescribed by the Secretary of Finance,
upon the CIR’s recommendation.5 As the Government’s agent, the employer collects tax and
29

serves as the payee by fiction of law.6 As the employee’s agent, the employer files the Pharmaceutical, Inc. vs. Hon. Secretary of Labor and Associated Labor Union (ALU) where
necessary income tax return and remits the tax to the Government.7 the Court declared:

Based on these considerations, we hold that the union has no cause of action against the In the present case, the Secretary was explicitly granted by Article 263(g) of the Labor Code
company. The company merely performed its statutory duty to withhold tax based on its the authority to assume jurisdiction over a labor dispute causing or likely to cause a strike or
interpretation of the NIRC, albeit that interpretation may later be found to be erroneous. The lockout in an industry indispensable to the national interest, and decide the same accordingly.
employer did not violate the employee’s right by the mere act of withholding the tax that may Necessarily, this authority to assume jurisdiction over the said labor dispute must include and
be due the government.8 extend to all questions and controversies arising therefrom, including cases over which the
labor arbiter has exclusive jurisdiction.
Moreover, the NIRC only holds the withholding agent personally liable for the tax arising from
the breach of his legal duty to withhold, as distinguished from his duty to pay tax. 9 Under Moreover, Article 217 of the Labor Code is not without, but contemplates, exceptions thereto.
Section 79 (B) of the NIRC, if the tax required to be deducted and withheld is not collected This is evident from the opening proviso therein reading ‘(e)xcept as otherwise provided
from the employer, the employer shall not be relieved from liability for any penalty or addition under this Code x x x.’ Plainly, Article 263(g) of the Labor Code was meant to make both the
to the unwithheld tax. Secretary (or the various regional directors) and the labor arbiters share jurisdiction, subject
to certain conditions. Otherwise, the Secretary would not be able to effectively and efficiently
Thus, if the BIR illegally or erroneously collected tax, the recourse of the taxpayer, and in dispose of the primary dispute. To hold the contrary may even lead to the absurd and
proper cases, the withholding agent, is against the BIR, and not against the withholding undesirable result wherein the Secretary and the labor arbiter concerned may have
agent10. The union’s cause of action for the refund or non-withholding of tax is against the diametrically opposed rulings. As we have said, ‘it is fundamental that a statute is to be read
taxing authority, and not against the employer. Section 229 of the NIRC provides: in a manner that would breathe life into it, rather than defeat it.

Sec.229. Recovery of Tax Erroneously or Illegally Collected. – No suit or proceeding shall be In fine, the issuance of the assailed orders is within the province of the Secretary as
maintained in any court for the recovery of any national internal revenue tax hereafter alleged authorized by Article 263(g) of the Labor Code and Article 217(a) and (5) of the same Code,
to have been erroneously or illegally assessed or collected, or of any penalty claimed to have taken conjointly and rationally construed to subserve the objective of the jurisdiction vested in
been collected without authority, or of any sum alleged to have been excessively or in any the Secretary.
manner wrongfully collected, until a claim for refund or credit has been duly filed with the
Commissioner; but such suit or proceeding may be maintained, whether or not such tax, NUWHRAIN-APL-IUF Dusit Hotel Nikko Chapter vs. CA, G.R. No. 163942, Nov. 11, 2008
penalty, or sum has been paid under protest or duress. Facts: Because of the collective bargaining deadlock, petitioner Union staged a strike against
the Hotel, herein private respondent. This strike was declared illegal by the SC.
Ace Navigation Co. Inc., vs Fernandez, G.R. No. 197309, October 10, 2012
It bears stressing at this point that we are upholding the jurisdiction of the voluntary arbitrator Issue: The effects of an illegal strike on employees.
or panel of voluntary arbitrators over the present dispute, not only because of the clear
language of the parties’ CBA on the matter; more importantly, we so uphold the voluntary Held: Regarding the Union officers and members’ liabilities for their participation in the illegal
arbitrator’s jurisdiction, in recognition of the State’s express preference for voluntary modes of picket and strike, Article 264(a), paragraph 3 of the Labor Code provides that “any union
dispute settlement, such as conciliation and voluntary arbitration as expressed in the officer who knowingly participates in an illegal strike and any worker or union officer who
Constitution, the law and the rules. It is settled that when the parties have validly agreed on a knowingly participates in the commission of illegal acts during a strike may be declared to
procedure for resolving grievances and to submit a dispute to voluntary arbitration then that have lost his employment status x x x.” The law makes a distinction between union officers
procedure should be strictly observed. and mere union members. Union officers may be validly terminated from employment for their
participation in an illegal strike, while union members have to participate in and commit illegal
Interphil Laboratories Employees Union vs. Interphil Laboratories, G.R. No. 142824, acts for them to lose their employment status. Thus, it is necessary for the company to
December 19, 2001 adduce proof of the participation of the striking employees in the commission of illegal acts
Facts: Petitioner is the sole and exclusive bargaining agent of the rank-and-file employees of during the strikes.
Respondent. They had a CBA.
Clearly, the 29 Union officers may be dismissed pursuant to Art. 264(a), par. 3 of the Labor
Prior to the expiration of the CBA, respondent company was approached by the petitioner, Code which imposes the penalty of dismissal on “any union officer who knowingly
through its officers. The Union inquired about the stand of the company regarding the participates in an illegal strike.” We, however, are of the opinion that there is room for
duration of the CBA which was set to expire in a few months. Salazar told the union officers leniency with respect to the Union members. It is pertinent to note that the Hotel was able to
that the matter could be best discussed during the formal negotiations which would start prove before the NLRC that the strikers blocked the ingress to and egress from the Hotel. But
soon. it is quite apparent that the Hotel failed to specifically point out the participation of each of the
Union members in the commission of illegal acts during the picket and the strike. For this
All the rank-and-file employees of the company refused to follow their regular two-shift work lapse in judgment or diligence, we are constrained to reinstate the 61 Union members.
schedule. The employees stopped working and left their workplace without sealing the
containers and securing the raw materials they were working on. Further, we held in one case that union members who participated in an illegal strike but were
not identified to have committed illegal acts are entitled to be reinstated to their former
To minimize the damage the overtime boycott was causing the company, Salazar positions but without backwages.
immediately asked for a meeting with the union officers. In the meeting, Enrico Gonzales, a
union director, told Salazar that the employees would only return to their normal work Club Filipino vs. Bautista, G.R. No. 168406, July 13, 2009
schedule if the company would agree to their demands as to the effectivity and duration of Facts: Petitioner and the union had a CBA which expired on May 31, 2000. Within the
the new CBA. Salazar again told the union officers that the matter could be better discussed freedom period, the union made several demands for negotiation but the company replied
during the formal renegotiations of the CBA. Since the union was apparently unsatisfied with that it could not muster a quorum, thus no CBA negotiations could be held. In order to compel
the answer of the company, the the company to negotiate, union filed a request for preventive mediation with NCMB but
overtime boycott continued. In addition, the employees started to engage in a work slowdown again failed. On April 2001, a notice of strike was filed by the union and thereafter, a strike
campaign during the time they were working, thus substantially delaying the production of the was held. Petitioner filed before the NLRC a petition to declare the strike illegal. The LA, in its
company. decision, declared that the strike is illegal. On appeal, the NLRC decision is affirmed the LA
decision. Upon elevation to CA, the court set aside the ruling of the LA and NLRC as far as
Respondent company filed with the National NLRC a petition to declare illegal petitioner other respondent but dismissed the other respondent. Hence, this petition.
union’s “overtime boycott” and “work slowdown” which, according to respondent company,
amounted to illegal strike. It also filed with Office Secretary of Labor a petition for assumption Issue: Whether the strike staged by respondent is legal.
of jurisdiction. Secretary of Labor Nieves Confesor issued an assumption order over the labor
dispute. Ruling: The court ruled in affirmative. It is undisputed that the notice of strike was filed by the
union without attaching the counter-proposal of the company. In cases of bargaining
Labor Arbiter Caday submitted his recommendation to the then Secretary of Labor Leonardo deadlocks, the notice shall, as far as practicable, further state the unresolved issues in the
A. Quisumbing. Then Secretary Quisumbing approved and adopted the report in his Order, bargaining negotiations and be accompanied by the written proposals of the union, the
finding illegal strike on the part of petitioner Union. counter-proposals of the employer and the proof of a request for conference to settle
differences. In cases of unfair labor practices, the notice shall, as far as practicable, state the
Issue: WON the Labor Secretary has jurisdiction to rule over an illegal strike. acts complained of, and efforts taken to resolve the dispute amicable. Any notice which does
not conform with the requirements of this and the foregoing section shall be deemed as not
Held: On the matter of the authority and jurisdiction of the Secretary of Labor and having been filed and the party concerned shall be so informed by the regional branch of the
Employment to rule on the illegal strike committed by petitioner union, it cannot be denied Board. The union cannot be faulted for its omission. The union could not have attached the
that the issues of “overtime boycott” and “work slowdown” amounting to illegal strike before counter- proposal of the company in the notice of strike it submitted to the NCMB as there
Labor Arbiter Caday are intertwined with the labor dispute before the Labor Secretary. was no such counter- proposal. The union filed a notice of strike, after several request for
negotiation proved futile. It was only after two weeks, when the company formally responded
The appellate court also correctly held that the question of the Secretary of Labor and to the union by submitting the first part of its counter-proposal. Nowhere in the ruling of the
Employment’s jurisdiction over labor-related disputes was already settled in International LA can we find any discussion of how respondents, as union officers, knowingly participated
30

in the alleged illegal strike. Thus, even assuming Arguendo that the strike was illegal, their Held: In Philippine Association of Free Labor Unions (PAFLU) v. Cloribel, this Court, through
automatic dismissal had no basis. Justice J.B.L. Reyes, stated the “innocent bystander” rule as follows:

Santa Rosa CocaCola Plant Ph. Union v CocaCola Bottlers, GR No. 164302, Jan 24, ‘07 The right to picket as a means of communicating the facts of a labor dispute is a phase of the
Facts: The Sta. Rosa Coca-Cola Plant Employees Union (Union) is the sole and exclusive freedom of speech guaranteed by the constitution. If peacefully carried out, it cannot be
bargaining representative of the regular daily paid workers and the monthly paid non- curtailed even in the absence of employer-employee relationship.
commission-earning employees of the Coca-Cola Bottlers Philippines, Inc. (Company) in its The right is, however, not an absolute one. While peaceful picketing is entitled to protection
Sta. Rosa, Laguna plant. as an exercise of free speech, we believe the courts are not without power to confine or
localize the sphere of communication or the demonstration to the parties to the labor dispute,
Upon the expiration of the CBA, the Union informed the Company of its desire to renegotiate including those with related interest, and to insulate establishments or persons with no
its terms. The CBA meetings commenced on July 26, 1999, where the Union and the industrial connection or having interest totally foreign to the context of the dispute. Thus the
Company discussed the ground rules of the negotiations. The Union insisted that right may be regulated at the instance of third parties or “innocent bystanders” if it appears
representatives from the Alyansa ng mga Unyon sa Coca-Cola be allowed to sit down as that the inevitable result of its exercise is to create an impression that a labor dispute with
observers in the CBA meetings. The Union officers and members also insisted that their which they have no connection or interest exists between them and the picketing union or
wages be based on their work shift rates. For its part, the Company was of the view that the constitute an invasion of their rights.
members of the Alyansa were not members of the bargaining unit. The Alyansa was a mere
aggregate of employees of the Company in its various plants; and is not a registered labor Thus, an “innocent bystander,” who seeks to enjoin a labor strike, must satisfy the court it is
organization. Thus, an impasse ensued. entirely different from, without any connection whatsoever to, either party to the dispute and,
therefore, its interests are totally foreign to the context thereof.
On August 30, 1999, the Union, its officers, directors and six shop stewards filed a “Notice of
Strike” with the NCMB. In the case at bar, petitioner cannot be said not to have such connection to the dispute. We
find that the “negotiation, contract of sale, and the post transaction” between Philtread, as
The Union decided to participate in a mass action organized by the Alyansa in front of the vendor, and Siam Tyre, as vendee, reveals a legal relation between them which, in the
Company’s premises. Thus, the Union officers and members held a picket along the front interest of petitioner, we cannot ignore. To be sure, the transaction between Philtread and
perimeter of the plant on September 21, 1999. As a result, all of the 14 personnel of the Siam Tyre, was not a simple sale whereby Philtread ceased to have any proprietary rights
Engineering Section of the Company did not report for work, and 71 production personnel over its sold assets. On the contrary, Philtread remains as 20% owner of private respondent
were also absent. As a result, only one of the three bottling lines operated during the day and 60% owner of Sucat Land Corporation which was likewise incorporated in accordance
shift. All the three lines were operated during the night shift with cumulative downtime of five with the terms of the Memorandum of Agreement with Siam Tyre, and which now owns the
(5) hours due to lack of manning, complement and skills requirement. The volume of land were subject plant is located. This, together with the fact that private respondent uses
production for the day was short by 60,000 physical cases versus budget. the same plant or factory; similar or substantially the same working conditions; same
machinery, tools, and equipment; and manufacture the same products as Philtread, lead us
On October 13, 1999, the Company filed a “Petition to Declare Strike Illegal” to safely conclude that private respondent’s personality is so closely linked to Philtread as to
bar its entitlement to an injunctive writ.
Issue: WON the strike, dubbed by petitioner as picketing, is illegal.
Univ. of the Immaculate Conception vs. Sec. of Labor, G.R. No. 178085, Sept 14, 2015
Held: Article 212(o) of the Labor Code defines strike as a temporary stoppage of work by the
concerted action of employees as a result of an industrial or labor dispute. In Bangalisan v. PLDT vs. Manggagawa ng Komunikasyon sa Pilipinas, G.R. No. 161783, July 13, 2005
CA, the Court ruled that “the fact that the conventional term ‘strike’ was not used by the FACTS: Petitioner Philippine Long Distance Telephone Co., Inc. (PLDT) is a domestic
striking employees to describe their common course of action is inconsequential, since the corporation engaged in the telecommunications business. Private respondent Manggagawa
substance of the situation, and not its appearance, will be deemed to be controlling.” ng Komunikasyon sa Pilipinas (MKP) is a labor union of rank and file employees in PLDT.

Picketing involves merely the marching to and fro at the premises of the employer, usually The members of respondent union learned that a redundancy program would be
accompanied by the display of placards and other signs making known the facts involved in a implemented by the petitioner. Thereupon it filed a Notice of Strike with the National
labor dispute. As applied to a labor dispute, to picket means the stationing of one or more Conciliation and Mediation Board (NCMB) on 04 November 2002. The Notice fundamentally
persons to observe and attempt to observe. The purpose of pickets is said to be a means of contained the following:
peaceable persuasion.
UNFAIR LABOR PRACTICES, to wit:
The basic elements of a strike are present in this case. They marched to and fro in front of
the company’s premises during working hours. Thus, petitioners engaged in a concerted 1. PLDT’s abolition of the Provisioning Support Division, in violation of the duty to
activity which already affected the company’s operations. The mass concerted activity bargain collectively with MKP in good faith.
constituted a strike.
2. PLDT’s unreasonable refusal to honor its commitment before this Honorable
For a strike to be valid, the following procedural requisites provided by Art 263 of the Labor Office that it will provide MKP its comprehensive plan/s with respect to personnel
Code must be observed: (a) a notice of strike filed with the DOLE 30 days before the downsizing / reorganization and closure of exchanges. Such refusal violates its duty to
intended date thereof, or 15 days in case of unfair labor practice; (b) strike vote approved by bargain collectively with MKP in good faith.
a majority of the total union membership in the bargaining unit concerned obtained by secret
ballot in a meeting called for that purpose, (c) notice given to the DOLE of the results of the 3. PLDT’s continued hiring of “contractual”, “temporary”, “project” and “casual”
voting at least seven days before the intended strike. These requirements are mandatory and employees for regular jobs performed by union members, resulting in the decimation of the
the failure of a union to comply therewith renders the strike illegal. It is clear in this case that union membership and in the denial of the right to self-organization to the concerned
petitioners totally ignored the statutory requirements and embarked on their illegal strike. employees.

MSF Tire & Rubber, Inc., vs CA, G.R. No. 128632, August 5, 1999 4. PLDT’s gross violation of the legal and CBA provisions on overtime work and
Facts: Respondent Union filed a notice of strike in the NCMB charging (Phildtread) with unfair compensation.
labor practice. Thereafter, they picketed and assembled outside the gate of Philtread’s plant.
Philtread, on the other hand, filed a notice of lockout. Subsequently, the Secretary of Labor 5. PLDT’s gross violation of the CBA provisions on promotions and job grade re-
assumed jurisdiction over the labor dispute and certified it for compulsory arbitration. evaluation or reclassification.

During the pendency of the labor dispute, Philtread entered into a Memorandum of On 11 November 2002, another Notice of Strike was filed by the private respondent, which
Agreement with Siam Tyre whereby its plant and equipment would be sold to a new contained the following: UNFAIR LABOR PRACTICES, to wit: PLDT’s alleged restructuring of
company, herein petitioner, 80% of which would be owned by Siam Tyre and 20% by its GMM Operation Services.
Philtread, while the land on which the plant was located would be sold to another company,
60% of which would be owned by Philtread and 40% by Siam Tyre. A number of conciliation meetings, conducted by the NCMB, National Capital Region, were
held between the parties. However, these efforts proved futile.
Petitioner then asked respondent Union to desist from picketing outside its plant. As the
respondent Union refused petitioner’s request, petitioner filed a complaint for injunction with On 23 December 2002, the private respondent staged a strike. On 31 December 2002, three
damages before the RTC. Respondent Union moved to dismiss the complaint alleging lack of hundred eighty three (383) union members were terminated from service pursuant to PLDT’s
jurisdiction on the part of the trial court. redundancy program.

Petitioner asserts that its status as an “innocent bystander” with respect to the labor dispute On 02 January 2003, the Secretary, Patricia Sto. Tomas, issued an Order[4] in NCMB- NCR-
between Philtread and the Union entitles it to a writ of injunction from the civil courts. NS-11-405-02 and NCMB-NCR-NS-11-412-02. Portions of the Order are reproduced
hereunder:
Issue: WON petitioner has shown a clear legal right to the issuance of a writ of injunction
under the “innocent bystander” rule. xxx Accordingly, the strike staged by the Union is hereby enjoined. All striking workers are
hereby directed to return to work within twenty four (24) hours from receipt of this Order,
31

except those who were terminated due to redundancy. The employer is hereby enjoined to
accept the striking workers under the same terms and conditions prevailing prior to the strike. The foregoing notwithstanding, the President of the Philippines shall not be precluded from
The parties are likewise directed to cease and desist from committing any act that might determining the industries that, in his opinion, are indispensable to the national interest, and
worsen the situation. xxx from intervening at any time and assuming jurisdiction over any such labor dispute in order to
settle or terminate the same.xxx
ISSUE: WHETHER THE SUBJECT ORDERS OF THE SECRETARY OF THE DOLE
EXCLUDING FROM THE RETURN-TO-WORK ORDER THE WORKERS DISMISSED DUE Phimco vs. Brillantes, G.R. No. 120751, March 17, 1999
TO THE REDUNDANCY PROGRAM OF PETITIONER, ARE VALID OR NOT. FACTS: On March 9, 1995, the private respondent, Phimco Industries Labor Association
(PILA), duly certified collective bargaining representative of the daily paid workers of the
RULING: Valid. . . . Assumption of jurisdiction over a labor dispute, or as in this case the petitioner PHIMCO filed a notice of strike with the NCMB against PHIMCO, a corporation
certification of the same to the NLRC for compulsory arbitration, always co-exists with an engaged in the production of matches, after a deadlock in the collective bargaining and
order for workers to return to work immediately and for employers to readmit all workers negotiation. Parties failed to resolve their differences PILA (during the conciliation
under the same terms and conditions prevailing before the strike or lockout. conferences), composed of 352 members, staged a strike.

Time and again, this Court has held that when an official bypasses the law on the asserted PHIMCO sent notice of termination to some 47 workers including several union officers.
ground of attaining a laudable objective, the same will not be maintained if the intendment or
purpose of the law would be defeated. Secretary Brillantes assumed jurisdiction over the labor dispute; issued a return-to-work
order.
One last piece. Records would show that the strike occurred on 23 December 2002. Article
263(g) directs that the employer must readmit all workers under the same terms and Hence, petitioner files this petition.
conditions prevailing before the strike. Since the strike was held on the aforementioned date,
then the condition prevailing before it, which was the condition present on 22 December ISSUE: whether or not the public respondent acted with grave abuse of discretion amounting
2002, must be maintained. to lack or excess of jurisdiction in assuming jurisdiction over subject labor dispute.

Undoubtedly, on 22 December 2002, the members of the private respondent who were HELD: YES, the petition is impressed with merit.
dismissed due to alleged redundancy were still employed by the petitioner and holding their
respective positions. This is the status quo that must be maintained. Art. 263, paragraph (g) of the Labor Code, provides:

Capitol Medical Center vs. Trajano, G.R. No. 155690, June 30, 2005 (g) When, in his opinion, there exist a labor dispute causing or likely to cause a strike or
FACTS: Petitioner is a hospital with address at Panay Avenue corner Scout Magbanua lockout in an industry indispensable to the national interest, the Secretary of Labor and
Street, Quezon City. Upon the other hand, Respondent is a duly registered labor union acting Employment may assume jurisdiction over the dispute and decide it or certify the same to the
as the certified collective bargaining agent of the rank-and-file employees of petitioner Commission for compulsory arbitration . . .
hospital.
The Labor Code vests in the Secretary of Labor the discretion to determine what industries
Respondent sent petitioner a letter requesting a negotiation of their Collective Bargaining are indispensable to the national interest. Accordingly, upon the determination by the
Agreement (CBA). Secretary of Labor that such industry is indispensable to the national interest, he will assume
jurisdiction over the labor dispute in the said industry. 8 This power, however, is not without
Petitioner, however, challenged the union’s legitimacy and refused to bargain with any limitation.
respondent. Subsequently petitioner filed with the (BLR), Department of Labor and
Employment, a petition for cancellation of respondent’s certificate of registration. It stressed in the case of Free telephone Workers Union vs. Honorable Minister of Labor and
Employment, et al., 10 the limitation set by the legislature on the power of the Secretary of
For its part, respondent filed with the (NCMB), National Capital Region, a notice of strike. Labor to assume jurisdiction over a labor dispute, thus:
Respondent alleged that petitioner’s refusal to bargain constitutes unfair labor practice.
Despite several conferences and efforts of the designated conciliator-mediator, the parties xxx cannot be any clearer, the coverage being limited to “strikes or lockouts adversely
failed to reach an amicable settlement. affecting the national interest. 11

Respondent staged a strike. In this case at bar, however, the very admission by the public respondent draws the labor
dispute in question out of the ambit of the Secretary’s prerogative, to wit.
Former Labor Secretary Leonardo A. Quisumbing, now Associate Justice of this Court,
issued an Order assuming jurisdiction over the labor dispute and ordering all striking workers While the case at bar appears on its face not to fall within the strict categorization of cases
to return to work and the management to resume normal operations, thus: imbued with “national interest”, this office believes that the obtaining circumstances warrant
the exercise of the powers under Article 263 (g) of the Labor Code, as amended. 12
xxx all striking workers are directed to return to work within twenty-four (24) hours from
the receipt of this Order and the management to resume normal operations and accept The private respondent did not even make any effort to touch on the indispensability of the
back all striking workers under the same terms and conditions prevailing before the match factory to the national interest. It must have been aware that a match factory, though
strike. Further, parties are directed to cease and desist from committing any act that may of value, can scarcely be considered as an industry “indispensable to the national interest” as
exacerbate the situation. it cannot be in the same category as “generation and distribution of energy, or those
undertaken by banks, hospitals, and export-oriented industries.” 13 Yet, the public
Moreover, parties are hereby directed to submit within 10 days from receipt of this Order respondent assumed jurisdiction thereover.
proposals and counter-proposals leading to the conclusion of the collective bargaining
agreement in compliance with aforementioned Resolution of the Office as affirmed by the To uphold the action of the public respondent under the premises would be stretching too far
Supreme Court. xxx the power of the Secretary of Labor as every case of a strike or lockout where there are
ISSUE: Whether or not Secretary of Labor cannot exercise his powers under Article 263 (g) inconveniences in the community, or work disruptions in an industry though not indispensable
of the Labor Code without observing the requirements of due process. to the national interest, would then come within the Secretary’s power. It would be practically
allowing the Secretary of Labor to intervene in any Labor dispute at his pleasure.
RULING: The discretion to assume jurisdiction may be exercised by the Secretary of Labor
and Employment without the necessity of prior notice or hearing given to any of the parties. This is precisely why the law sets and defines the standard: even in the exercise of his power
The rationale for his primary assumption of jurisdiction can justifiably rest on his own of compulsory arbitration under Article 263 (g) of the Labor Code, the Secretary must follow
consideration of the exigency of the situation in relation to the national interests. the law.

xxx In labor disputes adversely affecting the continued operation of such hospitals, clinics or FEU-NRMF vs FEU-NRMF Employees Association, G.R. No. 168362, October 12, 2006
medical institutions, it shall be the duty of the striking union or locking-out employer to provide FACTS: In 1994, petitioner FEU-NRMF (a medical institution organized and existing under
and maintain an effective skeletal workforce of medical and other health personnel, whose the Philippine laws), and respondent union (a legitimate labor organization and is the duly
movement and services shall be unhampered and unrestricted, as are necessary to insure recognized representative of the rank and file employees of petitioner), entered into a CBA
the proper and adequate protection of the life and health of its patients, most especially that will expire on 30 April 1996. In view of the forthcoming expiry, respondent union sent a
emergency cases, for the duration of the strike or lockout. In such cases, therefore, the letter-proposal to petitioner FEU-NRMF stating their economic and non-economic proposals
Secretary of Labor and Employment is mandated to immediately assume, within twenty-four for the negotiation of the new CBA.
(24) hours from knowledge of the occurrence of such a strike or lockout, jurisdiction over the
same or certify it to the Commission for compulsory arbitration. For this purpose, the Petitioner FEU-NRMF rejected respondent union’s demands and proposed to maintain the
contending parties are strictly enjoined to comply with such orders, prohibitions and/or same provisions of the old CBA reasoning that due to financial constraints, it cannot afford to
injunctions as are issued by the Secretary of Labor and Employment or the Commission, accede to a number of their demands. In an effort to arrive at a compromise, subsequent
under pain of immediate disciplinary action, including dismissal or loss of employment status conciliation proceedings were conducted before the NCMB, but the negotiation failed.
or payment by the locking-out employer of backwages, damages and other affirmative relief,
even criminal prosecution against either or both of them.
32

Respondent union filed a Notice of Strike before NCMB on the ground of bargaining In the case at bar, presumption of receipt of the copies of the Assumption of Jurisdiction
deadlock. Union staged a strike. Order AJO could not be taken for granted considering the adverse effect in case the parties
failed to heed to the injunction directed by such Order. Defiance of the assumption and
Petitioner FEU-NRMF filed a Petition for the Assumption of Jurisdiction or for Certification of return-to-work orders of the Secretary of Labor after he has assumed jurisdiction is a valid
Labor Dispute with the NLRC, underscoring the fact that it is a medical institution engaged in ground for the loss of employment status of any striking union officer or member.
the business of providing health care for its patients. Secretary of Labor granted the petition Employment is a property right of which one cannot be deprived of without due process. Due
and an Order assuming jurisdiction over the labor dispute was issued, thereby prohibiting any process here would demand that the respondent union be properly notified of the Assumption
strike or lockout and enjoining the parties from committing any acts which may exacerbate of Jurisdiction Order of the Secretary of Labor enjoining the strike and requiring its members
the situation. to return to work. Thus, there must be a clear and unmistakable proof that the requirements
prescribed by the Rules in the manner of effecting personal or substituted service had been
On September 6, 1996, Francisco Escuadra, the NLRC process server, certified that, on faithfully complied with. Merely posting copies of the AJO does not satisfy the rigid
September 5, 1996 at around 4:00 P.M., he attempted to serve a copy of the Assumption of requirement for proper service outlined by the above stated rules. Needless to say, the
Jurisdiction Order (AJO) to the union officers but since no one was around at the strike area, manner of service made by the process server was invalid and irregular. Respondent union
he just posted copies of the said Order at several conspicuous places within the premises of could not therefore be adjudged to have defied the said Order since it was not properly
the hospital. apprised thereof. Accordingly, the strike conducted by the respondent union was valid under
the circumstances.
Striking employees continued holding a strike until 12 September 1996, claiming that they
had no knowledge that the Secretary of Labor already assumed jurisdiction over the pending Union of Filipro Employees vs. NLRC, G.R. No. 91025, December 19, 1990
labor dispute as they were not able to receive a copy of the AJO. FACTS: On June 22, 1988, the petitioner Union of the Filipro Employees, the sole and
exclusive bargaining agent of all rank-and-file employees of Nestle Philippines, (private
The Secretary of Labor issued another Order directing all the striking employees to return to respondent) filed a Notice of Strike at the DOLE raising the issues of CBA deadlock and
work and the petitioner FEU-NRMF to accept them under the same terms and conditions unfair labor practice. Private respondent assailed the legal personality of the proponents of
prevailing before the strike. A Return to Work Agreement was executed by the disputing the said notice of strike to represent the Nestle employees, before the NCMB. This
parties. Subsequently, petitioner FEU-NRMF filed a case before the NLRC, contending that notwithstanding, the NCMB proceeded to invite the parties to attend the conciliation meetings
respondent union staged the strike in defiance of the AJO, hence, it was illegal. LA declared and to which private respondent failed to attend contending that it will deal only with a
the strike illegal and allowed dismissal of union officers for conducting the strike in defiance of negotiating panel duly constituted and mandated in accordance with the UFE Constitution
the AJO. Respondent union filed an Appeal before the NLRC. NLRC affirmed in toto the and By-laws. Thereafter, Company terminated from employment all UFE Union officers, and
Decision of the LA. Respondent union filed MR, it was denied. Respondent union brought a all the members of the negotiating panel for instigating and knowingly participating in a strike
Petition for Certiorari before CA. CA granted the Petition and reversed the Resolutions of staged at the Makati, Alabang, Cabuyao and Cagayan de Oro on September 11, 1987
NLRC. Petitioner filed MR but it was denied. Hence this petition. without any notice of strike filed and a strike vote obtained for the purpose. The union filed a
complaint for illegal dismissal. LA upheld the validity of the dismissal; NLRC en banc
ISSUE: Whether the service of the AJO was validly effected by the process server so as to affirmed. Subsequently, company concluded separate CBAs with the general membership of
bind the respondent union and hold them liable for the acts committed subsequent to the the union at Cebu/Davao and Cagayan de Oro units; Assailing the validity of these
issuance of the said Order. agreements, the union filed a case of ULP against the company with the NLRC-NCR
Arbitration Branch Efforts to resolve the dispute amicably were taken by the NCMB but
RULING: No. The process server resorted to posting the Order when personal service was yielded negative result. Petitioner filed a motion asking the Secretary of Labor to assume
rendered impossible since the striking employees were not present at the strike area. This jurisdiction over the dispute of deadlock in collective bargaining between the parties. On
mode of service, however, is not sanctioned by either the NLRC Revised Rules of Procedure October 28, 1988, Labor Secretary Franklin Drilon “certified” to the NLRC the said dispute
or the Revised Rules of Court. between the UFE and Nestle, Philippines.. which reads as follows: xxx “The NLRC is further
directed to call all the parties immediately and resolve the CBA deadlock within twenty (20)
The pertinent provisions of the NLRC Revised Rules of Procedure read: days from submission of the case for resolution.” Second Division of the NLRC promulgated
a resolution granting wage increase and other benefits to Nestle’s employees, ruling on non-
“Section 6. Service of Notices and Resolutions. economic issues, as well as absolving the private respondent of the Unfair Labor Practice
(a) Notices or summons and copies of orders, shall be served on the parties to the case charge. Petitioner finds said resolution to be inadequate and accordingly, does not agree
personally by the Bailiff or duly authorized public officer within 3 days from receipt thereof or therewith. It filed a motion for reconsideration, denied. Hence, this petition.
by registered mail; Provided that in special circumstances, service of summons may be
effected in accordance with the pertinent provisions of the Rules of Court; Provided further, ISSUE: WON WHETHER OR NOT THE SECOND DIVISION OF THE NLRC ACTED
that in cases of decisions and final awards, copies thereof shall be served on both parties WITHOUT JURISDICTION IN RENDERING THE ASSAILED RESOLUTION, THE SAME
and their counsel or representative by registered mail; Provided further, that in cases where a BEING RENDERED ONLY BY A DIVISION OF THE PUBLIC RESPONDENT AND NOT BY
party to a case or his counsel on record personally seeks service of the decision upon inquiry EN BANC;
thereon, service to said party shall be deemed effected upon actual receipt thereof; Provided
finally, that where parties are so numerous, service shall be made on counsel and upon such HELD: This case was certified on October 28, 1988 when existing rules prescribed that, it is
number of complainants, as may be practicable, which shall be considered substantial incumbent upon the Commission en banc to decide or resolve a certified dispute. However,
compliance with Article 224(a) of the Labor Code, as amended.” R.A. 6715 took effect during the pendency of this case. Aside from vesting upon each division
the power to adjudicate cases filed before the Commission, said Act further provides that the
An Order issued by the Secretary of Labor assuming jurisdiction over the labor dispute is not divisions of the Commission shall have exclusive appellate jurisdiction over cases within their
a final judgment for it does not dispose of the labor dispute with finality. Consequently, the respective territorial jurisdiction. Section 5 of RA 6715 provides as follows: xxxx The
rule on service of summons and orders, and not the proviso on service of decisions and final Commission may sit en banc or in five (5) divisions, each composed of three (3) members.
awards, governs the service of the Assumption of Jurisdiction Order. The Commission shall sit en banc only for purposes of promulgating rules and regulations
governing the hearing and disposition of cases before any of its divisions and regional
Under the NLRC Revised Rules of Procedure, service of copies of orders should be made by branches and formulating policies affecting its administration and operations. The
the process server either personally or through registered mail. However, due to the urgent Commission shall exercise its adjudicatory and all other powers, functions and duties through
nature of the AJO and the public policy underlying the injunction carried by the issuance of its divisions. xxxx In view of the enactment of Republic Act 6715, the aforementioned rules
the said Order, service of copies of the same should be made in the most expeditious and requiring the Commission en banc to decide or resolve a certified dispute have accordingly
effective manner, without any delay, ensuring its immediate receipt by the intended parties as been repealed. Confirmed in Administrative Order No. 36 (Series of 1989) promulgated by
may be warranted under the circumstances. Thus, personal service is the proper mode of the Secretary under his delegated rule-making power. Moreover, it is to be emphasized and it
serving the AJO. is a matter of judicial notice that since the effectivity of R.A. 6715, many cases have already
been decided by the 5 divisions of the NLRC. We find no legal justification in entertaining
Personal service effectively ensures that the notice desired under the constitutional petitioner’s claim considering that the clear intent of the amendatory provision is to expedite
requirement of due process is accomplished. If, however, efforts to find the party concerned the disposition of labor cases filed before the Commission. To rule otherwise would not be
personally would make prompt service impossible, service may be completed by substituted congruous to the proper administration of justice. ACCORDINGLY, PREMISES
service, that is, by leaving a copy, between the hours of eight in the morning and six in the CONSIDERED, the petition is DISMISSED. The Resolutions of the NLRC, dated June 5,
evening, at the party’s or counsel’s residence, if known, with a person of sufficient age and 1989 and August 8, 1989 are AFFIRMED, except insofar as the ruling absolving the private
discretion then residing therein (RULE 12 of Rev Rules of Court). respondent of unfair labor practice which is declared SET ASIDE.

Substituted service derogates the regular method of personal service. It is therefore required OWWA vs. Chavez, G.R. No. 169802, June 8, 2007
that statutory restrictions for effecting substituted service must be strictly, faithfully and fully FACTS: OWWA traces its beginnings to 1 May 1977, when the Welfare and Training Fund for
observed. Failure to comply with this rule renders absolutely void the substituted service Overseas main objective, inter alia, of providing social and welfare services to OFW,
along with the proceedings taken thereafter. The underlying principle of this rigid requirement including insurance coverage, social work, legal and placement assistance, cultural and
is that the person, to whom the orders, notices or summons are addressed, is made to remittances services, and the like.
answer for the consequences of the suit even though notice of such action is made, not upon
the party concerned, but upon another whom the law could only presume would notify such On 1 May 1980, PD 1694 was signed into law, formalizing the operations of a comprehensive
party of the pending proceedings. Welfare Fund (Welfund), as authorized and created under Letter of Instructions No. 537. On
16 January 1981, PD 1809 was promulgated, amending certain provisions of PD 1694.
33

Subsequently, EO 126 was passed renaming the Welfare Fund as the OWWA. On January The UNION it filed a notice of strike on the grounds of bargaining deadlock and ULP. During
9, 2004, as there was yet no formal OWWA structure duly approved by the DBM and CSC, the thirty (30) day cooling-off period, two union members were dismissed by petitioner.
the OWWA Board of Trustees passed Resolution No. 001, Series of 2004, depicting the Consequently, the UNION went on strike.
organizational structure and staffing pattern of the OWWA.
On January 23, 1995, the then Secretary of Labor, Ma. Nieves R. Confessor, issued an Order
On March 24, 2004, DBM Secretary Emilia T. Boncodin approved the organizational structure assuming jurisdiction over the labor dispute.
and staffing pattern of the OWWA. In her approval thereof, she stated that the total funding
requirements for the revised organizational structure shall be P107,546,379 for 400 positions. On March 10, 1995, the UNION filed another notice of strike, this time citing as a reason the
On May 31, 2004, an advisory was given to the officials and employees of the OWWA that UNIVERSITY’s termination of the individual respondents. The UNION alleged that the
the DBM had recently approved OWWA’s also placement staffing pattern. UNIVERSITY’s act of terminating the individual respondents is in violation of the Order of the
Secretary of Labor.
On June 3, 2004, DOLE Secretary Sto. Tomas issued Administrative Order No. 171 creating
a Placement Committee to evaluate qualifications of employees, and to recommend their On March 28, 1995, the Secretary of Labor issued another Order reiterating the directives
appropriate placement in the new organizational chart, functional statements and staffing contained in the January 23, 1995 Order. Hence, the UNIVERSITY was directed to reinstate
pattern of the OWWA. the individual respondents under the same terms and conditions prevailing prior to the labor
dispute.
On June 18, 2004, DOLE Acting Secretary Imson issued Administrative Order No. 186,
Series of 2004, prescribing the guidelines on the placement of personnel in the new staffing The UNIVERSITY filed a MR. In the Order dated August 18, 1995, then Acting Secretary
pattern of the OWWA. Herein respondents filed a complaint to nullify the organizational Jose S. Brilliantes denied the MR, but modified the two previous Orders by adding:
structure with Prayer for the Issuance of a Writ of Preliminary Injunction against petitioner
OWWA and its Board of Trustees. According to the respondents, the resulting decrease in Anent the Union’s Motion, we find that superseding circumstances would not warrant the
the number of employees due to Organizational Structure will result in the constructive physical reinstatement of the twelve (12) terminated employees.
dismissal of at least 110 employees. Meanwhile, the deployment of the regular central office
personnel to the regional offices will displace the said employees, as well as their families. Hence, they are hereby ordered placed under payroll reinstatement until the validity of their
Respondents challenged the validity of the new organizational structure of the OWWA. In termination is finally resolved.
fine, they contended that the same is null and void; hence, its implementation should be
prohibited. Issue: WON payroll reinstatement, instead of actual reinstatement, is proper.

RTC granted respondents’ prayer for a writ of preliminary injunction. Petitioner, thru OSG and Held: With respect to the Secretary’s Order allowing payroll reinstatement instead of actual
the RTC to restrain, for the meantime, the implementation of OWWA’s reorganization to reinstatement for the individual respondents herein, an amendment to the previous Orders
prevent injury until after the main case is heard and decided. Petitioner, thru OSG filed the issued by her office, the same is usually not allowed. Article 263(g) of the Labor Code
instant petition. aforementioned states that all workers must immediately return to work and all employers
must readmit all of them under the same terms and conditions prevailing before the strike or
lockout. The phrase “under the same terms and conditions” makes it clear that the norm is
ISSUE: Whether CA erred in affirming the RTC in its grant of the assailed writ of preliminary
actual reinstatement. This is consistent with the idea that any work stoppage or slowdown in
injunction.
that particular industry can be detrimental to the national interest.
RULING: No. Section 1, Rule 58 of the Rules of Court, defines a preliminary injunction as an
In ordering payroll reinstatement in lieu of actual reinstatement, then Acting Secretary of
order granted at any stage of an action prior to the judgment or final order requiring a party or
Labor Jose S. Brillantes said:
a court, an agency or a person to refrain from a particular act or acts. It persists until it is
dissolved or until the termination of the action without the court issuing a final injunction. To
Anent the Union’s Motion, we find that superseding circumstances would not warrant the
be entitled to an injunctive writ, and an urgent and paramount necessity for the writ to prevent
physical reinstatement of the twelve (12) terminated employees. Hence, they are hereby
serious damage. A writ of evidence required to justify the issuance of a writ of preliminary
ordered placed under payroll reinstatement until the validity of their termination is finally
injunction in the hearing thereon need not be conclusive or complete.
resolved.
Preliminary injunction is merely a provisional remedy, an adjunct to the main case subject to
As an exception to the rule, payroll reinstatement must rest on special circumstances that
the latter’s outcome, the sole objective of which is to preserve the status quo until the trial
render actual reinstatement impracticable or otherwise not conducive to attaining the
court hears fully the merits of the case. The status quo usually preserved by a preliminary
purposes of the law.
injunction is the last actual, peaceable and uncontested status which preceded the actual
controversy. The status quo ante litem is the state of affairs which is existing at the time of
The “superseding circumstances” mentioned by the Acting Secretary of Labor no doubt refer
the filing of the case. The trial court must not make use of its injunctive power to alter such
to the final decision of the panel of arbitrators as to the confidential nature of the positions of
status. In the case at bar, the RTC did not maintain the status quo when it issued the writ of
the twelve private respondents, thereby rendering their actual and physical reinstatement
preliminary injunction. Rather, it effectively restored the situation prior to the status quo, in
impracticable and more likely to exacerbate the situation. The payroll reinstatement in lieu of
effect, disposing the issue of the main case without trial on the merits. What was preserved
actual reinstatement ordered in these cases, therefore, appears justified as an exception to
by the RTC was the state OWWA, and the subsequent administrative orders pursuant to its
the rule until the validity of their termination is finally resolved. This Court sees no grave
passing. The RTC forgot that what is imperative in preliminary injunction cases is that the
abuse of discretion on the part of the Acting Secretary of Labor in ordering the same.
writ cannot be effectuated to establish new relations between the parties.
Furthermore, the issue has not been raised by any party in this case.
Courts should avoid issuing a writ of preliminary injunction which would in effect dispose of
Manila Diamond Hotel Employees’ Union vs CA, G.R.No. 140518, December 16, 2004
the main case without trial. In this case, the RTC also did not maintain the status quo but
Facts: The Union filed a petition for a certification election, which was dismissed by the
restored the landscape before the implementation of OWWA’s reorganization. In thus
DOLE. Despite the dismissal of their petition, the Union sent a letter to the Hotel informing the
issuing by the trial court. What was done by the RTC was quite simply a disposition of the
latter of its desire to negotiate for a collective bargaining agreement. The Hotel, however,
case without trial. Furthermore, we find that the RTC similarly prejudged the validity of the
refused to negotiate with the Union, citing the earlier dismissal of the Union’s petition for
issuances released by the OWWA Board of Trustees, as well as the other governmental
certification by DOLE.
bodies, which approved the organizational structure and staffing pattern of the OWWA. This
Court is not convinced that respondents were able to show a clear and unmistakable legal
Failing to settle the issue, the Union staged a strike against the Hotel. Numerous
right to warrant their entitlement to the writ. A mere blanket allegation that they are all
confrontations followed, further straining the relationship between the Union and the Hotel.
officers and employees of the OWWA without a showing of how they stand to be directly
The Hotel claims that the strike was illegal and dismissed some employees for their
injured a right in esse. There was no showing that Respondents are the employees who are
participation in the allegedly illegal concerted activity. The Union, on the other hand, accused
in grave danger of being displaced due to the reorganization. Injunction is not a remedy to
the Hotel of illegally dismissing the workers.
protect or enforce contingent, abstract, or future rights; it will not issue to protect aright not in
esse and which may never arise, or to restrain an action which did not give rise to a cause of
A Petition for Assumption of Jurisdiction under Article 263(g) of the Labor Code was later filed
action. Indeed, the question as to the validity of the OWWA reorganization remains the
by the Union before the Secretary of Labor. Thereafter, Secretary of Labor Trajano issued an
subject in the main case pending before the trial court. Its annulment is outside the realm of
Order directing the striking officers and members of the Union to return to work within twenty-
the instant Petition.
four (24) hours and the Hotel to accept them back under the same terms and conditions
prevailing prior to the strike.
Univ. of the Immaculate Conception vs. Sec. of Labor, G.R. Nos. 151379, Jan 14, 2005
Facts: This case stemmed from the collective bargaining negotiations between petitioner
After receiving the above order the members of the Union reported for work, but the Hotel
University of Immaculate Concepcion, Inc. (UNIVERSITY) and respondent The UIC Teaching
refused to accept them and instead filed a Motion for Reconsideration of the Secretary’s
and Non- Teaching Personnel and Employees Union (UNION). The UNION, as the certified
Order.
bargaining agent of all rank and file employees of the UNIVERSITY, submitted its collective
bargaining proposals to the latter on February 16, 1994. However, one item was left
Acting on the motion for reconsideration, then Acting Secretary of Labor Español modified the
unresolved and this was the inclusion or exclusion of some positions in the scope of the
one earlier issued by Secretary Trajano and instead directed that the strikers be reinstated
bargaining unit.
only in the payroll.
34

Issue: WON payroll reinstatement is proper in lieu of actual reinstatement under Article
263(g) of the Labor Code.

Held: Payroll reinstatement in lieu of actual reinstatement is not sanctioned under the
provision of the said article.

The Court noted the difference between UST vs. NLRC and the instant case. In UST case
the teachers could not be given back their academic assignments since the order of the
Secretary for them to return to work was given in the middle of the first semester of the
academic year.

The NLRC was, therefore, faced with a situation where the striking teachers were entitled to a
return to work order, but the university could not immediately reinstate them since it would be
impracticable and detrimental to the students to change teachers at that point in time.

In the present case, there is no similar compelling reason that called for payroll reinstatement
as an alternative remedy. A strained relationship between the striking employees and
management is no reason for payroll reinstatement in lieu of actual reinstatement.

Under Article 263(g), all workers must immediately return to work and all employers must
readmit all of them under the same terms and conditions prevailing before the strike or
lockout.

The Court pointed out that the law uses the precise phrase of “under the same terms and
conditions,” revealing that it contemplates only actual reinstatement. This is in keeping with
the rationale that any work stoppage or slowdown in that particular industry can be inimical to
the national economy.

The Court reiterates that Article 263(g) was not written to protect labor from the excesses of
management, nor was it written to ease management from expenses, which it normally incurs
during a work stoppage or slowdown. This law was written as a means to be used by the
State to protect itself from an emergency or crisis. It is not for labor, nor is it for management.

Porcello vs. Rudolf Lietz, G.R. No. 196539, October 10, 2012
Pacific Consultants vs. Schonfeld, G.R. No. 166920, February 19, 2007
Balite vs. SS Ventures International, Inc, G.R. No. 195109, February 4, 2015
SMART Comm vs. Solidum, G.R. Nos. 197763/197836, December 7, 2015
Milan vs. NLRC, G.R. No. 202961, February 4, 2015
Michelin Asia Application Center, Inc. vs Ortiz, G.R. No. 189861, November 19, 2014
Baronda vs. Court of Appeals, G.R. No. 161006, October 14, 2015
Montero vs. Times Transportation, G.R. No. 190828, March 16, 2015
One Shipping Corp vs. Peñafiel, G.R. No. 192406, January 21, 2015

You might also like